Sie sind auf Seite 1von 80

2009 Self-Assessment Examination

Preferred Responses, Discussions and References – Book 2


2009 Self-Assessment Examination
Overview
This examination is offered as a convenient self-education exercise by the
American Society for Surgery of the Hand and is available to the medical
profession, especially those health care professionals focusing on the care of the
hand and upper extremity. Designed to assist the physician in reviewing basic
principles of hand care, the exam also helps to keep the physician abreast of new
developments and concepts within the specialty.
The examination covers diagnostic and therapeutic problems, both surgical and
non-operative, basic science knowledge, and fundamental principles of hand
surgery. The interpretation of the illustrative material (clinical photographs and
radiographs) is an integral part of this examination.
The 2009 examination consists of the Question Book (Book 1) containing 150
questions, an answer sheet, evaluation, and the Discussion Book (Book 2) with
preferred responses, discussion and literature references.
The American Society for Surgery of the Hand guarantees anonymity of
examination results. Remember to record your examination number, as the
scoring will be by the number and not by name. Please keep a copy of
your answer sheet for your record; originals cannot be returned. It is
recommended that answers sheets be mailed via overnight or second-day
delivery so that the receipt of the score sheets can be verified.

Objectives
■ Detect and differentiate injury mechanisms, pathologies and applied anatomy
associated with upper extremity conditions treated by hand surgeons.
■ Differentiate and apply the range of non-operative and surgical treatment
options to deal with upper extremity problems that present to a hand
surgeon’s practice.
■ Develop informed evidenced-based guided clinical decision practice options
in the treatment of upper extremity disorders.
■ Evaluate, by self-assessment, basic cognitive abilities and clinical skills needed
by hand surgeons to successfully complete the maintenance of certification.

Copyright© 2009
Timelines
June 12, 2009
American Society for Answer sheets due back to the ASSH Office.
Surgery of the Hand
6300 North River Road July 31, 2009
Suite 600 Individual scores, Book 2, containing preferred responses, discussions, and
Rosemont, IL 60018-4256 references, and an evaluation mailed to participants.
(847) 384-8300
Please note: late submissions received at the central office after June 12, 2009,
Fax: (847) 384-1435
cannot be scored and will not be eligible for CME credit.
www.assh.org

Accreditation
The American Society for Surgery of the Hand is accredited by the Accreditation
Council for Continuing Medical Education to provide continuing medical
education for physicians.

2009 Self-Assessment Examination | 1


Credit Hours Disclosures
The ASSH designates this educational activity for a maximum • Seth D. Dodds, MD
of 20.00 AMA PRA Category 1 Credits™. Physicians should Integra: speaking and educational honoraria
only claim credit commensurate with the extent of their • Jay F. Pomerance, MD
participation in the activity. Consultant for Wyeth Pharmaceuticals, and
Answer sheets must be received by ASSH Central Office by Gerson Lehrman Group, LLP
June 12, 2009, to be eligible for Category 1 CME credit. • Lance A. Rettig, MD
Biomet Instructor/Faculty for surgeon cadaver courses
2009 Self-Assessment • David C. Ring, MD
Consultant for Small Bone Innovations, Wright Medical, Smith
Examination Committee and Nephew, Acumed, and Tornier. Research/Institutional
◆ John M. Bednar, MD support from Small Bone Innovations, Wright Medical, Smith
Committee Chair and Nephew, Acumed, Tornier (all unrestricted gifts). Research/
Institutional support from Joint Active Systems, American
◆ O. Alton Barron, MD
Foundation for Surgery of the Hand, Orthopaedic Trauma
◆ Kyle D. Bickel, MD, FACS
Association, and Stryker (grants to specific research). Royalties
◆ Louis W. Catalano, III, MD
from Hand Innovations. Honoraria/paid travel from Small Bone
◆ Fred G. Corley, MD
Innovations, Wright Medical, Smith and Nephew, Acumed,
Seth D. Dodds, MD
Tornier AC International, AO North America, Depuy, and several
◆ Scott G. Edwards, MD
universities. Ownership interests with Mimedex and Illuminoss.
◆ Harris Gellman, MD
◆ Harry A. Hoyen, MD • Russell Shatford, MD, MPH, MSEd
◆ Richard S. Idler, MD Owns stock in Minnesota Mining and Manufacturing (3M)
◆ Howard M. Matsuba, MD • Andrew L. Terrono, MD
◆ Kimberly K. Mezera, MD Part owner of Boston Out Patient Surgical Suites
◆ David T. Netscher, MD
◆ Miguel A. Pirela-Cruz, MD
Jay F. Pomerance, MD Disclaimer
◆ Ghazi M. Rayan, MD The material presented in this continuing medical education
Lance A. Rettig, MD program is being made available by the American Society
David C. Ring, MD for Surgery of the Hand for educational purposes only. This
◆ Marco Rizzo, MD material is not intended to represent the only, or necessarily
Russell Shatford, MD, MPH, MSEd the best, methods or procedures appropriate for the medical
◆ David J. Slutsky, MD situation discussed. Rather, this educational material is
◆ Curtis M. Steyers, MD intended to present an approach, view, statement, or opinion of
Andrew L. Terrono, MD the authors or presenters, which may be helpful, or of interest,
◆ Eric P. Thorson, MD to other practitioners.
◆ Montri D. Wongworawat, MD
The examinees agree to participate in this medical education
◆ David S. Zelouf, MD
program sponsored by ASSH with full knowledge and
◆ Bridget Treccia
awareness that they waive any claim that they may have
ASSH Staff Liaison
against ASSH for reliance on any information presented in
this examination.
◆ Nothing of value
The authors preceded by a ◆ have indicated that they have Confidentiality
not received anything of value in the form of: research or
The Self-Assessment Exam is the sole property of the
institutional support, stock or stock options, equipment or
American Society for Surgery of the Hand (ASSH) and
services, paid travel, royalties or as a consultant or employee
scores are confidential. Individual results will only be shared
of a commercial company or institution related directly or
with the test-taker. Each participant is assigned a number
indirectly to the subject of the program.
and no names are used in the scoring process to further
ensure confidentiality.

2 | American Society for Surgery of the Hand


1. A 17 year-old healthy woman presents with a 2. The clinical photos are those of a 48 year-old
three-year history of a progressive swan-neck construction worker who presented with
deformity that developed following a dorsal complaints of arm swelling and burning pain
laceration to the DIP of the long finger (open following a direct blow to his forearm, which
mallet finger). Initial treatment was simple occurred three months ago. He has not had
skin closure. At times, the PIP joint will lock in any diagnostic workup. The most appropriate
hyperextension. There is full passive motion of the diagnosis is:
finger. The most reliable surgical treatment is:
A. Complex regional pain syndrome
A. Distal intrinsic release B. Vascular thoracic outlet syndrome
B. SORL (Oblique retinacular ligament) reconstruction C. Factitious edema
C. Crossed intrinsic transfer D. Subclavian vein thrombosis
D. Interosseous slide E. Chronic compartment syndrome
E. Lumbrical release
Preferred Response: C
Preferred Response: B
Discussion: The patient is exhibiting a clear case of
Discussion: This is a secondary swan-neck deformity factitious edema that he was effectively masking by
following terminal extensor tendon disruption with always wearing a long-sleeved shirt to hide the stigmata
extensor forces now concentrated at the PIP joint. of a proximal humeral tourniquet he was using to
The patient is young, non-rheumatoid, and has PIP create the distal edema. While psychological testing is
joint hyperextension with intermittent locking. The appropriate, in-patient treatment is not indicated as the
best treatment for this is an SORL reconstruction, next step. Direct confrontation of the patient with the
which serves as a passive tether passing dorsal to truth is necessary, and no further medical diagnostic
the DIP joint and volar to the PIP joint. Active PIP joint workup or limb-specific treatment is necessary.
extension thereby extends the DIP joint while PIP joint
hyperextension is prevented by the volar tether. A distal References:
intrinsic release is appropriate for post-traumatic intrinsic 1. Taskaynatan MA, et al. Factitious disorders encountered in
patients with the diagnosis of RSD. Clin Rheumatol 2005;
contracture with isolated limited PIP flexion and a positive
24(5):521-6.
intrinsic tightness test. A crossed intrinsic transfer is
used when there is ulnar drift of the digits and notable 2. Louis DS, Lamp MK, Greene TL. The upper extremity and
psychiatric illness. J Hand Surg 1985; 10:687-693. (This paper
contracture of the ulnar intrinsics. The ulnar intrinsics are
reviews a large series of patients (33) who presented with a
released and transferred radially. The interosseous slide is variety of recognizable problems consistent with factitious illness.
used in cases of spastic contracture of the interosseous The follow-up averaged 4½ years. At the time this was written,
muscles. self-causation was suspected but not established as it is now.)

References:
1. Smith R. Intrinsic Contracture. In: Green DP, ed. Operative Hand
Surgery. 4th ed. New York: Churchill Livingstone, 1999.

2. Thompson JS, et al. The spiral oblique retinacular ligament.


SORL. J Hand Surg 1978; 3:482-487.

Question 2, Figure 1

Question 2, Figure 2

2009 Self-Assessment Examination | 3


3. A 20 year-old college baseball player presents 4. The patient in the photo is manifesting a clinical
with a two-month history of non-dominant hand deformity secondary to an isolated peripheral
pain, especially noticeable when batting. He was nerve injury. This nerve is derived from which
not aware of any discrete trauma to his hand. He cervical root(s)?
primarily complains of 2 months of non-dominant
A. C2
ulnar-sided hand pain with paresthesias in the ring
and small fingers. The radiological study most B. C3-C4
likely to identify the problem is: C. C5-C7
A. Chest radiograph D. C8
B. Elbow MRI E. T1
C. Scaphoid view of the wrist Preferred Response: C
D. Upper limb angiography
Discussion: The patient in the photo has dramatic
E. Wrist CT winging of the inferior scapula secondary to long thoracic
nerve palsy. The long thoracic nerve derives from the
Preferred Response: E
ventral rami of C5-C7. After conservative treatment failed,
Discussion: Hook of the hamate fractures are most the patient underwent split pectoralis transfer to the
commonly seen in golfers, baseball players, and racquet inferior scapular angle.
sport athletes. While a carpal tunnel view may detect
References:
many hamate fractures, those at the base of the hamate
are easily missed. A CT scan will readily detect the 1. Aval SM, et al. Neurovascular injuries to the athlete’s shoulder:
Part I. JAAOS 2007; 15(4):249-256.
fractures. A chest x-ray should be ordered when a patient
has ulnar nerve symptoms in the hand, shoulder pain 2. Wiater JM, Flatow EL. Long thoracic nerve injury. Clin Orthop
and a history of smoking to detect a pancoast tumor. An 1999; 368:17-27.
elbow MRI might detect some ulnar nerve pathology, but
the clinical presentation would likely be more specific for
cubital tunnel syndrome. A clenched fist PA view is useful
in diagnosing scapholunate ligament disruption.

References:
1. Hirano K, Inoue G. Classification and treatment of hamate
fractures. J Hand Surg 2005; 10(2-3):151-7.

2. Jacobson JA, et al. Golf and racquet sports injuries. Semin


Musculoskelet Radiol 2005; 9(4):346-59.

Question 4, Figure 1

4 | American Society for Surgery of the Hand


5. A 40 year-old healthy woman presents with
a history of sharp pain and cold sensitivity
beneath the nail plate of her dominant long finger.
There are no visible lesions or abnormalities, but
tenderness is reproducible with pressure on the
nail plate. MRI of the finger is shown in Figure 1,
and a lesion is found at surgery, as shown in
Figure 2. The most probable diagnosis is:
A. Ganglion cyst
B. Epidermal inclusion cyst
C. Glomus tumor
D. Neurofibroma
E. Giant cell tumor of tendon sheath
Question 5, Figure 1
Preferred Response: C

Discussion: Glomus tumors are benign hamartomatous


lesions most frequently seen in the subungual soft
tissues. The majority of patients have no visible signs
of a tumor and are diagnosed based on symptoms,
physical findings, and MRI. Exquisite tenderness to
localized pressure is the hallmark of physical examination
and is often coupled with very localized cold sensitivity.
T2 weighted MRI often reveals intense signal activity in
the area of tenderness, confirming the lesion. Findings
at surgery are most commonly those of a solitary solid
soft tissue mass in the subungual adventitia beneath
the sterile matrix with multiple vascular lumina. Ganglion
cysts most often occur adjacent to joint capsule or
tendon sheaths and are fluid filled, rather than solid.
Epidermal inclusion cysts are limited to the immediate Question 5, Figure 2
subcutaneous tissues and are not found beneath the nail
plate. Neurofibromas are rarely tender and not located in the
subungual soft tissues, but are located in neuroectodermal
tissues. Giant cell tumors typically present adjacent
to tendinous structures and appear as heterogeneous
multiloculated solid soft tissue masses on MRI.

References:
1. McDermott EM, Weiss A-PC. Glomus tumors. J Hand Surg 2006;
31A:1397-1400.

2. Bhaskaranand K, Navadgi BC. Glomus tumor of the hand.


J Hand Surg 2002; 27B:229-231.

2009 Self-Assessment Examination | 5


6. A healthy 30 year-old woman presents with
a painful index finger. Ulceration at the tip
has developed over several weeks. She has
severe cold sensitivity in the fingertip, which
has not responded to oral analgesics. Her
clinical appearance is shown in Figure 1, and
an arteriogram of the hand is shown in Figure 2.
The most appropriate treatment at this time is:
A. Surgical sympathectomy of the common digital and
proper digital arteries
B. Extremity warming, niacin, and a calcium-channel
blocker
C. Amputation and primary closure at the level of the
DIP joint Question 6, Figure 2
D. Oral antibiotics and close observation
E. Excision of the lesion and a thenar flap
7. A malunited fracture-dislocation of the PIP joint
Preferred Response: A is shown in Figure 1. The most appropriate donor
graft to restore congruous PIP joint alignment is:
Discussion: Raynaud’s disease occurs most often in
young women and can progress to digital ulceration A. A toe PIP joint transfer
and gangrenous necrosis. This patient presented with B. The contralateral PIP joint
ulceration and tissue loss, as depicted in Figure 1. Medical
C. Distal radius graft
management alone is unlikely to result in sufficient
improvement to prevent further tissue loss and gangrenous D. Dorsal hemi-hamate graft
changes at this late stage. Several reports have shown E. Iliac crest bone graft
that surgical sympathectomy improved local circulation
and allowed for ulcer healing in most cases after failure of Preferred Response: D
conservative measures, preventing amputation. Long- Discussion: PIP joint fracture-dislocations are highly
term results, however, have been equivocal, especially in complex and unstable injuries. Adequate reconstruction
patients with underlying collagen-vascular disease. requires reconstitution of a smooth and properly-
References: configured articular surface and reconstruction of a
stable volar buttress to correct dorsal subluxation. When
1. McCall TE, Petersen DP, Wong LB. The use of digital artery
sympathectomy as a salvage procedure for severe ischemia more than 50% of the articular surface is involved in
of raynaud’s disease and phenomenon. J Hand Surg 1999; the fracture, grafting techniques have been found to
24A:173-177. be superior to volar plate arthroplasty. Autologous toe
2. Wilgis EFS. Evaluation and treatment of chronic digital ischemia. PIP joint grafts have been used with some success, but
Ann Surg 1981; 193:693-698. the donor-site morbidity is considerable. Contralateral
PIP joint grafts also have unacceptable donor-site
considerations. Distal radius cortico-cancellous grafting
does not restore the proper alignment or smooth
articular cartilage necessary for PIP joint resurfacing.
An autologous hemi-hamate graft from the dorsal
articular surface of the 4th and 5th CMC joints has been
used with great success in several series. When properly-
fashioned and rigidly-fixed, it restores near-anatomic
configuration of the middle phalangeal base and provides
a stable volar buttress to correct dorsal subluxation of the
joint with mimimal donor-site morbidity.

Question 6, Figure 1

6 | American Society for Surgery of the Hand


References: 8. Treatment to correct the malunion shown in
1. Capo JT, Hastings H II, Choung E, Kinchelow T, Rossy W, Figure 1 is likely to:
Steinberg B. Hemicondylar hamate replacement arthroplasty
A. Produce no improvement in patient outcome
for proximal interphalangeal joint fracture dislocations: An
assessment of graft suitability. J Hand Surg 2008; 33A:733-739. B. Improve arc of motion and decrease symptoms
2. Williams RMM, Kiefhaber TR, Sommerkamp TG, Stern PJ. C. Improve motion but not symptoms
Treatment of unstable dorsal proximal interphalangeal fracture/
D. Improve symptoms but not motion
dislocations using a hemi-hamate autograft. J Hand Surg 2003;
28A:856-865. E. Improve appearance of the wrist with no impact
on symptoms

Preferred Response: B

Discussion: Dorsal angulation of distal radius fractures is


a common deformity which alters the normal kinematics
of wrist motion. Increased wrist extension and decreased
flexion can lead to adaptive midcarpal instability and
increasing symptoms over time. Surgical correction of
dorsal angulation has been shown to improve both the
arc of motion and symptoms due to midcarpal instability.

References:
1. Taleisnik J, Watson HK. Midcarpal instability caused by malunited
fractures of the distal radius. J Hand Surg 1984; 9A:350-357.
Question 7, Figure 1
2. Lichtman DM, Schneider JR, Swafford AR, Mack GR. Ulnar
midcarpal instability – clinical and laboratory analysis. J Hand
Surg 1981; 6:515-523.

3. Batra S, Gupta A. The effect of fracture-related factors on the


functional outcome at 1 year in distal radius fractures. Injury
2002; 33:499-502.

4. Grewal R, MacDermid JC. The risk of adverse outcomes in extra-


articular distal radius fractures is increased with malalignment in
patients of all ages but mitigated in older patients. J Hand Surg
2007; 32A:962-970.

Question 7, Figure 2

Question 7, Figure 3

Question 8, Figure 1

2009 Self-Assessment Examination | 7


9. A 65 year-old woman presents with inability to 10. A 13 year-old boy suffered multiple gunshot wounds
extend the ring and small finger MCP joints of to his wrist and hand. He presents 1 year after injury
her left hand. Radiographs are shown in Figure 1. with a wrist flexion contracture and loss of active
Appropriate treatment is: digital flexion in all fingers. Clinical exam and
electrodiagnostic studies demonstrate absent
A. Extension splinting of the ring and small fingers
median nerve function distal to the wrist. The bony
B. Two-stage tendon grafting of the ring and small skeleton is intact. Appropriate treatment is:
finger extensor tendons
A. Scar debridement, skin grafting, and carpal
C. Total wrist fusion
tunnel release
D. Primary tendon repairs of the ring and small finger
B. Scar debridement, flexor tenolysis, neuroma
extensor tendons
resection and segmental grafting to the median
E. Resection of the distal ulna and tendon reconstruction nerve defect, with z-plasty closure of skin
Preferred Response: E C. Wrist extensor to digital flexor transfer, wrist fusion,
and carpal tunnel release
Discussion: Rheumatoid arthritis often affects the wrist
D. Scar and flexor muscle debridement, free
causing tenosynovitis of the extensor tendons, disruption microvascular active muscle and nerve transfer to
of the radioulnar ligaments, and distal radioulnar the flexor compartment, with skin grafting
joint capsule. Instability and erosion of the distal ulna
E. Amputation and myoelectric prosthesis to the wrist
contribute to rupture of the extensor digitorum communis
and hand
and extensor digiti minimi tendons. Successful treatment
involves correction of the distal ulnar prominence with a Preferred Response: B
Darrach resection and tendon transfer to restore digital
extension. Primary and secondary tendon repairs are Discussion: Traumatic ischemic contractures of the
not feasible with rheumatoid tendon ruptures due to upper extremity are classified by etiology and severity.
inflammatory destruction of the tendon substance and Type I injuries result from injury to vascular structures
retraction of the proximal tendon ends. proximal to the affected fascial compartment with distal
ischemic contracture. Type II injuries result from direct
References: trauma to tissues including muscles, nerves, and vessels.
1. Klemens T. Treatment of the wrist in rheumatoid arthritis. Both groups are further classified as mild, moderate, and
J Hand Surg 2008; 33A:113-123. severe depending on the extent of local tissue and nerve
2. Nalebuff E. Surgical treatment of tendon rupture in the involvement. The present case (Figure 1) is an example of
rheumatoid hand. Surg Clin North Am 1969; 49:811-822. a Type II – Moderate contracture with contracture of the
3. Ishikawa H, Hanyu T, Tajima T. Rheumatoid wrists treated with deep and superficial flexor muscles and the median nerve
synovectomy of the extensor tendons and the wrist joint combined resulting from direct injury.
with a Darrach procedure. J Hand Surg 1992; 17A:1109-1117.
Appropriate treatment requires exploration and
determination of the degree of necrosis and fibrosis of
the involved tissues. In this case, (Figure 2) debridement
of cutaneous and fascial scar, tenolysis of the flexor
tendons, resection of the neuroma of the median nerve
with segmental nerve grafting, and z-plasty of the skin
were performed. This allowed return of active digital
flexion, protective sensation of the median nerve, and
release of the flexion contracture, without the need for
free tissue transfer and with preservation of the intact
musculotendinous units.

Question 9, Figure 1

8 | American Society for Surgery of the Hand


References: 11. A patient presents with symptoms of severe cold
intolerance and joint stiffness progressing over
1. Gulgonen A. Compartment Syndrome. In: Green DP, Hotchkiss
RN, Pederson WC, Wolfe SW (eds): Green’s Operative Hand 1 to 2 years’ duration. Physical findings include
Surgery. 5th ed. Philadelphia: Churchill Livingston/Elsevier, 2005: digital pallor, sclerodactyly, multiple distal fingertip
1996-2006. ulcerations, severe PIP and DIP joint stiffness, and
2. Holden CEA. The pathology and prevention of Volkmann’s telangectasias. Likely laboratory findings are:
ischemic contracture. J Bone Joint Surg 1979; 61B:296-300. A. Positive rheumatoid factor and negative ANA
3. Tsuge K. Treatment of established Volkmann’s contracture. B. Positive rheumatoid factor and positive ANA with a
J Bone Joint Surg 1975; 57A:925-929.
speckled pattern
C. Negative rheumatoid factor and positive ANA with a
homogenous pattern
D. Positive rheumatoid factor and positive ANA with a
nucleolar pattern
E. Negative rheumatoid factor and negative ANA

Preferred Response: D

Discussion: The clinical scenario presented is one


of scleroderma involving the hands, with Raynaud’s
Question 10, Figure 1 phenomenon, ulceration, sclerodactyly, contractures, and
telangectasias. Common laboratory findings in diffuse
scleroderma include a positive ANA test with a nucleolar
pattern in 60-90% of patients and a positive rheumatoid
factor in 15-30%. Further ANA subtype testing can
differentiate between limited and diffuse scleroderma,
which is more likely to produce severe hand involvement.
The homogenous pattern of ANA fluorescence is most
commonly associated with systemic lupus erythematosus.
The speckled pattern is most often seen in mixed
connective tissue disease and Sjogren’s syndrome, but
can be seen in other connective tissue diseases as well.

References:
Question 10, Figure 2
1. Serup J, Staun-Olsen P. Antinuclear antibodies and anti-DNA
antibodies in scleroderma. Allergy 1986; 41:452-456.

2. Jolly M, Smaron M, Utset TO, Ellman M. Are isolated antinuclear


antibodies a marker of scleroderma? J Clin Rheumatol 2003;
9:291-295.

2009 Self-Assessment Examination | 9


12. The fracture of the thumb proximal phalanx seen 13. Surgical treatment of stroke patients often involves
in Figure 1 is best treated by: the release of which of the following contractures?
A. Reduction and casting A. Elbow flexion/forearm supination/
B. Reduction and pin fixation wrist and finger extension

C. Open reduction and pin fixation B. Elbow flexion/forearm supination/


wrist and finger flexion
D. Open reduction and rigid internal fixation
C. Elbow flexion/forearm pronation/
E. Open reduction, internal fixation, and bone grafting wrist and finger flexion
Preferred Response: E D. Elbow extension/forearm supination/
wrist and finger extension
Discussion: This is a pathologic fracture at the site of
E. Elbow extension/forearm pronation/
an enchondroma of the proximal phalanx. The tumor
wrist and finger flexion
has replaced the medullary bone in the majority of the
phalanx and thinned the cortex, weakening the bone and Preferred Response: C
precipitating the fracture. Stable extra-articular pathologic
fractures of the phalanges are best treated with Discussion: 80-90% of patients survive their cerebral
immobilization alone, until fracture healing restores some vascular accidents, frequently resulting in significant
stability to the bone followed by excavation and grafting disability. Spontaneous neurologic recovery can occur
of the tumor. This fracture is an articular fracture of the for up to six months following the stroke. After this time,
MCP joint with substantial displacement. Open reduction surgery is performed to release spastic muscles and joint
of the articular surface is necessary to minimize the risk contractures, as well as use tendon transfers to position
of traumatic arthritis. The best treatment is therefore the involved extremity in a more functional position.
open reduction, internal fixation, and curettage with bone Common joint contractures include shoulder adduction,
grafting of the phalanx. elbow flexion, forearm pronation, wrist and finger flexion,
intrinsic muscle spasticity, and thumb-in-palm deformity.
References:
References:
1. Todai P, Hoglund M, Lugnegard H. Is the treatment of
enchondroma of the hand by simple curettage a rewarding 1. Tafti MA, Cramer SC, Gupta R. Orthopaedic management of the
method? J Hand Surg 1990; 15B:331-334. upper extremity of stroke patients. J Am Acad Orthop Surg 2008;
16(8):462-470.
2. Wulle C. On the treatment of enchondroma. J Hand Surg 1990;
15B:320-330. 2. Pomerance JF, Keenan MA: Correction of severe spastic flexion
contractures in the nonfunctional hand. The J Hand Surg 1996;
21:828-833.

Question 12, Figure 1

10 | American Society for Surgery of the Hand


14. A 30 year-old recreational boxer injured his right Discussion: A recent Cochrane Review explored the
hand throwing a “really hard punch” two days benefits of silicon gel sheeting in the treatment of
ago. Physical examination revealed tenderness hypertrophic scars and keloids. Thirteen studies were
and moderate ballotable fullness over the dorsal included. The authors concluded that silicone sheeting
aspect of the long finger MP joint with EDC reduced the incidence of hypertrophic scarring in people
tendon subluxation during MP joint flexion. prone to scarring and improved scar elasticity, but could
Active MP joint ROM was 20º to 90º. Radiographs find no cosmetic benefit in postoperative scarring.
revealed no fractures. Treatment should consist of The authors did emphasize that all studies were highly
the following: susceptible to bias and most were of poor quality.
A. Observation References:
B. Aspiration of the fluid 1. O’Brien L, Pandit A. Silicon gel sheeting for preventing and
treating hypertrophic and keloid scars. Cochrane Database Syst
C. Dynamic MP joint extension splinting for six weeks
Review 2006: Jan.
D. Operative treatment with repair of the MP joint
2. Tan E, Chua SH, Lim JTE. Topical silicone gel sheet versus
dorsal capsule
intralesional injections of triamcinolone acetonide in the
E. Operative treatment with repair of the sagittal band treatment of keloids – a patient-controlled comparative clinical
trial. Journal of Dermatological Treatment 1999; 10(4):251-4.
Preferred Response: E

Discussion: This clinical scenario describes a boxer


16. The long-term results of intra-articular distal
with a dorsal MP joint capsular tear (the ballotable
radius fractures treated operatively demonstrate:
fullness over the MP joint) and a sagittal band rupture
(EDC tendon subluxation with MP joint flexion). In this A. Frequent, disabling pain
scenario, repair of the sagittal band is recommended. B. Significant grip strength weakness
Controversy exists about the need for concomitant repair
C. High levels of patient function and satisfaction
of the dorsal capsular tear as over-tightening the dorsal
capsule may limit post-operative MP joint flexion. Acute D. No evidence of radiocarpal joint space narrowing
sagittal band ruptures without dorsal capsular tears in E. Poor wrist motion producing functional limitations
lower-demand patients have been successfully treated
with customized splinting. Preferred Response: C

References: Discussion: Intra-articular distal radius fractures are


one of the most common injuries that a hand surgeon
1. Hame SL, Melone CP Jr. Boxer’s knuckle in the professional evaluates. Since the fracture involves the radiocarpal
athlete. Am J Sports Med 2000 Nov-Dec; 28(6):879-82.
and/or the distal radio-ulnar joints, obvious concern
2. Catalano LW, Gupta S, Ragland R, Glickel SZ, Johnson C, Barron exists about the development of premature wrist arthritis.
OA. Closed treatment of non-rheumatoid extensor dislocations at A recent study by Goldfarb et al provides data that
the metacarpophalangeal joint. J Hand Surg 2006; 31A:242-245.
the surgeon can communicate to the patient to allay
patient fears. At a mean of 15-year follow-up, 14/16
patients functioned at a high level and the average
15. Application of silicone gel sheeting to a scar:
Musculoskeletal Functional Assessment score was 10
A. Will eliminate the scar hypertrophy (a normal score). Strength and range of motion were
B. Is more effective than steroid injections unchanged from the 7-year follow-up study. Despite the
high level of patient function, radiographic evidence of
C. Is more effective when coated with Vitamin E
wrist arthrosis was noted in 13/16 wrists.
D. Is more effective than non-silicon gel sheeting
References:
E. Provides no cosmetic benefit
1. Goldfarb CA, Rudzki JR, Catalano LW III, Hughes M, Borrelli J Jr.
Preferred Response: E Fifteen-Year outcomes of displaced intra-articular fractures of the
distal radius. J Hand Surg 2006; 31A:633-639.

2. Catalano LW III, Cole RJ, Gelberman RH, Evanoff BA, Gilula LA,
Borrelli J Jr. Displaced intra-articular fractures of the distal aspect
the radius. J Bone Joint Surg 1997; 79A:1290-1302.

2009 Self-Assessment Examination | 11


17. Which flap is most appropriate to treat the soft
tissue defect shown in Figure 1?
A. Kite flap
B. Axial flag flap
C. Digital artery island flap
D. Fillet flap
E. Reversed cross finger flap

Preferred Response: C

Discussion: This skin defect required some type of flap


coverage due to exposed flexor tendon. The digital artery
island flap (Figure 2) is raised based upon the radial or Question 17, Figure 1
ulnar digital artery/veins of the long or ring fingers (the
index and small fingers have small radial and ulnar digital
arteries, respectively, so harvesting a digital artery in
these digits can lead to cold intolerance). The flap is
dissected proximally and can be rotated to cover defects
along the length of the involved or adjacent digit (Figure
3). The Kite flap is a proximally-based flap based on
the first dorsal metacarpal artery and is raised from the
distal-radial portion of the index finger metacarpal. It is
used for thumb MP/IP joint skin defects. The axial flag
flap is based upon a dorsal digital artery of any digit
and is elevated from the dorsal aspect of the donor digit
proximal phalanx. It can be rotated to cover adjacent
finger dorsal or volar MP joint skin defects. A fillet flap
Question 17, Figure 2
obtains skin from a severely damaged finger and involves
removing the bone and tendons from the injured digit
leaving vascularized skin for coverage. The reverse cross
finger flap is used to cover defects on the dorsum of digits.
It involves harvesting a flap of subcutaneous tissue at the
middle phalanx from an adjacent finger and rotating the
flap to cover the defect. A full-thickness skin graft is then
used to cover the flap of subcutaneous tissue.

Reference:
1. Rose EH. Local arterialized island flap coverage of difficult hand
defects preserving donor digit sensibility. Plast Reconstr Surg
1983; 72:848-857.

Question 17, Figure 3

12 | American Society for Surgery of the Hand


18. Proper treatment of extravasation of Yeasts (e.g., Candida) are most common in the fingernail
Vincristine includes: and represent up to 33% of fingernail onychomycosis.
Multiple trials have demonstrated that terbinafine is
A. Application of ice packs
the most effective treatment for infections caused by
B. Local injection of dimethyl sulfoxide (DMSO) dermatophytes. However, for Candida infections, a
C. Hot compresses and local injection of hyaluronidase recent randomized trial revealed a 92% cure rate with
D. I.V. administration of sodium thiosulfate itraconazole versus a 40% cure rate with terbinafine.
These studies highlight the importance of culturing
E. Subcutaneous injection of sodium thiosulfate
the nailplate and/or debrided tissue prior to starting
Preferred Response: C the drug treatment. Debridement of the nailplate in
addition to terbinafine has been shown to result in
Discussion: Unfortunately, extravasation of cancer
higher cure rates that terbinafine alone in the treatment
chemotherapeutic agents is a frequently-encountered
of onychomycosis.
clinical scenario that can potentially cause permanent
local injury. Initial treatment regardless of the type of References:
agent involves immediately stopping the infusion and 1. Mishra M, Panda P, Tripathy S, Sengupta A, Mishra K. An open
splinting/elevation of the involved extremity. Although randomized comparative study of oral itraconazole pulse and
it is unreasonable to memorize all of the specific terbinafine pulse in the treatment of onychomycosis. Indian J
antidotes for each agent, some basic principles should dermatol verereol Leprol. 2005; 71:262-266.
be committed to memory. Heat is applied for vinca 2. Nunley KS, Cornelius L. Current management of onychomycosis.
alkaloid (e.g., vincristine) extravasation to produce J Hand Surg 2008; 33A:1222-1214.
vasodilation and increase blood flow to the area. Ice
packs are used for anthracycline (e.g., doxorubicin)
extravasation. Use of heat for anthracycline and ice for 20. Women undergoing elective hand surgery
vinca alkaloid extravasation may result in harmful effects. following an ipsilateral axillary node dissection
Additional treatment of vinca alkaloid extravasation should be informed that:
includes injection of hyaluronidase around the site
A. There is a higher risk of postoperative infection
of extravasation. DMSO injections are used to treat
anthracycline extravasation. Sodium thiosulfate is B. New lymphedema will likely occur
injected locally for extravasation of alkylating agents such C. Delayed wound healing is probable
as cisplatin or nitrogen mustard. D. Surgery can be performed without
References: additional complications
1. Schrijvers DL. Extravasation: a dreaded complication of E. A sympathetically mediated pain syndrome is
chemotherapy. Annals of Onc. 2003; 14 (supplement 3):iii26-iii30. more likely
2. Ener RA, Meglathery S, and Styler M. Extravasation of systemic Preferred Response: D
hemato-oncological therapies. Annals of Onc. 2004; 15:858-862.
Discussion: Breast cancer survivors are frequently told to
have no interventions on the ipsilateral side of the axillary
19. The most effective treatment of onychomycosis node dissection. This poses a major problem when the
caused by Candida infections is: patient requires surgery on the affected side. Two studies
A. Terbinafine have clearly demonstrated that there is no additional risk
of infection, delayed wound healing, development of a
B. Itraconazole
pain syndrome or new lymphedema after elective surgery
C. Mechanical removal of the nail plate alone on an extremity having undergone a prior ipsilateral
D. Griseofulvin axillary node dissection.
E. Amphotericin B References:
Preferred Response: B 1. Hershko DD and Stahl S. Safety of elective hand surgery
following axillary node dissection for breast cancer. Breast J.
Discussion: Onychomycosis is a common clinical 2007; 13(3):287-290.
condition and can substantially negatively impact a 2. Dawson WJ, Elenz DR, Winchester DP, Feldman JL. Elective
patient’s quality of life. Onychomycosis is caused by hand surgery in the breast cancer patient with prior ipsilateral
dermatophytes, yeasts, and non-dermatophyte molds. axillary dissection. Ann Surg Oncol 1995; 2(2):132-137.

2009 Self-Assessment Examination | 13


21. Acrosyndactyly (Figure 1), or fusion of the terminal 22. A study in which patients are recalled and
portion of two or more digits with proximal sinuses compared on the basis of two different types
between the digits, is seen in: of treatment (e.g., pinning versus ORIF of distal
radius fractures) is called a:
A. Constriction band syndrome
B. Poland’s syndrome A. Randomized controlled study

C. Cleft hand B. Non-randomized, non-blinded controlled study

D. Simple, incomplete syndactyly C. Retrospective comparative study

E. Simple, complete syndactyly D. Case-control study


E. Case series
Preferred Response: A
Preferred Response: C
Discussion: Acrosyndactyly is a typical feature of
Constriction Band Syndrome (Streeter’s dysplasia). This Discussion: As a result of the recent emphasis on
disorder occurs when amniotic bands wrap around the evidence base medicine, all surgeons should be familiar
digits producing simple and/or complex syndactyly, with the different types of therapeutic studies that
digital shortening, and potentially, acute vascular generate five different levels of evidence. Level I evidence
compromise of the digit. Acrosyndactyly arises from consists of high-quality, randomized, controlled trials with
normal developing fingertips being fused together by statistical significance and narrow confidence intervals.
the amniotic bands while the proximal aspect of the web Studies that generate Level II evidence are lesser-quality
space remains patent, resulting in the interdigital cleft. controlled trails and prospective comparative studies.
Level III evidence is generated by case-control and
Reference: retrospective comparison studies. A case control study is
Flatt AE. The Care of Congenital Hand Anomalies. St. Louis: Quality a study in which patients are enrolled on the basis of an
Medical Publishing, 1994:276-291.
outcome (cases) and compared to patients who did not
have that outcome (controls). A retrospective comparative
study is defined as a study that was started after the first
patient was enrolled and compares patients treated by
two different techniques at the same institution. Level IV
evidence arises from case series and Level V evidence
consists of expert opinion.

Reference:
Phillips B, Bell C, Sackett D, Badenech B, Straus S, Haynes B,
Dawes M. Oxford Centre for Evidence-Based Medicine Levels of
Evidence. May 2001.

23. The patient seen in Figures 1, 2 and 3 has a


long‑standing ulnar nerve injury above the elbow
and is being evaluated for correction of his claw
Question 21, Figure 1 finger deformity. What is the most appropriate
tendon transfer?
A. ECRL dorsal to the intermetacarpal ligament
B. FDS of the ring finger dorsal to the
intermetacarpal ligament
C. FDS of the ring finger volar to the
intermetacarpal ligament
D. EIP volar to the intermetacarpal ligament
E. EIP dorsal to the intermetacarpal ligament

Preferred Response: D

14 | American Society for Surgery of the Hand


Discussion: The cause of clawing from an ulnar nerve
palsy (MP joint hyperextension and PIP joint flexion) is a
loss of intrinsic muscle function. The force of the extrinsic
extensor is dissipated at the MP joint so all operations for
clawing are designed to prevent MP joint hyperextension.
Figure 3 demonstrates a Fowler’s test in which you test
active PIP extension while preventing MP hyperextension.
If you can get full PIP extension with Fowler’s test, there
is a good prognosis for any operation for ulnar clawing.

Although all of the above responses are recognized


transfers for claw finger deformity, the appropriate transfer Question 23, Figure 3
in this patient with this history is a transfer of the extensor
indicis proprius volar to the inter-metacarpal ligament.
A volar capsulorraphy in a person with good extensor 24. Surgical correction of a PIP joint contracture in a
tendons available to his MP joints will usually stretch out patient with a long standing Dupuytren’s disease
after time. If the flexor digitorum sublimis tendon of the ring results in a passive correction at the time of
finger is used, there is no other tendon to flex the finger. surgery to 15º from a 95º fixed flexion contracture.
One of the basic tenants of tendon transfer is to transfer Postoperatively, the patient is unable to extend the
the tendon volar to the axis of the metacarpophalangeal PIP joint actively. What is the most likely cause?
joint and this is accomplished by transferring the tendon
volar to the intermetacarpal ligament. A. Failure to adequately release the deforming cords
B. Failure to adequately release the volar plate
References:
C. Shortening of the flexor digitorum profundus
1. Anderson, GA. Ulnar nerve palsy. In: Green DP, Hotchkiss R,
Pederson W, Wolfe S eds. Operative Hand Surgery. 5th ed.
and superficialis
Philadelphia, Churchill Livingston, 2005:1168-1183. D. Contracture and adhesions of the collateral and
2. Hastings, Hill II, Davidson S(RN). Tendon transfers for ulnar nerve accessory collateral ligaments
palsy. Hand Clin 1988; 4(2):167-178. E. Attenuation of the extensor mechanism

Preferred Response: E

Discussion: Although all of these responses can


contribute to flexion contracture of the PIP joint in
Dupuytren’s disease, attenuation of the extensor
mechanism after long-standing PIP joint flexion
contracture is a common cause of an inability to actively
Question 23, Figure 1 extend the joint postoperatively. Intraoperatively, passive
extension was obtained to 15° after releasing the
appropriate structures. The patient, after surgical release,
could not actively extend the PIP joint, secondary to
attenuation of the central slip of the extensor tendon.
In long-standing Dupuytren’s contracture and PIP
joint flexion contracture, the central slip is commonly
stretched and attenuated over the PIP joint. If this is
not recognized, active extension of the PIP joint is not
restored despite release of the offending structures.

References:
1. Andrew JG. Contracture of the proximal interphalangeal joint in
Dupuytren’s disease. J Hand Surg 1991; 16B:446.

2. Crowley B, Hawkin MA. The proximal interphalangeal joint in


Dupuytren’s disease. Hand Clin 1999; 15(1):137-148.

Question 23, Figure 2

2009 Self-Assessment Examination | 15


25. Cryotherapy for the treatment of injury has been References:
shown to: 1. Lister GD. The theory of the transposition flap and its practical
application in the hand. Clin Plast Surg 1981; 8:115-127.
A. Prevent local hemorrhage
2. Lister GD. Local flaps in the hand. Hand Clin 1985; 1(4):621-647.
B. Prevent edema
C. Reduce tenderness and hematoma
D. Decrease pain
E. Increase muscle spasm

Preferred Response: D

Discussion: Cold affects inflammatory responses


following soft tissue injury by reducing enzymatic
function and decreasing metabolism. It produces
vasoconstriction, which may alter the extension of a
hematoma but does not prevent local hemorrhage after
an injury. Edema is not prevented by cold application.
Muscle spasm is reduced by cold application through
its effect on the muscle spindle. Cold relieves pain Question 26, Figure 1 Question 26, Figure 2

through its effect on the pain fibers. Pain relief from


cold is caused by blocking the sensory transmission of
the pain impulses caused by slowing or elimination of
nerve conduction. Tenderness or soreness in the muscle
caused by palpation is not affected by cold.

References:
1. Matsen FA, Questad K, Matsen AL, et al. The effect of local
cooling on post-fracture swelling. Clin Orthop Relat Res 1975;
109:201.

2. Meeusen R, Lievens P. The use of cryotherapy in sports injuries.


Sports Med 1986; 3:398-414.

Question 26, Figure 3 Question 26, Figure 4


26. A 60 year-old patient with a soft tissue defect has
a rotation flap for coverage, as seen in Figures 1-5.
The survival of the skin graft placed on the defect
created by the rotation flap is dependent on:
A. Preservation of a vascular bed
B. The size of the defect that accepts the graft
C. Blood supply to the flap itself
D. The length of the flap
E. The width of the flap

Preferred Response: A

Discussion: The figures depict a transposition flap on the


dorsum of the hand involving the extensor tendons. The Question 26, Figure 5
survival of the flap depends upon the blood supply which,
in turn, is dependent on the anatomy, the length, width and
breadth of the flap and the tension under which it is sutured.

Preservation of the paratenon over the extensor tendon is


crucial in allowing the skin graft to survive and mature.

16 | American Society for Surgery of the Hand


27. A 40 year-old woman with breast cancer 28. A 49 year-old man has a diagnosis of acute
complains of chronic fatigue, pain in multiple gouty arthropathy in his wrist. This is his first
joints and muscles, and a rash over her hand with symptomatology. In addition to splinting and ice
painful MCP and PIP joints. Her laboratory studies packs, the most appropriate initial drug treatment
show a markedly elevated SGOT, CPK and SGPT. is which of the following?
The image of her hand is seen in Figure 1. What is
A. Oral colchicine
the most likely diagnosis?
B. Intravenous colchicine
A. Rheumatoid arthritis
C. Non-steroidal anti-inflammatory drugs
B. Scleroderma
D. Corticosteroids orally
C. Systemic lupus erythematosus
E. Intraarticular corticosteroids
D. Dermatomyositis
Preferred Response: C
E. Polymyositis
Discussion: Under most circumstances, the primary
Preferred Response: D
treatment for acute gouty arthritis is a non-steroidal
Discussion: Dermatomyositis has a characteristic rash anti-inflammatory drug or a cox-2 inhibitor. Oral or
over the eyelids and the dorsum of the hand. It is also intraarticular glucocorticoids are the second choice.
often associated with malignancy, particularly abdominal
Colchicine, either orally or intravenously, is the third
sarcomas, lymphomas and breast cancer. Elevated
choice because of the significant toxicity associated
muscle enzymes, fatigue and weakness are characteristic
with colchicine. Gastrointestinal side effects, renal
of dermatomyositis.
dysfunctions, and bone marrow suppressions all occur
Gottron’s papules are symmetric, lacy, pink to violaceous with colchicine administration.
raised or macular areas found on the dorsum of the
References:
MP and the PIP joints, the elbow, patella and the
medial malleoli. Periungual telangiectasias and capillary 1. Wortmann RL. Gout and hyperuricemia. Current Opinion in
Rheum 2002; 14:281-286.
changes are also seen about the hand, particularly
along the dorsum of the hand. The characteristic rash of 2. Roberts W, Liang MH, Stern SH. Colchicine in acute gout. JAMA
dermatomyositis is described as being a violaceous rash 1987; 257:1920-1921.
present around the eyes.

References:
1. Callen J. Dermatomyositis. Lancet 2000; 255:53-57.

2. Euwer RL, Sontheimer RD, Braverman IM. Dermatomyositis


and polymyositis. In: Demis DJ, ed. Clin Dermatol. 1st ed.
Philadelphia: JB Lippincott, 1994:1-14.

Question 27, Figure 1


2009 Self-Assessment Examination | 17
29. A 35 year-old man sustains the injury seen in
Figures 1 and 2. Which wrist ligament remains
intact in this injury?
A. Scapholunate
B. Lunotriquetral
C. Dorsoradial carpal
D. Short radiolunate
E. Long radiolunate

Preferred Response: D

Discussion: In the progression of perilunate instability,


in stage 1, the scapholunate ligaments fail—usually from Question 29, Figure 2
the palmar aspect to the dorsal aspect. In stage 2, there
becomes a progressive lunocapitate disassociation as
the distal row of carpal bones follow the capitate dorsally,
allowing the lunate to open the space of Poirier. 30. The rate at which an action potential is conducted
along an axon is increased with the presence of:
In stage 3, the lunotriquetral ligament is torn or an
A. Nodes of Ranvier
avulsion fracture of the triquetrum occurs. Also in stage
3, with a complete rupture of the lunotriquetral ligament, B. Local anesthetic
the ulnar expansions of the long radiolunate ligament C. Decreased axon diameter
usually tear, leaving the short radiolunate ligament and D. Nerve compression
the ulnolunate ligament as the lunate’s only stabilizing
E. Increased age of the patient
forces as it continues to rotate volarly.
Preferred Response: A
References:
1. Green DP, Hotchkiss RN, Pederson WC, Wolfe SW, eds. Discussion: The presence of myelin, Nodes of Ranvier,
Green’s Operative Hand Surgery. 5th ed. Philadelphia: Churchill and larger axon diameter, all increase the rate of a nerve’s
Livingston/Elsevier, 2005: ch.14.
action potential. Nerve conduction slows with increased
2. Mayfield JK, Johnson RP, Kilcoyne RK. Carpal dislocations: age. Local anesthetics, such as lidocaine and marcaine,
pathomechanics and progressive perilunar instability. J Hand cause a nerve conduction block which causes slowing or
Surg 1980; 5A:2226-41.
interruption of nerve conduction.

References:
1. Bodine SC, Lieber RL. Peripheral nerve physiology, anatomy,
and pathology. In: Orthopaedic Basic Science, Buckwalter
JA, Einhorn TA, Simon SR, ed. 2000, American Academic of
Orthopaedic Surgeons. p. 617-682.

2. Ramamurthy S, Anderson D. Anesthesia. In: Green DP, Hotchkiss


RN, Pederson WC, Wolfe SW, eds. Green’s Operative Hand
Surgery. 5th ed. Philadelphia: Churchill Livingston/Elsevier,
2005:25-30.

Question 29, Figure 1

18 | American Society for Surgery of the Hand


31. Considering the physiology of bone healing, Discussion: Similar to SLAC wrist arthritis, SNAC wrist
scaphoid fractures heal by: arthritis can be staged:
A. Primary bone healing Stage I – degeneration of the radial styloid and radial
B. Secondary bone healing aspect of the scaphoid
C. Callus formation Stage II – degeneration progresses to the level of the
D. Internal fixation scaphoid fracture and involves the scaphocapitate joint
E. Enchondral ossification Stage III – degeneration of the capitolunate joint
Preferred Response: A Stage IV – pan-carpal arthrosis
Discussion: Scaphoid fractures do not make callus (nor Scaphoid nonunion, in this case, has led to flexion of
do they heal by secondary bone healing). They heal by the distal scaphoid and radioscaphoid arthritis with
primary bone healing with bridging trabeculae. degeneration of the scaphoid facet of the radius and the
References: scaphocapitate joint. In Stage II, degeneration attempts
to achieve healing at the scaphoid will not treat the
1. Singh HP, Forward D, Davis TRC, Dawson JS, Oni JA, Dowing
ND. Partial Union of acute scaphoid fractures. J Hand Surg 2005; arthritis of the scaphocapitate joint and radial styloid.
30B:5:440–445. Scaphoid excision with fusion of the capitate and lunate
or four-corner fusion can provide reliable pain relief for
2. Dias JJ. Definition of union after acute fracture and surgery
for fracture nonunion of the scaphoid. J Hand Surg 2001; radioscaphoid arthritis secondary to scaphoid nonunion.
26B:321–325.
References:
1. Enna M, Hoepfner P, Weiss AP. Scaphoid excision with four-
corner fusion. Hand Clin 2005; 21(4):531-538.
32. What is the most reasonable surgical treatment for
this 38 year-old laborer with radial wrist pain? 2. Cohen MS, Kozin SH. Degenerative arthritis of the wrist:
proximal row carpectomy versus scaphoid excision and
A. Arthroscopic debridement four-corner arthrodesis. J Hand Surg 2001; 26A(1):94-104.
B. Vascularized bone grafting of the scaphoid with 3. Siegel JM, Ruby LK. A critical look at intercarpal arthrodesis:
internal fixation review of the literature. J Hand Surg 1996; 21A(4):717-723.
C. Scaphoid excision with capitolunate fusion
D. Total wrist fusion
E. Total wrist arthroplasty

Preferred Response: C

Question 32, Figure 1 Question 32, Figure 2

2009 Self-Assessment Examination | 19


33. What will optimize midcarpal motion in a patient Discussion: External fixation with two pins in the thumb
with a radioscapholunate fusion? metacarpal and two pins in the index metacarpal
provides rigid stability to allow the soft tissues of the first
A. Fusion with headless screws
web space to heal while minimizing contracture.
B. Excision of the distal pole of the scaphoid
C. Excision of the entire triquetrum References:
1. Acartürk TO, Ashok K, Lee WP. The use of external skeletal
D. Midcarpal interposition arthroplasty fixation to facilitate the surgical release of wrist flexion and thumb
E. Release of the radioscaphocapitate ligament web space contractures. J Hand Surg 2006; 31A(10):1619-1625.

2. Littler JW. The prevention and the correction of adduction


Preferred Response: B
contracture of the thumb. Clin Orthop Relat Res 1959; 13:182-192.
Discussion: Radioscapholunate fusion without excision
of the distal scaphoid creates a bridge between the
radius and the distal carpal row through the stout 35. When comparing treatment results after
ligaments of the STT joint. Without excision of the incision and drainage of suppurative flexor
distal pole of the scaphoid, the distal row is “locked” to tenosynovitis, the use of postoperative continuous
the proximal row which has been fused to the radius. catheter irrigation:
STT forces are high as flexion and radial deviation are A. Demonstrates improved healing times and
prevented. There is risk of scaphoid fracture due to these functional recovery
high forces as well. Excision of the distal portion of the
B. Is not significantly different from intraoperative
scaphoid (30%) “unlocks” the proximal and distal carpal
irrigation alone
rows, leading to improved mobility and less degeneration
of the STT joint. C. Demonstrates significantly prolonged healing times
D. Is significantly more effective in treating
Weiss and Rodner, in a literature review paper, have MRSA infections
noted when the distal pole of the scaphoid is excised in
a radiocarpal fusion, the expected flexion/extension arc E. Diminishes the need for postoperative antibiotics
of the wrist is 50-66% of the opposite side. These results Preferred Response: B
have also been shown by Garcia-Elias and others.
Discussion: Lille et al. demonstrated, in a retrospective
References: review of 20 patients treated without postoperative
1. Garcia-Elias M, Lluch A, Ferreres A, Papini-Zorli I, Rahimtoola irrigation versus 55 patients with postoperative
ZO. Treatment of radiocarpal degenerative osteoarthritis by continuous irrigation, no significant differences between
radioscapholunate arthrodesis and distal scaphoidectomy.
these two groups with respect to length of hospital stay,
J Hand Surg 2005; 30A(1):8-15.
complication rates, or postoperative range of motion.
2. Nagy L, Büchler U. Long-term results of radioscapholunate While this study is level IV evidence, it does not show a
fusion following fractures of the distal radius. J Hand Surg 1997;
significant benefit of postoperative continuous irrigation.
22B(6):705-10.

3. Weiss KM, Rodner CE. Osteoarthritis of the wrist. J Hand Surg References:
2007; 32A:725-746. 1. Lille S, Hayakawa T, Neumeister MW, et al. Continuous
postoperative catheter irrigation is not necessary for the
treatment of suppurative flexor tenosynovitis. J Hand Surg 2000;
34. What is the most effective means of preventing 25B(3):304-307.
a thumb web space contracture during the initial 2. Carter SJ, Burman SD. Treatment of digital tenosynovitis by
management of a mutilated hand? irrigation with peroxide and oxytetracycline. Review of nine
cases. Annals of Surgery 1996; 163:645-650.
A. Thumb spica splint to maintain the web space
B. CMC fusion
C. External fixation to maintain the web space
D. Web space wound closure
E. Early motion

Preferred Response: C

20 | American Society for Surgery of the Hand


36. Diagnosis of the infection pictured in Figures 1 37. The most common complication after surgical
and 2 in a 52 year-old nurse is confirmed by which treatment of infectious flexor tenosynovitis is:
laboratory test?
A. Digital stiffness
A. Lowenstein-Jensen agar B. Nerve injury
B. Tzanck smear C. Wound dehiscence
C. Rapid Plasma Reagin (RPR) D. Tendon rupture
D. Ziehl Neelsen prep with fluorescent stain E. Recurrent infection
E. Fungal culture
Preferred Response: A
Preferred Response: B
Discussion: Lille et al. demonstrated a 19% digital
Discussion: These clinical pictures demonstrate a stiffness rate in 62 patients followed after flexor
herpetic whitlow infection which is common in health care tenosynovitis incision and drainage. Recurrent infection
professionals. Optimal management in this case is to first occurred in 8% of patients.
confirm the diagnosis by unroofing a vesicle and obtaining
References:
a viral swab for Tzanck smear (viral cultures or PCR assay
are also possibilities). While superinfection with bacteria 1. Lille S, Hayakawa T, Neumeister MW, et al. Continuous
postoperative catheter irrigation is not necessary for the
may occur in cases of herpetic whitlow, these images do
treatment of suppurative flexor tenosynovitis. J Hand Surg 2000;
not show signs of significant bacterial infection. Supportive 25B(3):304-307.
care for herpetic whitlow infections may include anti-virals
2. Pollen AG. Acute infection of tendon sheaths. The Hand 1974;
such as acyclovir, if the lesions are particularly painful.
6:21-25.
References:
1. Adams RA, Botte MJ. Hand infections: treatment
recommendations for specific types. J Am Acad Orthop Surg 38. Prospective randomized trials assessing the value
1996; 4:219-230. of extracorporeal shock wave therapy (ESWT) for
2. Smith P. Lister’s The Hand: Diagnosis and Treatment, 4th ed. lateral epicondylitis have shown significant:
London: Churchill Livingstone, 2002:319. A. Improvement in functional outcomes after ESWT
compared to a steroid injection
B. Improvement in function over six months not
significantly different from placebo
C. Improvement in function over six months with better
functional outcomes than placebo
D. Improvement in pain over six months with better
pain scores than placebo
E. Worsening in pain and function compared to placebo

Preferred Response: B
Question 36, Figure 1 Discussion: Cochrane review and prospective,
randomized, controlled trials on the use of extracorporeal
shock wave therapy for lateral epicondylitis demonstrate
no significant difference over placebo.

References:
1. Staples MP, Forbes A, Ptasznik R, Gordon J, Buchbinder R.
A randomized controlled trial of extracorporeal shock wave
therapy for lateral epicondylitis (tennis elbow). J Rheumatol. 2008
Oct; 35(10):2038-46. Epub 2008 Sep 15.

2. Buchbinder R, Green SE, Youd JM, Assendelft WJ, Barnsley L,


Smidt N. Shock wave therapy for lateral elbow pain. Cochrane
Database Syst Rev. 2005 Oct 19;(4):CD003524. Update of:
Cochrane Database Syst Rev. 2002;(1):CD003524.
Question 36, Figure 2

2009 Self-Assessment Examination | 21


39. An experienced, senior hand surgeon publishes Discussion: Wounds, measuring less than 1 square cm,
her results on a new fixation technique for ulnar affecting the volar aspect of the finger or the fingertip
shortening osteotomies in 10 patients, including itself, have a remarkable ability to heal with dressing
functional outcomes measured by objective and changes. This axiom is especially true in cases of wound
subjective data, as well as pre and postoperative contamination after there has been a formal debridement
DASH scores. This type of study is an example of of all nonviable soft tissue.
what level of evidence?
References:
A. Type I 1. Vedder NB, Hanel DP. The Mangled Upper Extremity. In: Green
B. Type II DP, Hotchkiss RN, Pederson WC, Wolfe SW, eds. Green’s
Operative Hand Surgery. 5th ed. Philadelphia: Churchill
C. Type III Livingston/Elsevier, 2005:1587.
D. Type IV 2. Smith P. Lister’s The Hand: Diagnosis and Treatment, 4th ed.
E. Type V London: Churchill Livingstone, 2002:113-114.

3. Louis DS, Palmer AK, Burney RE. Open treatment of digital tip
Preferred Response: D
injuries. JAMA 1980; 244(7):697-698.
Discussion: Level of evidence has been classified for the
purpose of scientific publication:

Level I evidence – large, randomized, clinical trials with


clear results or meta-analyses of randomized trials

Level II evidence – randomized trials, uncertain results


and high quality prospective cohort

Level III evidence – case-control studies

Level IV evidence – case series with no controls

Level V evidence – expert opinion, physiology or


bench research
Question 40, Figure 1
References:
1. Szabo RM. Show me the evidence. J Hand Surg 2008; 33A:150–156.

2. MacDermid JC. An introduction to evidence-based practice for 41. As the forearm rotates from supination to
hand therapists. J Hand Ther 2004; 17:105–117. pronation, load-bearing on the articular surface of
the sigmoid notch of the distal radius:
A. Remains in the central third
40. Optimal wound management for this through
B. Moves distal to proximal along the dorsal 50%
and through gun shot wound (Figure 1) without
neurovascular, tendon, or bone injury after C. Moves from dorsal to volar in the central third
debridement includes: D. Moves from proximal volar to distal dorsal
A. Split thickness skin graft E. Moves from distal dorsal to proximal volar
B. Full thickness skin graft Preferred Response: D
C. Primary closure
D. Cross finger flap
E. Dressing changes

Preferred Response: E

22 | American Society for Surgery of the Hand


Discussion: Anatomic studies have demonstrated that Discussion: CMT is a type of inherited peripheral nerve
the radius of curvature of the sigmoid notch is greater disease. While onset is usually in the first or second decade,
than that of the ulnar head. This leaves the sigmoid notch it may occur later in life. Its initial clinical presentation
a degree of freedom to translate moving the contact point is variable but frequently patients have problems with
from a volar position in supination to a more dorsal contact tripping and falling. On electrodiagnostic testing, patients
point in pronation. In addition, as the radius rotates around will typically have changes in both motor and sensory
the ulna from supination to pronation, there is a relative conduction velocities, although clinically sensory symptoms
shortening of the radius with respect to the ulna. are unusual. Significant slowing of all motor nerves in
This moves the contact surface from proximally in all extremities will be found with muscle denervation
supination to distally in pronation. The combination of affecting the more distal motors. The pathology in
these two translational forces moves the contact point from CMT relates to defects in the production of myelin with
proximal volar in supination to distal dorsal in pronation. progressive axonal degeneration and muscle denervation.

References: ALS or amyotrophic lateral sclerosis is a neuro-


1. Af Ekenstam F, Hagert CG. Anatomic studies on the geometry degenerative disease affecting the motor neurons. With
and stability of the distal radioulnar joint. Scand J Plast Reconstr loss of the lower motor neurons in the anterior horn of
Surg 1985; 19:17-25. the spinal cord, there is muscle denervation and atrophy.
2. Palmer AK, Werner FW. Biomechanics of the distal radioulnar ALS does not produce a significant slowing in nerve
joint. Clin Orthop Relat Res 1984; 187:26-25. conduction velocity. Cubital tunnel syndrome would
3. Af Ekenstam FW, Palmer AK, Glisson RR. Load on the radius not result in denervation of radial or median innervated
and ulna in different positions of the wrist and forearm: a cadaver muscles. A C8 radiculopathy would not account for the
study. Acta Orthop Scand 1984; 55:363-365. increases in sensory latency, the changes in the flexor
carpi radialis, or the history of frequent tripping. Carpal
tunnel syndrome would not account for the ulnar nerve or
42. A 35 year-old male presents with bilateral arm radial nerve electrodiagnostic abnormalities.
weakness. An EMG/nerve conduction study of
References:
his more symptomatic right upper extremity is
performed and reveals the following results. 1. Chance PF, Bird TD. In: Kasper DL, Braunwald E, et al. Ed
Harrison’s Principals of Internal Medicine. New York: McGraw-
What is the most likely diagnosis?
Hill, 2005:2510-2513.
NCV Normal
2. Wood VE, et al. Treatment of upper limb in charcot-marie-tooth
Right median motor NCV 25 meters/sec >49 meters/sec disease. J Hand Surg 1995; 20B:511-8.
Right median palmar 2.4 ms <2.2 ms
sensory latency
Right ulnar motor NCV 31 m/sec >49 m/sec
Right ulnar palmar 2.4 ms <2.2 ms
sensory latency

EMG Insertional Activity Fibrillations Motor Unit Action


Normal or Increased 0 to 4+ Potentials
(R)EIP Increased 2+ Large, complex
(R)APB Increased 3+ Large, complex,
fast-firing
(R)FDI Increased 3+ Large, complex
(R)FCR Slightly increased 1+ Some normal, some large
(R) Triceps Normal None Normal
(R) Biceps Normal None Normal
(R) Deltoid Normal None Normal

A. ALS (motor neuron disease)


B. Cubital tunnel syndrome
C. C8 radiculopathy
D. Charcot-Marie-Tooth polyneuropathy
E. Carpal tunnel syndrome

Preferred Response: D
2009 Self-Assessment Examination | 23
43. A 60 year-old homeless man is brought to the 44. When discussing with a patient the significance
emergency room after spending the night outside in of a 2 mm articular incongruity three months
sub-zero weather. On examination, all of his fingers after treatment of an intraarticular fracture of
are white, cold, and without capillary refill. For the distal radius by another physician, the patient
treatment, you recommend immediate rewarming is advised:
of his hands in water 104º Fahrenheit, pain
A. An immediate intraarticular osteotomy is necessary
medication, tetanus prophylaxis, antibiotics, and
ibuprofen. The role of ibuprofen is to: B. Within five to seven years, a wrist fusion will
be required
A. Supplement pain management
C. While radiographic changes of joint degeneration
B. Decrease red cell sludging caused by vasoconstriction are likely, they may not be progressive and pain will
C. Limit platelet adhesiveness and thrombosis not correlate with these changes
D. Decrease sympathetically mediated vasospasm D. There will be no further x-ray changes and normal
motion and grip strength can be anticipated
E. Decrease the toxicity of the arachidonic acid cascade
E. Radiographic changes of joint degeneration will be
Preferred Response: E progressive and pain will correlate with the severity
of these changes
Discussion: Frostbite injury has been divided into the
phases of cooling, freezing, and rewarming. Cell death in Preferred Response: C
the cooling phase occurs as a result of vasoconstriction.
With vasoconstriction, there is sludging of blood and Discussion: A correlation between intraarticular
thrombosis. The resultant ischemia triggers chemical and incongruity following distal radius fractures and the
cellular mediated inflammation. The chemical mediators radiographic findings of post-traumatic arthritis were
include prostaglandins, bradykinins, thromboxane, documented by Knirk and Jupiter in 1986. Subsequent
and histamine. During the freezing phase, cell death studies have shown a correlation between the amount
results from cellular dehydration as water freezes in the of incongruity and the development of radiographic
interstitial space creating an ionic gradient intracellular changes of post-traumatic osteoarthritis. There is also a
dehydration. When ice forms within the cell membrane, correlation with a decrease in motion and a decrease in
there is a loss of membrane integrity and cell death. grip strength with intraarticular fractures in comparison
The interstitial ice crystals melt, and the inflammatory to the uninjured extremity. These studies have also
response is reactivated. The result is a toxic local shown that while radiographic changes consistent
environment, which results in additional tissue loss. with post-traumatic arthritis may develop, they may
not be progressive and the severity of changes graded
Attempts have been made to limit tissue thrombosis with according to the system of Knirk and Jupiter do not
heparin and dextran, but studies are inconclusive in their correlate with pain. In this setting with a healed fracture,
ability to preserve tissue. Thrombolytic agents, such as there is presently no evidence to support the need for
streptokinase and urokinase have been shown to be of an intraarticular osteotomy to address this incongruity
some benefit. It remains unclear as to whether or not nor is there evidence that this will progress to require
regional blocks and hyperbaric oxygen can limit tissue a wrist fusion.
loss in the rewarming phase. Clinical studies have shown
improved tissue survival in all degrees of frostbite with References:
the use of ibuprofen to inhibit the inflammatory effects of 1. Knirk JL, Jupiter JB. Intraarticular fractures of the distal end of
the arachidonic acid cascade. the radius in young adults. J Bone Joint Surg 1986; 68A:647-59.

2. Catalano WL III, Kohl JR, Gelberman RH et al. Displaced


References: intraarticular fractures of the distal aspect of the radius: long-
1. Heggers JP, Robson MC, et al. Experimental and clinical term results in young adults after open reduction and internal
observations on frostbite. Ann Emerg 1987; 16(9):1056-62. fixation. J Bone Joint Surg 1997; 79A:1290-302.

2. Su CW, Lohman R, Gottlieb LJ. Frostbite of the upper extremity. 3. Forward DP, Davis TRC, Sithole JS. Do young patients with
Hand Clin 2000; 16(2):235-47. malunited fractures of the distal radius inevitably develop
symptomatic post-traumatic arthritis? J Bone Joint Surg 2008;
3. Bruen KJ, Ballard JR, et al. Reduction of the incidence of
90B(5):629-637.
amputation and frostbite injury with thrombolytic therapy. Arch
Surg 2007; 142(6):546-51.

24 | American Society for Surgery of the Hand


45. Eight weeks after closed reduction and 46. The most important factor inherent to the skin
percutaneous pinning of carpometacarpal graft itself in regard to contracture is the:
dislocations of the index through small fingers,
A. Skin graft donor site
a patient demonstrates active and passive MP
extension of only 45º. The patient has complete B. Total percentage of dermis within the graft
finger flexion and good grip strength. An intrinsic C. Absolute thickness of the graft
tightness test is positive for all digits. The initial D. Technique used in harvest of the graft
treatment for this patient should be:
E. Age of the patient
A. A1 pulley releases
Preferred Response: B
B. Static progressive MP extension splinting
C. Botox injections of the interosseous muscles Discussion: In both experimental and clinical settings,
full-thickness skin grafts have been shown to produce
D. Lateral band excision
less contracture than split-thickness skin grafts. When
E. Volar MP capsulectomies looking at factors inherent to the skin graft, it is the
percentage of the dermis grafted, not the absolute
Preferred Response: B
thickness of the graft that determines the amount of
Discussion: Following a high energy injury involving contracture (i.e., the greater the percentage of the
dislocation of multiple CMC joints, one would expect dermis included in the graft at any given donor site the
the possibility of injury to the interosseous muscles. less the degree of contracture). Other factors shown
The presence of MP flexion contractures and a positive to decrease contracture are early grafting, positioning
intrinsic tightness test would be consistent with myostatic devices, and pressure application to the graft during the
contracture of the interosseous muscles, either due remodeling phase.
to direct trauma or ischemia from a compartment
References:
syndrome or devascularization. While surgical release
of the intrinsics or an intrinsic slide might be required in 1. Corps BUM: The effect of graft thickness, donor site, and graft
bed on graft shrinkage in the hooded rat. BR J Plast Surg 1969;
the future, the initial treatment would be to attempt to
22:125.
aggressively stretch out the myostatic contracture. This
would best be accomplished using a static progressive 2. Nedelec B, Gharary A, et al. Control of wound contraction, basic
and clinical features. In Tredget EE, Ed. Thermal Injuries. Hand
MP extension splint.
Clin 2000; 16(2):289-302.
Trauma of this nature would not be expected to produce
stenosing tenosynovitis of the flexor sheaths. As these
are myostatic contractures and not spastic contractures,
Botox has no role in the treatment. Lateral band excision
would only address the effects of the intrinsic tightness
at the PIP level but would not result in improvement of
active and passive MP extension as the intrinsics would
maintain their attachments to the base of the proximal
phalanx. Unless there was direct trauma to the palmar
aspects of the MP joints, volar capsular scarring would
not be anticipated in this injury and releasing the volar
aspect of the MP joint would have no impact on the
myostatic contracture of this injury.

Reference:
Peimer,CA. Intrinsic Muscle Dysfunction and Contractures.
In: Peimer, CA ed: Surgery of the Hand and Upper Extremity.
New York: McGraw-Hill, 1996:1559-82.

2009 Self-Assessment Examination | 25


47. A 35 year-old male laborer presents with a three- 48. Six months following muscle debridement and flexor
month history of activity-related, ulnar-sided wrist compartment release for a Volkmann’s contracture
pain. He has no prior history of injury to the wrist. of moderate severity, a patient can now actively
An MRI of the wrist is obtained (Figure 1). The extend the wrist and fingers simultaneously to a
patient’s diagnosis is most consistent with: neutral position. While all fingers can be passively
flexed into the palm, active flexion lacks 3 cm from
A. Ulnocarpal impaction syndrome
the fingertips to the palm. The patient complains
B. Kienböck’s disease of poor grip strength. Which of the following will
C. Pseudogout best increase active flexion and grip strength?
D. Multiple enchondromas A. A flexor pronator slide
E. Multi-focal giant cell tumors B. An intense three-month strengthening program
Preferred Response: A C. Superficialis to profundus tendon transfer
D. A free gracilis muscle transfer
Discussion: The MRI is remarkable for edema in both
the head of the ulna and ulnar aspect of the lunate E. An ECRL to profundus tendon transfer
accompanied by cystic changes within the medullary bone. Preferred Response: E
The patient has radiographic findings of an ulnar positive
variance and symptoms induced with manual labor. These Discussion: In a Volkmann’s contracture of moderate
findings are most consistent with an ulnocarpal impaction severity, the extensor compartment is typically spared. An
syndrome. The MRI images do not fit the diagnosis of ECRL to profundus transfer would restore digital flexion and
Kienböck’s disease as the pathology is located ulnarly add strength to the patient’s grip. A flexor pronator slide
within the lunate as opposed to radially or throughout the would add no additional muscle power to flexion and might
body of the lunate. While Kienböck’s disease can occur further impair the patient’s ability to flex the fingers into the
in an ulnar positive variance, it is less frequent, and one palm. A superficialis to profundus tendon transfer would
would not expect corresponding changes in the head of sacrifice the existing strength imparted to grip by the
the ulna. There is an absence of inflammatory synovitis superficialis muscles and further lengthen the profundus
and other cystic changes in the wrist to be consistent with tendons, which would impair active digital flexion. Additional
pseudogout or chondrocalcinosis. Multi-centric giant cell therapy might strengthen grip through the superficialis
tumors would be extremely unusual. Multiple enchondromas muscles but would not likely improve total digital flexion.
would also be unlikely and not associated with surrounding A free gracilis muscle transfer could be considered
edematous changes in the adjacent marrow. but would be less predictable and unnecessary in an
extremity where an appropriate muscle transfer exists.
References:
1. Escobedo EB, Bergman G, Hunter JC. MR imaging of ulnar Reference:
impaction. Skeletal Radiol 1993; 24:85-90. Botte MJ. Compartment Syndromes and Ischemic Contractures.
2. Tomaino MM, Elfar J. Ulnar impaction syndrome. Hand Clin 2005; In: Berger RA, Weiss APC, eds. Hand Surgery. Philadelphia:
21:567-575. Lippincott Williams & Wilkins, 2004:1555-1613.

3. Zanetti M, Saupe N, Nagy L. Role of MR imaging in chronic wrist


pain. Eur Radiol 2007; 17:927-938.
49. A study is planned to evaluate the overall clinical
4. Imaeda T, Nakamura R, Shionoya K, Makino N. Ulnar impaction outcome of patients with rheumatoid arthritis who
syndrome: MR imaging findings. Radiology 1996; 201:495-500.
undergo wrist fusion. Which instrument would be the
most appropriate patient-reported questionnaire?
A. 36-Item Short-Form Health Survey (SF-36)
B. Australian/Canadian Osteoarthritis Hand Index
(AUSCAN)
C. Disabilities of the Arm and Shoulder (DASH)
Questionnaire
D. O’Connor’s Decision Regret Scale (DRS)
E. Michigan Hand Outcomes Questionnaire (MHQ)

Question 47, Figure 1 Preferred Response: E

26 | American Society for Surgery of the Hand


Discussion: Patient–reported questionnaires can be 51. Six hours after undergoing a total elbow
broad, general health questionnaires, region-specific, or arthroplasty, a 62 year-old female rheumatoid
disease-specific. The SF-36 is too general to be applied patient develops a desquamating rash, high
specifically to track the outcomes of hand surgery. The fever, and hypotension. Blood tests show an
AUSCAN is a disease-specific questionnaire for arthritis. elevated BUN and creatinine, as well as elevated
The O’Conner’s Decision Regret Scale measures patient liver function tests. She has to be transferred
satisfaction with the decisions made, but does not to the intensive care unit. The surgical wound
necessarily measure functional outcome. Both DASH is not tender and has no drainage or erythema.
and MHQ are region-specific questionnaires that have Perioperative prophylactic antibiotics had
been validated, have good internal consistency, and are been given. The test most likely to prove your
responsive to treatment changes. However, the MHQ is suspected diagnosis is:
the benchmark for rheumatoid arthritis because it collects
A. Positive blood culture for Staphylococcus aureus
not only functional data, but also information regarding
aesthetics, symptoms, and satisfaction. B. Positive stool test for Clostridium difficile toxin
C. Radiograph of elbow shows gas shadows in the
References: soft tissues
1. Alderman AK, Chung KC. Measuring outcomes in hand surgery.
D. CAT scan of the chest shows pulmonary embolus
Clin Plast Surg 2008; 35:239-250.
E. Positive blood test for Staphylococcus aureus toxin 1
2. Chung KC, Kowalski CP, Myra Kim H, et al. Patient outcomes
following Swanson silastic metacarpophalangeal joint Preferred Response: E
arthroplasty in the rheumatoid hand: a systematic overview.
J Rheumatol 2000; 27:1395-1402. Discussion: This patient has toxic shock syndrome which
is a toxemia rather than a septicemia. The wound is often
benign appearing. Necrotizing fasciitis might have gas
50. In the time course of wound healing, which cells in the soft tissues, but air in the soft tissue would not be
predominate during the inflammatory phase: surprising after total joint replacement. The extremity is
A. Macrophages often very tender with necrotizing fasciitis and would be
expected to have some local signs to include erythema
B. Lymphocytes
and edema. Most surgical cases of toxic shock syndrome
C. Fibroblasts are associated with S. aureus toxin 1 (TSST-1) and its
D. Endothelial Cells presence is confirmed in patient serum by a reverse
E. Keratinocytes passive latex agglutination test.

Preferred Response: A The patient must be transferred to the ICU for monitoring,
the wound is debrided, and implants or drains must be
Discussion: In the time course of different cells appearing removed. The antibiotic of choice is clindamycin which is
in the wound during the healing process, macrophages bacteriostatic and inhibits TSST-1 production.
and neutrophils predominate during inflammation,
lymphocytes peak later and fibroblasts predominate References:
during the proliferative phase. Keratinocytes and 1. Netscher DT, Lee-Valkov P. Infections of the hand. In: Mathes SJ,
endothelial cells appear late in wound healing with Hentz VR eds. Plastic Surgery 2nd ed. Philadelphia: Saunders/
Elsevier, 2006:759-790.
epithelialization and angiogenesis respectively.
2. Grayson MJ, Saldina MJ. Toxic shock syndrome complicating
References: surgery of the hand. J Hand Surg 1987; 12:1082-1084.
1. Witte M, Barbul A. General principles of wound healing. Surg Clin 3. Esperson F, Baek L, Kjaelgard P, et al. Detection of staphyloccal
North Am 1997; 77:509-528. toxic shock syndrome toxin-1 by a latex agglutination kit. Scand
2. Broughton G, Janis JE, Attinger CE. The basic science of wound J Infect Dis 1988; 20:449-450.
healing. Plast Reconstr Surg 2006; 117(7Suppl):12S-34S.

2009 Self-Assessment Examination | 27


52. A 60 year-old healthy construction worker presents 53. A 40 year-old mechanic had sudden onset of
with a lesion (Figure 1) on the dorsum of his hand. pain in his dominant right hand middle finger.
It has enlarged over several months, is not fixed The middle finger is cyanotic and the nail bed has
deeply, and now measures 2.5 cm in diameter. splinter hemorrhages. Allen testing shows delayed
Incisional biopsy revealed it to be a squamous cell flow through the ulnar artery with perfusion of
carcinoma of Broders grade II differentiation. The the finger tips after 10 seconds. The specific
preferred treatment option for this lesion is: diagnostic test that may give pathognomonic
evidence for this condition is:
A. Mohs micrographic surgical excision
B. Cryotherapy A. Nail fold vital capillaroscopy

C. Surgical excision with 6 mm margins B. Arteriography

D. Radiation therapy C. Digital pulse-volume recordings

E. Curettage and electrodesiccation D. Segmental digital pressures


E. Color duplex imaging
Preferred Response: C
Preferred Response: B
Discussion: Cryotherapy and curettage are
recommended only for very small and low-risk Discussion: This patient has hypothenar hammer
squamous cancers. Mohs micrographic surgery would be syndrome with stenosis in the ulnar artery and distal
unnecessary for this tumor. While this tumor is probably embolization to the finger. Nail fold capillaroscopy may be
too large to be appropriately treated by cryotherapy helpful in diagnosis of small vessel disease, but not in this
or curettage and electrodesiccation, it is nonetheless condition. Segmental digital pressures and PVR tracings
relatively low-risk by its location, histologic differentiation may be helpful in evaluation of arterial flow and in making
and size (T1 lesion). Squamous carcinomas greater than some therapeutic decisions. Color duplex imaging may
2 cm are twice as likely to recur locally and three times diagnose stenotic lesions and vessel wall aneurysms.
as likely to metastasize than are tumors less than 2 cm However, the “corkscrew” sign on arteriography is
in diameter (9.2% versus 1.4%). Recommended margins pathognomonic for hypothenar hammer syndrome and
of excision are 4 mm for low-risk tumors and 6 mm (plus is due to alternating stenosis and dilatation of the ulnar
underlying fat) for high-risk tumors. artery. Distal embolic occlusion will also be diagnosed by
arteriography.
While radiation therapy is very effective for treatment
of smaller squamous carcinomas, it requires multiple References:
treatments, and may produce progressive changes 1. Hammond DC, Matloub HS, Yousif NJ, Sanger JR. The
including atrophy, fibrosis, telangiectasis, and even frank corkscrew sign in hypothenar hammer syndrome. J Hand Surg
ulcerations in the treated areas. 1993; 18B:767-769.
2. McClinton MA, Wilgis EFS. Ischemic conditions of the hand. In:
References: Mathes SJ, Hentz VR eds. Plastic Surgery 2nd ed. Philadelphia:
1. Rudolph R, Zelac DE. Squamous cell carcinoma of the skin. Plast Saunders/Elsevier, 2006:791-822.
Reconstr Surg 2004; 114:82e-94e.

2. Brodland DG, Zitelli JA. Surgical margins for excision of primary


cutaneous squamous cell carcinoma. J Am Acad Dermatol 1992;
27:241-248.

Question 52, Figure 1

28 | American Society for Surgery of the Hand


54. A young quadriplegic patient (whose hand is References:
shown in Figures 1 and 2) has already undergone 1. Hentz VR, McAdams TR. Restoration of upper extremity
surgical restoration of elbow extension and now function in tetraplegia. In: Hentz VR ed. Plastic Surgery 2nd ed.
desires improvement in hand function. He is able Philadelphia: Saunders/Elsevier, 2006; Vol. 8: 507-541.
to extend the wrist (but with radial deviation) 2. Hentz VR. Surgical strategy: matching the patient with the
and can pronate the forearm, each function with procedure. Hand Clin 2002; 18:503-518.
strength of grade 4. He has grade 3 strength of
finger and thumb extension, and only a flicker of
finger flexion. The best potential muscles available
for transfer are:
A. Brachioradialis, extensor carpi radialis longus,
pronator teres
B. Brachioradialis, extensor carpi radialis brevis
C. Pronator teres, extensor indicis proprius
D. Brachioradialis, extensor carpi radialis brevis,
pronator teres
E. Extensor carpi radialis longus, pronator teres,
extensor indicis proprius

Preferred Response: A
Question 54, Figure 1
Discussion: In order to answer this question correctly,
one has to determine the cervical level of injury. From
upper cervical to lower cervical, in succession, the
following muscles would become innervated – deltoid,
biceps, brachioradialis, ECRL, ECRB, pronator teres,
FCR, finger extensors, thumb extensors.

If this patient is able to actively pronate the forearm,


then by inference brachoradialis and both ECRL and
ECRB will have good function (being at higher spinal
levels). The patient radially deviates the wrist in extension
because ECU is not functioning. Since this patient
would be expected to have excellent biceps function,
brachioradialis can be used for tendon transfer. Any
muscle with at least grade 4 strength can be used in Question 54, Figure 2
transfer which will be downgraded by one (resulting in
useful function of at least grade 3 against gravity). Any
muscles of grade 3 strength or less will be unsuitable
for transfer.

ECRL and ECRB are available for transfer and one would
favor leaving the central wrist extensor and so ECRL
would be preferred. A strong wrist extensor (ECRB) must
then be left intact.

While there will be no active forearm pronation remaining


once pronator teres is transferred, it nonetheless can be
used as the patient will incorporate gravity to substitute
for forearm pronation.

2009 Self-Assessment Examination | 29


55. Early protected dynamic splinting following hand 56. A 60 year-old female with a 10-year history of
replantation at the transmetacarpal level is best rheumatoid arthritis has a hyperextension deformity
accomplished by a functional brace that maintains of her dominant thumb IP joint (Figure 1). She
the following wrist and MP joint positions: cannot actively flex that joint. The joint is passively
mobile and her x-ray is shown (Figure 2). The most
A. Wrist in neutral, MP joints in extension
suitable surgical treatment is:
B. Wrist in 30° extension, MP joints in extension
A. Thumb IP joint fusion
C. Wrist in 30° extension, MP joints in flexion
B. Middle finger FDS transfer to the thumb
D. Wrist in 30° flexion, MP joints in flexion
C. Distal stump FPL tenodesis of the distal phalanx
E. Wrist in neutral, MP joints in flexion
D. Primary FPL repair
Preferred Response: E E. Palmaris longus transfer to the thumb
Discussion: Splinting after replantation of transmetacarpal Preferred Response: B
hand amputations must prevent intrinsic minus hand
deformity and yet maintain extrinsic extensor and flexor Discussion: In the presence of a normal IP joint,
tendon function. The splint must mimic intrinsic functions. arthrodesis would not be the best treatment. Tenodesis at
The distal intrinsic tendons become adherent in their the IP joint will not restore active flexion.
canals in a shortened position when the MP joints are
Tendon grafts can sometimes be used for the so-called
maintained flexed. The dynamic craning outrigger splint
Mannerfelt lesion secondary to FPL attrition rupture at the
attempts to maintain the visco-elastic properties of the
level of the carpal scaphoid bone. However, if the distal
extensor mechanism to enable extension of the DIP and
FPL cannot reach into the wrist incision, then a tendon
PIP joints through the extensor tendons, when no intrinsic
transfer is preferred. The Palmaris longus will have
function is present.
difficulty reaching its distal target.
Wrist in neutral (or at most 15° extension) enables both
The superficial flexor of the middle finger is the preferred
extrinsic flexor and extensor repairs to be protected.
motor because it is longer and avoids compromise of grip
References: function of the ring and small fingers. It is detached in
1. Scheker LR, Netscher, DT, et al. Functional results of dynamic the distal palm and is anchored to the volar aspect of the
splinting after transmetacarpal, wrist and distal forearm thumb distal phalanx deep to the terminal stump of FPL.
replantation. J Hand Surg 1995; 20B:584-590.
References:
2. Scheker LR, Hodges A. Brace and rehabilitation after replantation
and revascularization. Hand Clin 2001; 17:473-480. 1. Feldon P, Terrono AL, Nalebuff EA, Millender LH. Rheumatoid
arthritis and other connective tissue diseases. In: Green P, Hotchkiss
RN, Pederson WC, eds. Green’s Operative Hand Surgery 4th ed.
Philadelphia: Churchill Livingston, 1999:1680-1682.

2. Mannerfelt LG, Norman O. Attrition ruptures of flexor tendons in


rheumatoid arthritis caused by bony spurs in the carpal tunnel. A
clinical and radiological study. J Bone Joint Surg 1969; 51B:270-277.

30 | American Society for Surgery of the Hand


Discussion: Evaluation of segmental arterial pressures
is the single most valuable test for the patient who has
upper extremity ischemia. A digital/brachial index of 0.7
or less indicates compromised circulation. Resection
of an occluded ulnar arterial segment produces a
sympathectomy effect as described by Leriche.
Reconstruction is necessary if the digital/brachial index is
less than 0.7. If the patient were to have been a diabetic,
then arterial pressure indices may be inaccurate.

While cold stress testing may be indicative of an


Question 56, Figure 1 exaggerated sympathetic response, the most significant
preoperative evaluation that indicates collateral flow is
adequate and that arterial segment resection without
reconstruction can be done is the adequacy of collateral
flow as determined by digital/brachial index.

The arteriographic “corkscrew” sign is pathognomonic of


hypothenar hammer syndrome. Arteriography is a “static”
evaluation and gives little dynamic information regarding
blood flow. Vital capillaroscopy may indicate small vessel
disease. Laser Doppler perfusion indicates diminished
digital blood flow, but does not really help in treatment
decisions in this case.

References:
1. Chloros GD, Smerlis NN, Zhongyu L, Smith TL, Smith BP,
Koman LA. Non-invasive evaluation of upper extremity vascular
Question 56, Figure 2 perfusion. J Hand Surg 2008; 33A:591-600.

2. McClinton MA, Wilgis EFS. Ischemic conditions of the hand. In:


Mathes SJ, Hentz VR eds. Plastic Surgery 2nd ed. Philadelphia:
57. A 46 year-old non-diabetic male laborer presents Saunders/Elsevier, 2006: Ch. 194: 791-822.
with a 3-week history of constant pain in the ulnar 3. Zimmerman NB, Zimmerman ST, McClinton MA, et al. Long-term
three digits of his dominant right hand. All fingers recovery following surgical treatment for ulnar artery occlusion.
are cyanotic and the little finger has an ulcer at the J Hand Surg 1994; 19A:17-21.
tip (Figure 1). The Allen test shows no arterial flow
through the ulnar artery. Which preoperative study
will guide surgical treatment:
A. Digital plethysmography and digital/brachial index
B. Fingertip temperature re-warming following cold
stress testing
C. Forearm and hand arteriography
D. Vital nailfold capillaroscopy
E. Laser Doppler fingertip fluxmetry

Preferred Response: A

Question 57, Figure 1

2009 Self-Assessment Examination | 31


58. Biomechanical analysis has shown that the digital While no reconstruction restores normal tendon excursion/
pulley reconstruction technique which produces joint motion relationship, the three loop reconstruction at
the maximum strength is: A2 and A4 pulleys restored a more normal relationship
than the belt-loop reconstruction. Data on strength and
A.
stiffness by the Mayo clinic study, suggest that the triple
loop pulley reconstruction may be sufficient even to allow
early protected active finger motion.

References:
1. Widstrom CJ, Johnson G, Doyle JR, Manske PR, Inhofe P. A
mechanical study of six digital pulley reconstruction techniques:
part I. Mechanical effectiveness. J Hand Surg 1989; 14A:821-825.

B. 2. Widstrom CJ, Doyle JR, Johnson G, Manske PR, McGee R. A


mechanical study of six digital pulley reconstruction techniques:
part II. Strength of individual reconstructions. J Hand Surg 1989;
14A:826-829.

3. Lin T, Amadio PC, An KN, Cooney WP, Chao EYS. Biomechanical


analysis of finger flexor pulley reconstruction. J Hand Surg 1989;
14B:278-282.

C.
59. Which group of patients with a congenital first
web space contracture is associated with the
highest rate of complication when treated with a
dorsal transposition flap from the index finger?
A. Radial polydactyly
D. B. Symbrachydactyly
C. Radial club hand
D. Arthrogryposis
E. Cleft hand
Preferred Response: B
E.
Discussion: Friedman and Wood had published an
extensive series on the use of dorsal transposition
flaps for various etiologies of thumb-index web
space contractures. There was a 100% complication
rate when this procedure was used in patients with
symbrachydactyly. There is insufficient flap length and
the contracture too severe to be adequately treated
with a dorsal transposition flap. Other investigators
Preferred Response: B
have also commented on the severe web contracture in
Discussion: In an initial two-part study, the Karev method symbrachydactyly which often needs a staged approach.
was strong and provided the best effectiveness. The
References:
single loop tendon graft at A2 and A4 originally described
1. Coombs CJ, Mutimer KL. Tissue expansion for the treatment
by Bunnell was modified by Strickland as an interweave
of complete syndactyly of the first web. J Hand Surg 1994;
of the two ends. This gave a strength equal to that of the 19A:968-972.
Karev repair, but not as effective. The weakest repairs
2. Friedman R, Wood VE. The dorsal transposition flap for
were the Lister extensor retinaculum and Weilby tendon
congenital contractures of the first web space: a 20-year
weave into the rim of residual pulley. experience. J Hand Surg 1997; 22A:664-670.

32 | American Society for Surgery of the Hand


60. Which molecule has shown promise in inhibiting not occur in gout. Calcification of the articular disc of the
lipid peroxidation in limbs subjected to ischemia TFCC occurs in pseudo gout. Cartilage space narrowing
reperfusion, such as when a tourniquet is used on occurs late, not early, in gout.
the upper arm in hand surgery?
References:
A. Dexmedetomidine 1. Straub LR, Smith JW, Carpenter GK Jr., Dietz GH. The surgery of
B. Uric acid gout in the upper extremity. J Bone Joint Surg 1961; 43A:731-752.

C. Lactate dehydrogenase 2. Moore JR, Weiland AJ. Gouty tenosynovitis of the hand. J Hand
Surg 1985; 10A:291-295.
D. Cyclo-oxygenase
E. Aspirin

Preferred Response: A

Discussion: Prolonging ischemia time may allow


longer tourniquet times during complex procedures or
allow replantation of severed digits or limbs previously
Question 61, Figure 1
thought to be non-replantable. Efforts have been made
at the molecular level to accomplish this. Yagmurdur
and associates have shown dexmedetomidine to
inhibit lipid peroxidation which leads to injury due to 62. A 72 year-old man with a longstanding history of
ischemia. Dexmedetomidine significantly attenuated gout on 75 mg Indocin/day presents with a non-
plasma hypoxanthine production in the ischemia and painful mobile mass over the volar aspect of the
plasma malondialdehyde production in the reperfusion small finger proximal phalanx. He reports limited
periods. Blood creatine phosphokinase and uric acid flexion of the DIP joint for the past 2 weeks. X-rays
levels were significantly lower in the dexmedetomidine are normal, other than age-related degenerative
group compared with those in the control group after change. The most effective treatment would be:
reperfusion. The other molecules listed do not assist in A. Kenalog injection directly at the level of the finger mass
lowering lipid peroxidation.
B. Increasing the dose of Indocin to 150 mg daily for
References: the next 10 days
1. Yagmurdur H, Ozcan N, Dokumaci F, Kamer K, Yilmaz F, C. Stopping the Indocin and changing to Allopurinol
Basar H. Dexmedetomidine reduces the ischemia-reperfusion D. Debridement of the flexor tendons
injury markers during upper extremity surgery with tourniquet.
J Hand Surg 2008; 33A:941-947. E. Repair of the FDP tendon using a bridge graft
2. Concannon MJ, Kester CG, Welsh CF, Puckett CL. Patterns of Preferred Response: D
free-radical production after tourniquet ischemia: implications for
the hand surgeon. Plast Reconstr Surg 1992; 89:846–852. Discussion: The flexor tendon sheath was originally
thought to protect the flexor tendons from uric acid
crystal deposition. This is not true. In gout, flexor tendons
61. A consistent early radiographic finding of gout can become involved with uric acid being directly
arthropathy in the hand/wrist is: deposited into the substance of the flexor tendons. Early
A. Symmetric cartilage space narrowing on they present with painless mobile masses in the digits
which can limit motion. If this goes unchecked, rupture
B. Peri-articular erosions
can occur which requires debridement and repair using a
C. Osteosclerosis bridge graft or tendon transfer.
D. Osteolysis of the tufts of the distal phalanges
References:
E. Calcification of the articular disc of the TFCC 1. Weniger FG, Davison SP, Risin M, Salyapongse AN, Manders EK.
Gouty flexor tenosynovitis of the digits: report of three cases.
Preferred Response: B
J Hand Surg 2003; 28(4):669-672.
Discussion: One of the earliest findings of gout is per- 2. Wurapa RK, Zelouf DS. Flexor tendon rupture caused by gout:
articular erosions. Osteopenia and osteosclerosis are not a case report. J Hand Surg 2002; 27(4):591-593.
typical in gout. Osteolysis of the distal phalanges does

2009 Self-Assessment Examination | 33


63. The single presenting clinical finding in newborn References:
compartment syndrome of the forearm not present 1. Foucher G, Braun JB. A new island flap transfer from the dorsum
in adult forearm compartment syndrome is: of the index to the thumb. Plast Reconstr Surg 1979; 63:344-379.

A. No pain on passive finger extension 2. Earley MJ. The arterial supply of the thumb, first web and index
finger and its surgical application. J Hand Surg 1986; 11B:163-174.
B. Soft compartments on palpation
C. Sentinel skin lesion
D. Cyanosis of the involved extremity
E. There are no differences

Preferred Response: C

Discussion: Ragland had presented the over 20 years


experience of newborn forearm compartment syndrome
from the Texas Scottish Rite Children’s Hospital in
Dallas, Texas. They found 24 cases of newborn forearm
compartment syndrome—much more common than
previously expected. All of the cases presented with a
sentinal skin lesion on the involved forearm. It ranged
from a lesion resembling a bruise, to circumferential
lesions. The smaller the lesion is, the less tissue damage.
Question 64, Figure 1
References:
1. Caouette-Laberge L, Bartoluzzi P, Egerszegi EP, Marton D.
Neonatal Volkmann’s ischemic contracture of the forearm: a 65. In addition to nail deformity, the other primary
report of five cases. Plast Reconstr Surg 1992; 90:621-628.
problem in reconstructed thumbs after radial
2. Ragland R III, Moukoko D, Ezaki M, Carter PR, Mills J. Forearm polydactyly is:
compartment syndrome in the newborn: report of 24 cases.
J Hand Surg 2005; 30A:997-1003. A. Inadequate length
B. Rotational deformity
C. Joint angulation
64. The artery to the flap shown in Figure 1 is a branch
of the: D. Poor sensation
E. Poor motion
A. Radial artery
B. Dorsal Carpal Arch Preferred Response: C
C. Princeps Pollicis artery Discussion: Goldfarb has shown most thumbs after radial
D. Deep palmar arch polydactyly reconstruction have satisfactory length and
E. Digital artery girth. Dobyns noted this as well. The main problems
appear to be interphalangeal joint angulation which was
Preferred Response: A similar to reduced nail width in Goldfarb’s study.

Discussion: The flap shown in Figure 1 is a first dorsal References:


metacarpal artery flap commonly used for thumb 1. Goldfarb CA, Patterson JM, Maender A, Manske PR. Thumb size
resurfacing. Earley first described the arterial anatomy of and appearance following reconstruction of radial polydactyly.
the first web area and Foucher initially described the first J Hand Surg 2008; 33A:1348-1353.
metacarpal artery flap which is sometimes called a “kite” 2. Dobyns JH, Lipscomb PR, Cooney WP. Management of thumb
flap. The first dorsal metacarpal artery is a branch off the duplcation. Clin Orthop Relat Res 1986; 195:26-44.
radial artery.

34 | American Society for Surgery of the Hand


66. What is the diagnosis of the infant seen in Figure 1? 67. Subungual squamous cell carcinoma is most
commonly associated with:
A. Congenital radioulnar synostosis
B. Ulnar dimelia A. Onychomycosis

C. Arthrogryposis multiplex congenita B. Chronic psoriatic nail disease

D. Aperts syndrome C. Subungual exostosis

E. Congenital contractural arachnodactyly of the elbow D. Human papilloma virus


E. Acute subungual paronychial infection
Preferred Response: C
Preferred Response: D
Discussion: Elbow extension contracture is a common
presentation in arthrogryposis multiplex congenita. The Discussion: Theunis et al studied three patients with
presence of wrist and digital flexion contractures as subungual squamous cell carcinoma and found a
seen in this child is a common finding in this condition. relationship between these patients’ diseases and
Congenital contractural arachnodactyly of the elbow or human papillomavirus particularly HPV-16. Porembski
Beals syndrome is associated with flexion rather than et al reported a patient with subungual squamous cell
extension contracture of the elbow. Congenital radioulnar carcinoma arising in more than one digit and who also
synostosis is not usually associated with elbow extension had positive immunostaining for human papillomavirus.
contracture. Ulnar dimelia is hand duplication and Apert’s The patient worked for a wastewater utility company for
syndrome is associated with syndactyly. 22 years and placed his hands in stagnant wastewater
daily. Saccone et al reported on subungual malignant
References: degeneration following chronic perionychial infection.
1. Viljoen D. Congenital contractural arachnodactyly (Beals Acute paronychial infection is not associated with
syndrome). J Med Genet 1994 Aug; 31(8):640-3.
malignant degeneration. All other conditions listed have
2. Mennen U. Surgical aspects of neuromuscular disorders of no relationship to subungual carcinoma.
the upper extremity. In: Gupta A, Kay S, Schecker L, eds. The
growing hand. Philadelphia: Mosby/Elsevier, 2000:443-446. References:
1. Theunis A, Andre J, Noel JC. Evaluation of the role of genital
human papillomavirus in the pathogenesis of subungual
squamous cell carcinoma. Dermatology, 1999; 198:206–208.

2. Porembski M. and. Rayan G. M subungual carcinoma in multiple


digits. J Hand Surg 2007; 32B(5):547–549.

3. Saccone PG, Rayan GM. Subungual malignant degeneration


following chronic perionychial infection. Orthopaedic Review
1993; 22:623-26.

Question 66, Figure 1

2009 Self-Assessment Examination | 35


68. The effect of reversed interposition vein graft 69. A 17 year-old high school football player presents
for ulnar artery thrombosis (UAT) on cold stress two days after a hyperdorsiflexion injury to the
expected outcomes is: wrist (Figure 1). The most appropriate treatment
for this patient is:
A. Response worse than preoperative
B. Response less than normal controls A. Short arm thumb spica cast application

C. No improvement B. Short arm thumb spica cast application with


bone stimulation
D. Response similar to normal controls
C. Open reduction and screw fixation
E. Response better than normal controls
D. Long arm cast application
Preferred Response: D E. Application of a muenster cast
Discussion: Chloros et al reported the outcomes of 12 Preferred Response: C
patients (13 hands) at 2 years’ minimum follow-up after
using reversed interpositional vein grafts for the treatment Discussion: Non-displaced proximal pole fractures of
of symptomatic patients with UAT. Patients were evaluated the scaphoid are considered unstable because of the
before surgery and at final follow-up using the following tenuous blood supply. These fractures have a propensity
instruments: (1) cold sensitivity severity scale, (2) visual for nonunion and require prolonged immobilization when
analog pain scale, (3) symptom and function scale, and (4) treated non-operatively. Open reduction and screw
Wake Forest University symptom scale (pain, numbness, stabilization of the proximal pole fragment is the best
and cold intolerance). Digital microvascular perfusion treatment.
testing (laser Doppler perfusion imaging and isolated
References:
cold stress testing) was also performed, and compared
with normal controls. All patients were evaluated for graft 1. Rettig ME, Raskin KB. Retrograde compression screw fixation
of acute proximal pole scaphoid fractures. J Hand Surg 1999;
patency as determined by Allen’s testing and/or Doppler
24A(6):1206-10.
ultrasound. Ten of the 13 grafts were patent at final follow-
up (77% patency rate). In all the patients with patent 2. Gelberman RH, Panagis JS, Taliesnik J, Baumgartner M. The
arterial anatomy of the human carpus. Part I: the extraosseous
grafts, the symptom scale, visual analog pain scale, the
vascularity. J Hand Surg 1983; 8:367-375.
cold sensitivity severity scale, and the isolated cold stress
testing responses were significantly improved at final
follow-up. Isolated cold stress testing responses were
not different from those of normal controls. The changes
in the function scale, Wake Forest University scale, and
laser Doppler perfusion imaging were not significant. The
authors concluded that successful arterial reconstruction
in symptomatic posttraumatic UAT decreases symptoms,
improves function and microvascular physiology, and
has a positive effect on the health-related quality of life.
Dethmers and Houpt treated 27 cases of hypothenar
hammer syndrome with revascularizations. At follow-up,
color-coded Duplex sonography revealed 13 grafts to be
patent, five occluded, one partially thrombosed, seven
grafts with aneurysmal dilatations and one coiled graft.
The Duplex outcomes of the revascularizations, however,
did not correspond well with the clinical outcomes. Question 69, Figure 1

References:
1. Chloros GD, Lucas RM, Li Z, Holden MB, Koman LA.
Post-traumatic ulnar artery thrombosis: outcome of arterial
reconstruction using reverse interpositional vein grafting at 2
years minimum follow-up. J Hand Surg 2008; 33A(6):932-40.

2. Dethmers RS, Houpt P. Surgical management of hypothenar and


thenar hammer syndromes: a retrospective study of 31 instances
in 28 patients. J Hand Surg 2005; 30B(4):419-23.

36 | American Society for Surgery of the Hand


70. A 52 year-old golfer presents with a two-month 71. The anterolateral thigh flap is based upon which of
history of pain along the ulnar aspect of the palm, the following pedicles:
which is exacerbated by grip and grasp activities.
A. Medial femoral circumflex artery
A CT scan of the injured hand is seen in Figure 1.
The most appropriate treatment for this patient is: B. Ascending branch of the lateral femoral
circumflex artery
A. Long arm cast application
C. Descending branch of the posterior femoral
B. Open reduction and internal fixation of the fracture circumflex artery
C. Ulnar gutter short arm cast immobilization D. Inferior epigastric artery
D. Fracture fragment excision E. Descending branch of the lateral femoral
E. Short arm splint with activity modification circumflex artery

Preferred Response: D Preferred Response: E

Discussion: Fractures of the hook of the hamate may Discussion: The pedicle for the anterolateral thigh flap is
occur secondary to repetitive trauma in stick-handling the descending branch of the lateral femoral circumflex
sports. Acute (<3 weeks) non-displaced fractures may artery. The flap is well suited for large defects of the
be treated with cast immobilization. In cases of nonunion upper extremity. Conversion to a sensate flap is possible
or delayed diagnosis excision or ORIF are possible with the lateral femoral cutaneous nerve.
treatment options. ORIF has a nonunion rate approaching
Reference:
30%, including complications such as flexor tendon
Saint-Cyr M, Gupta A. Microsurgical reconstruction of the upper
rupture.
extremity: state of the art. Hand Clin 2007; 23(1):41-42.
References:
1. Stark HH, Jobe FW, Boyes JH, Ashworth CR. Fracture of the
hook of the hamate in athletes. J Bone Joint Surg 1977;
59A:575-582.

2. Scheufler O, Andresen R, Radmer S, Erdmann D, Exner K,


Germann G. Hook of hamate fractures: critical evaluation of
different therapeutic procedures. Plast Reconstr Surg 2005;
115:488-497.

Question 70, Figure 1

2009 Self-Assessment Examination | 37


72. A 62 year-old female with Dupuytren’s contracture 73. In Dupuytren’s disease, the diseased fascia
underwent a palmar fasciectomy of the ring contains collagen of primarily type:
and small finger. One week after surgery, she
A. I
was doing well with near full composite flexion,
minimal discomfort, and minimal swelling. Two B. II
weeks after surgery, she presents with a two-day C. III
history of increasing pain, stiffness, and swelling D. IV
(Figure 1). What is the most appropriate step
E. VI
in treatment?
A. Narcotic administration and immobilization Preferred Response: C

B. Stellate ganglion block Discussion: Normal palmar tissue consists of primarily


C. Oral antibiotics with serial examinations type I collagen. Type IV and VI collagen may be seen in
small amounts during the early stages of Dupuytren’s
D. Medrol dose pack with light range of motion exercises
diseases. However, Type III is the most common collagen
E. Hospital admission with IV antibiotics within the diseased fascia.
Preferred Response: D Reference:
Discussion: The patient demonstrates signs and Glimcher MJ, Peabody HM. Collagen organization. In: McFarlane
RM, McGrouther DA, Flint MH eds. Dupuytren’s Disease: Biology
symptoms consistent with a “flare reaction” following
and Treatment. New York: Churchill Livingstone, 1990:72-85.
subtotal palmar fasciectomy. This recognized
complication is often noted 2-3 weeks postoperative
with a sudden decline in function and increase in pain.
74. A patient returns three months after undergoing
The clinical picture reveals no obvious signs of infection.
free vascularized fibular grafting for a humeral
These reactions, when diagnosed early, usually respond
segmental defect with complaints of leg pain at
to anti-inflammatory medication and occupational
the donor site. The distal fibula was resected 4 cm
therapy focusing on motion exercises.
above the ankle joint. The most likely etiology of
References: the patient’s leg pain is:
1. Prosser R, Conolly WB. Complications following surgical treatment A. Sural nerve entrapment
for Dupuytren’s contracture. J Hand Ther 1996; 9(4):344-8.
B. Valgus instability of the ankle
2. Bulstrode NW, Jenec B, Smith PJ. The complications of Dupuytren’s
contracture surgery. J Hand Surg 2005; 30A(5):1021-5.
C. Tibial stress fracture
D. Saphenous nerve entrapment
E. Peroneal tendonitis

Preferred Response: B

Discussion: Tibial stress fracture and peroneal tendon


weakness are recognized complications of donor site
morbidity after vascularized fibular harvest. To minimize
the risk of valgus instability of the ankle, the fibula should
be harvested at a minimum of 7-8 cm proximal to the
ankle joint. In this case, the distal osteotomy site is in
close proximity to the mortise resulting in instability.

References:
1. Vail TP, Urbaniak JR. Donor-site morbidity with use of
vascularized autogenous fibular grafts. J Bone Joint Surg 1996;
78A:204-211.

2. Bishop AT. Vascularized Bone Grafting. In: Green DP, Hotchkiss


RN, Pederson WC, eds. Green’s Operative Hand Surgery. 4th ed.
Philadelphia: Churchill Livingston, 1999:1224-1226.
Question 72, Figure 1

38 | American Society for Surgery of the Hand


75. A college student presents 4 weeks after 76. A 54 year-old obese female presents 5 weeks
sustaining an injury to the ring finger of the after undergoing radial head replacement for a
dominant hand while playing flag football. comminuted radial head fracture/dislocation of
Ultrasound assessment of the digit reveals that the elbow with a new radiograph (Figure 1). What
the distal stump of the flexor tendon is located treatment is recommended?
at the level of the PIP joint. Inspection of the
A. Open reduction with cast immobilization
DIP joint demonstrates mild hyperextension.
The recommended treatment is: B. Open reduction with lateral collateral ligament repair
or reconstruction
A. DIP joint arthrodesis
C. Closed reduction
B. Direct repair of the flexor digitorum profundus
D. Open reduction with radial head implant exchange,
C. Single stage tendon graft reconstruction lateral collateral ligament repair/reconstruction with
D. Two stage flexor tendon reconstruction or without hinged elbow external fixator application
E. Non-operative management E. Open reduction with removal of the radial head implant
and lateral collateral ligament repair or reconstruction
Preferred Response: B
Preferred Response: D
Discussion: With Type II FDP avulsions (retraction at the
level of PIPJ – Figure 1), direct repair (Figure 2) is usually Discussion: The patient has recurrent instability of the
possible up to 6 weeks post-injury. In Type I injuries (more elbow despite radial head implant arthroplasty. In this
proximal retraction into the palm), reconstructive options particular case the radial head implant is oversized.
may be required if diagnosis is delayed. Anatomic studies have identified that the articular surface
of the radial head is positioned an average of 0.9mm
References: proximal to the lateral edge of the coronoid articular
1. Leddy JP, Packer JW. Avulsion of the profundus insertion in surface. In addition to re-sizing the implant, reconstruction
athletes. J Hand Surg 1977; 2A:66-68. or repair of the lateral collateral ligament is required to
2. Stamos BD, Leddy JP. Closed flexor tendon disruption in stabilize the elbow. If the elbow is still unstable hinged
athletes. Hand Clin 2000; 16:359-365. external fixator application may be indicated. In cases of
chronic elbow dislocation, external fixator application is
often required. Removal of the prosthesis is not a viable
option as it would further destabilize the elbow.

References:
1. Doornberg JN. Reference points for radial head prosthesis size.
J Hand Surg 2006; 31A:53-57.

2. Morrey BF, Tanaka S, An KN. Valgus instability of the elbow. A


definition of primary and secondary restraints. Clin Orthop Relat
Res 1991; 265:187-195.

3. Jupiter JB, Ring D. Treatment of unreduced elbow dislocations


with hinged external fixation. J Bone Joint Surg Am 2002;
Question 75, Figure 1 84(9):1630-5.

Question 75, Figure 2


Question 76, Figure 1

2009 Self-Assessment Examination | 39


77. A patient with long-standing rheumatoid arthritis 78. Which of the following best describes the
presents with a swan-neck deformity of the recommended guidelines for an HIV positive
index finger and complains of painful snapping hand surgeon who would like to continue the
with attempted flexion of the digit. Examination practice of medicine:
demonstrates that the deformity is passively
A. The hand surgeon should avoid all contact
correctable. Radiographs of the involved digit with patients
reveal minimal arthrosis. What is the most
appropriate treatment: B. The hand surgeon should be able to return to all
of the usual and customary duties as an operating
A. Mobilization and rerouting of the lateral bands deep hand surgeon
to the A3 pulley
C. If the patient is informed about the HIV positive
B. Proximal interphalangeal joint capsulodesis status and consents to treatment, then the
C. Proximal interphalangeal joint arthrodesis physician should be able to engage in all duties as
an operating hand surgeon
D. Extensor tendon tenolysis with dorsal transfer of the
lateral bands D. A physician that is HIV positive should not engage
in any activity that creates an identified risk of
E. Proximal interphalangeal joint capsulectomy
transmission of the disease
Preferred Response: A E. If the physician is seropositive and demonstrates
no active signs of HIV disease, then the physician
Discussion: In surgical correction of flexible swan-neck
should be able to return to all the usual and
deformities, the lateral band is positioned palmar to
customary duties as an operating hand surgeon
the axis of rotation of the PIP joint. The procedure is a
motion sparing operation which maintains the PIP joint Preferred Response: D
in a resting posture of approximately 35° of flexion.
Arthrodesis of the PIP joint may be indicated in cases of Discussion: A physician who is HIV positive must
rigid deformity or significant arthritic deterioration of the recognize activities that place the patient at risk. It is
joint. Attenuation and synovitis of the palmar restraints of recommended that the physician consult with colleagues
the PIP joint in patients with rheumatoid arthritis would to determine which activities the physician can engage in
preclude a capulodesis procedure. without creating risk to patients.

References: References:
1. Code of Medical Ethics, Current Opinions with Annotations,
1. Gainor BJ, Hummel GL. Correction of rheumatoid swan-neck
American Medical Association, 1997.
deformity by lateral band mobilization. J Hand Surg 1985;
10A:370-376. 2. Health Care Workers Infected With the Human Immunodeficiency
Virus. The Next Steps. JAMA 1992; 268(5):601.
2. Tonkin MA, Hughes J, Smith KL. Lateral band translocation for
swan-neck deformity. J Hand Surg 1992; 17A:260-267.

79. The histology of giant cell tumor of tendon sheath


is most similar to which of the following:
A. Atypical mycobacterial infection
B. Rheumatoid arthritis
C. Pigmented villonodular synovitis
D. Synovial cell sarcoma
E. Malignant fibrous histiocytoma

Preferred Response: C

Discussion: Histological and immunohistochemical


analysis support a common synovial cell origin for both
GCTS and PVNS.

Reference:
O’Connell JX, Fanburg JC, Rosenberg AE. Giant cell tumor of tendon
sheath and pigmented villonodular synovitis: immunophenotype
suggests a synovial cell origin. Hum Pathol 1995 Jul; 26(4):429-30.
40 | American Society for Surgery of the Hand
80. An 80 year-old man presents complaining of 81. The parascapular flap is based on which artery:
severe wrist pain that makes it difficult to sleep.
A. The descending branch of the circumflex
He has used a wrist splint for years, but it is no scapular artery
longer adequate. He has a 25º arc of wrist motion.
His radiographs are shown in Figure 1. What do B. The transverse branch of the circumflex
scapular artery
you recommend?
C. The angular branch of the thoracodorsal artery
A. Scaphoid excision
D. The serratus anterior branch of the
B. Arthrodesis of the lunate and capitate
thoracodorsal artery
C. Scaphoid excision and arthrodesis of the lunate
E. The vertical branch of the thoracodorsal artery
and capitate
D. Total wrist arthrodesis Preferred Response: A
E. Proximal row carpectomy Discussion: There are several flaps based upon the
circulation around the scapula. Each is based off of a
Preferred Response: D
specific vessel.
Discussion: Since the primary goal is pain relief, there
Reference:
is very little remaining motion, and motion-sparing
procedures typically lose additional motion, total wrist Gilbert A, Teot L. The free scapuar flap. Plast Reconstr Surg 1982;
69(4):601-4.
arthrodesis is the best option for this patient. Wrist
arthrodesis has a high fusion rate when using plates.
Patients experience improved pain relief and function.

References:
1. Weiss KE, Rodner CM. Osteoarthritis of the wrist. J Hand Surg
2007; 32A(5):725-46.

2. Hastings H II, Weiss AP, Quenzer D, Wiedeman GP, Hanington


KR, Strickland JW. Arthrodesis of the wrist for post-traumatic
disorders. J Bone Joint Surg 1996; 78A(6):897-902.

Question 80, Figure 1

2009 Self-Assessment Examination | 41


82. The injury shown in the figures is at greatest 83. Which of the following scaphoid fractures requires
risk for: operative treatment?
A. Nonunion A. Non-displaced distal pole
B. Growth arrest B. Displaced tubercle fracture
C. Infection C. Non-displaced waist fracture
D. Tendon detachment D. Displaced waist fracture
E. Instability E. Non-displaced proximal pole fracture

Preferred Response: C Preferred Response: D

Discussion: The nail and nail plate often avulse from Discussion: Scaphoid fractures account for 60% of all carpal
under the eponychial fold (Figures 3 and 4) with these fractures. Avascular necrosis is reported in 13 to 50% of
injuries, making them open fractures. The colonized scaphoid fractures with a higher incidence in proximal pole
germinal matrix can then become interposed in the fractures. Nonunion occurs in up to 12% of patients in an
physis (Figure 5) and may become infected. untreated fracture and 10% of operatively treated fractures.

References: Internal fixation is indicated for displaced waist and displaced


1. Waters PM. Operative carpal and hand injuries in children. proximal pole fractures due to their high risk of nonunion
J Bone Joint Surg 2007; 89A(9):2064-74. and AVN. Displacement of more then 1 mm is defined as an
2. Al-Qattan MM. Extra-articular transverse fractures of the base of unstable scaphoid fracture. The most important consideration
the distal phalanx (Seymour’s fracture) in children and adults. during internal fixation with a headless screw is placement of
J Hand Surg 2001; 26B(3):201-6. the screw in the center of the scaphoid. Central placement of
the screw produces union rates approximating 95%.

Nondisplaced scaphoid fractures can be treated by


immobilization for 8 to 12 weeks with a thumb spica cast.
There is no consensus on elbow immobilization in a long arm
cast. The healing rate of nondisplaced waist fractures treated
with cast immobilization is 88 to 95% if treated within 3
weeks of injury. Prospective randomized studies of operative
treatment of acute nondisplaced or minimally displaced
fractures of the scaphoid have failed to demonstrate a clear
benefit of early fixation compared to cast immobilization.
Long term outcomes do not differ between internal fixation
Question 82, Figure 1 Question 82, Figure 4
and cast immobilization. Economic analysis has shown
internal fixation to be better from a social perspective due to
earlier return to work. Fixation of nondisplaced fractures is
therefore not required as it is in displaced waist fractures but
is determined based on patient and surgeon preference after
presenting the available literature evidence to the patient.

Question 82, Figure 2 Displaced tubercle fractures are not associated with
nonunion, AVN or carpal collapse and do not require fixation.

References:
1. Bhat M, McCarthy M, Davis TR, Oni JA, Dawson S. MRI and plain
radiography in the assessment of displaced fractures of the waist
of the carpal scaphoid. J Bone Joint Surg 2004; 86B(5):705-13.
Question 82, Figure 5
2. Szabo RM, Manske D. Displaced fractures of the scaphoid.
Clin Orthop Relat Res 1988; (230):30-8.

3. Kawamura K, Chung KC. Treatment of scaphoid fractures and


nonunions. J Hand Surg 2008; 33A(6):987-997.

Question 82, Figure 3

42 | American Society for Surgery of the Hand


84. Uncontrolled studies claim that radiation 85. The unpredictable long-term outcome of Bennett’s
therapy and corticosteroid injection can slow fracture of the thumb metacarpal is felt to be due
the progression of Dupuytren’s disease. Which in part to unrecognized:
of the following makes uncontrolled studies of
A. Articular impaction
Dupuytren’s disease difficult to interpret?
B. Volar ligament injury
A. Regression to the mean
C. Trapezial injury
B. The placebo effect
D. Trapezoid injury
C. The nocebo effect
E. Nonunion
D. The Hawthorne effect
Preferred Response: A
E. The natural history of untreated disease
Discussion: Most Bennett’s fractures have a good short
Preferred Response: E
and long-term outcome. Arthrosis and pain have been
Discussion: When the natural history of a disease is variable, ascribed to more extensive articular injury, which is
uncertain, or incompletely defined, nothing can be said about often underappreciated.
the efficacy of treatment (particularly prophylaxis) without
References:
a control group. All diseases are variably symptomatic and
a lessening of symptoms may just be a transition from a 1. Oosterbos CJ, de Boer HH. Nonoperative treatment of Bennett’s
fracture: a 13-year follow-up. J Orthop Trauma 1995; 9(1):23-7.
peak or waxing of symptoms towards the average level
of symptoms. The placebo effect takes place when a patient 2. Thurston AJ, Dempsey SM. Bennett’s fracture: a medium to long-
experiences improvement because they believe that they term review. Aust N Z J Surg 1993; 63(2):120-3.
should. The nocebo effect takes place when a patient 3. Buchler U, McCollam SM, Oppikofer C. Comminuted fractures
has a bad experience due to negative expectations. The of the basilar joint of the thumb: combined treatment by external
Hawthorne effect takes place when the result is changed fixation, limited internal fixation, and bone grafting. J Hand Surg
1991; 16A(3):556-60.
by the change in the environment, such as the artificial
study conditions and observations.

References:
1. Harrington A. The Placebo Effect: An Interdisiplinary Exploration.
First Harvard University Press, 1999.

2. Moerman DE. Meaning, Medicine and the “Placebo Effect”.


Cambridge University Press, 2002.

2009 Self-Assessment Examination | 43


86. What is the name of the structure (**) shown in Discussion: Joint forces across the base of the thumb
Figure 1? can be significantly higher than forces with simple
pinch activities. At the IP joint, joint contact forces can
A. Grayson’s Ligament
be 4 times the force generated at the tip. The MP joint
B. Retrovascular Cord will exhibit 6 times the tip strength. However, at 12-13
C. Cleland’s Ligament times the tip strength, the CMC joint will experience the
D. Spiral Cord greatest joint contact forces. Other investigators have
estimated that, depending on posture, between 6 and
E. Pre-tendinous Cord
24 fold increased in load at the CMC joint compared to
Preferred Response: D the applied force. This increased load is a major factor
contributing to the incidence of arthritis at this joint.
Discussion: This patient has Dupuytren’s Disease and
the structure shown is the spiral cord. The spiral cord References:
forms from the pre-tendonis band, spiral band, Grayson’s 1. Cooney WP III, Chao EY. Biomechanical analysis of static forces
ligament and the lateral digital sheet. Spiral cords are in the thumb during hand function. J Bone Joint Surg 1977;
often in the proximity of the neurovascular structures 59:27-36.
and can often displace them. The retrovascular cord 2. Imaeda T, An KN, Cooney WP III. Functional anatomy and
is implicated in causing DIP contractures. Cleland’s biomechanics of the thumb. Hand Clin 1992; 8:9-15.
ligament is typically spared in Dupuytren’s Disease. 3. Giurintano DJ, Hollister AM, Buford WL, Thompson DE, Myers
LM. A virtual five-link model of the thumb. Med Eng Phys 1995;
References: 17:297-303.
1. McGrouther DA. Dupuytren’s contracture. In: Green DP,
Hotchkiss RN, Pederson WC, Wolfe SW, eds. Green’s Operative
Hand Surgery. 5th ed. Philadelphia: Churchill Livingston/Elsevier,
88. Closed reduction of the fracture shown in Figure 1 is
2005:159-185.
best achieved by longitudinal traction, downward
2. MacFarlane RM. Patterns of diseased fascia in the fingers in pressure at the apex of the fracture, extension of
Dupuytren’s contracture. Plast Reconstr Surg 1974; 54:31-44.
the distal fragment and thumb:
A. Pronation
B. Adduction
C. Radial abduction
D. Palmar abduction
E. Supination

Preferred Response: A

Discussion: The deforming forces on extra-articular base


of the thumb fractures result in apex dorsal angulation
Question 86, Figure 1 of the metacarpal. Proximally, the abductor pollicis
longus exerts an extension force on the proximal fracture
fragment. The thumb adductor, flexor pollicis brevis and
87. When pinching between the thumb and index finger abductor pollicis brevis cause the distal fragment to be
with 5 kg load, what is the estimated joint contact flexed and supinated. Up to 30° of angulation is generally
force acting on the trapeziometacarpal joint? pretty well tolerated due to the compensatory motion
of the CMC joint. These fractures are often amenable to
A. 1 kg closed reduction or percutaneous fixation. The deformity
B. 5 kg includes a supination force and gentle pronation of the
C. 10 kg distal fragment (in addition to extension) will achieve a
more anatomic reduction.
D. 20 kg
E. 60 kg

Preferred Response: E

44 | American Society for Surgery of the Hand


References: be performed. If the DRUJ is unstable and there is an
1. Burton RI, Eaton RG. Common hand injuries in the athlete. ulnar styloid fracture, reduction and fixation of the ulnar
Orthop Clin North Am 1973; 4:809-838. styloid (Figure 3) usually affords stability. An irreducible
2. Soyer AD. Fractures of the base of the first metacarpal: current DRUJ suggests either interposition ECU, EDQ or EDC,
treatment options. J Am Acad Orthop Surg 1999; 7:403-412. buttonhole of the ulna through the DRUJ capsule or
periosteal or bony interposition. Another cause of
3. Stern, PJ. Fractures of the metacarpal and phalanges. In:
Green DP, Hotchkiss RN, Pederson WC, Wolfe SW, eds. irreducible DRUJ is malreduction of the radius such
Green’s Operative Hand Surgery. 5th ed. Philadelphia: Churchill as inappropriate restoration of the radial bow. In these
Livingston/Elsevier, 2005:277-341. cases, the interposed structure needs to be removed and
stabilization achieved (if necessary).

References:
1. Giannoulis FS, Sotereanos DG. Galeazzi fractures and
dislocation. Hand Clin 2007; 21:153-163.

2. Hughston JC. Fractures of the distal radial shaft: Mistakes in


management. J Bone Joint Surg 1957; 39:249-64.

Question 88, Figure 1

89. Following open reduction and internal fixation of


the radius in the fracture shown in Figures 1 and 2,
the patient has instability of the distal radioulnar
joint. What is the recommended treatment?
Question 89, Figure 1 Question 89, Figure 2
A. Immobilization with the forearm in full supination
for 6 weeks
B. Open radioulnar joint ligament repair
C. Open reduction and fixation of the ulnar styloid
D. Immobilization with the forearm in full pronation
for 6 weeks
E. Exploration and removal of interposed extensor
carpi ulnaris tendon

Preferred Response: C

Discussion: Some of the most common complications Question 89, Figure 3


following operative treatment of Galeazzi fracture
treatment involve the distal radioulnar joint. The energy
of these injuries typically goes through the DRUJ. Thus
it is extremely important that the stability of this joint
be assessed following fixation of the radius fracture. If
the DRUJ is stable, the forearm can be immobilized in
supination to allow for healing of the stabilizing volar
and dorsal ligaments. If the DRUJ is unstable and
there is no ulnar styloid fracture pinning of the radius
and ulna in a reduced position with TFC repair can

2009 Self-Assessment Examination | 45


90. The patient shown in Figure 1 has acute onset of 91. What is the most likely diagnosis for the bone changes
pain and swelling of the index finger. Diagnosis seen on x-ray of the patient shown in Figure 1?
has been confirmed to be due to uric acid deposition
A. Infection
disease. What would be the most appropriate next
step in improving this patient’s pain? B. Mafucci’s Syndrome
C. Osteoporosis
A. Surgical exploration and debridement
D. Particulate induced osteolysis
B. Distal interphalangeal arthrodesis
E. Vitamin D deficiency
C. Administration of allopurinol
D. Aspiration and drainage with an 18 gauge needle Preferred Response: D
E. Diet modification including avoidance of seafood Discussion: The patient shown in the figure has an
and red meat advanced case of silicone induced osteolysis resulting
Preferred Response: D from the silicone metacarpophalangeal joint arthroplasties
seven years previously. Incidences of osteolysis following
Discussion: Patients with distal interphalangeal joint silicone implant arthroplasty have been reported to be
gouty tophi can often present with significant pain and greater than 50% at average 5.7 year follow-up. Most
swelling. The diagnosis is often confused with infection, of these tend to affect the cortices of the phalanges
which must be ruled out. Confounding things further the and metacarpals. An association between osteolysis
two conditions may coexist. In patients with significant and implant fractures has been observed. Patients with
swelling and skin changes, surgical treatment can be severe osteolysis have an increased risk of metacarpal
problematic and carry an increased risk of delayed and phalangeal fractures.
wound healing. Aspiration with a large bore needle
through an area of uninvolved skin is a relatively safe References:
alternative to facilitate decompression of the tophus 1. Parkkila TJ, et al. Grading of radiographic osteolytic changes
after silastic metacarpophalangeal arthroplasty and a prospective
while avoiding the complications of surgery. DIP joint
trial of osteolysis following use of Swanson and Sutter
arthrodesis may be helpful in cases where there is prostheses. J Hand Surg 2005; 30B:382-7.
persistent pain and arthritis following subsidence of the
2. Parkkila T, et al. Osteolysis after Sutter metacarpophalangeal
flare. Allopurinol and diet modification can be considered
arthroplasty: a prospective study of 282 implants followed up
in the long-term to minimize recurrence of flares, but for 5.7 years. Scand J Plast Reconstr Surg Hand Surg 2006;
would be of little use in the acute setting. 40:297-301.

References:
1. Mudgal CS. Management of tophaceous gout of the distal
interphalangeal joint. J Hand Surg 2006; 31B:101-103.

2. Fitzgerald BT, Setty A, Mudgal CS. Gout affecting the hand and
wrist. J Am Acad Orthop Surg 2007; 15:625-35.

Question 91, Figure 1

Question 90, Figure 1

46 | American Society for Surgery of the Hand


92. The best surgical treatment option for the lesion 93. A 33 year-old female presents to the emergency
shown in Figure 1 is: room with a 13 cm ecchymotic lesion with small
superficial open wounds over the volar aspect
A. Primary repair of the tendon
of her forearm and wrist. She recalls a brown
B. Tendon grafting with a palmaris longus tendon graft recluse spider bite 3-4 days prior. She states that
C. Staged tendon reconstruction using silicone rods the wound has just developed over the past 24
D. Arthrodesis of the thumb interphalangeal joint hours, but the area of ecchymosis has not grown.
Her vital signs and lab work are normal and she
E. FPL tenodesis
complains of mild discomfort. The next step in
Preferred Response: B management should be:

Discussion: The lesion shown in Figure 1 is a flexor A. Administration of Dapsone


pollicis longus (FPL) rupture or Mannerfelt Lesion. This B. Immediate wide surgical debridement
injury is often due to arthritis and osteophytes at the C. Topical corticosteroids
scaphotrapeziotrapezoid (STT) joint. The FPL glides
D. Administration of anti-venom
superficial to this joint and is vulnerable to attritional
changes secondary to the arthritis. Because of the E. Clean wounds and apply sterile dressings
attritional nature of these injuries, primary repair is usually Preferred Response: E
not feasible. Reconstruction with tendon graft, or tendon
transfer (mainly the FDS of the ring or long finger) are Discussion: Brown recluse spider bites are most common
typically the best treatment options. in the south and the midwestern parts of the United
States. The bites can result in painful, necrotic and
References: slow-healing wounds. One study examining outcomes
1. Mannerfelt L, Norman O. Attrition ruptures of flexor tendons in of recluse spider bites in the upper extremity noted a
rheumatoid arthritis caused by bony spurs in the carpal tunnel.
20% incidence of functional complications. DeLozier
A clinical and radiological study. J Bone Joint Surg 1969;
51B(2):270-7. et al noted that patients who underwent early surgical
debridement had more complications than those who
2. Ertel AN, et al. Flexor tendon ruptures in patients with rheumatoid
were more conservatively treated. Another report
arthritis. J Hand Surg 1988; 13A(6):860-6.
suggested no benefit with dapsone, corticosteroids,
nitroglycerin or high dose Vitamin C. In fact, dapsone
and steroids were associated with slower healing rates
and dapsone increased the possibility of scarring.
Predictors of more rapid healing include: lower severity
level, less erythema & necrosis at presentation, the
absence of diabetes, younger age and earlier medical
attention. While systemic complications can occur,
they are rare. Wright et al reported only two cases (of
111) with systemic complications: one patient with mild
hemolytic anemia and another with mild coagulopathy.
Most caregivers recommend conservative wound care in
the immediate setting. Delayed surgical management of
persistent or infected wounds is preferred.

Question 92, Figure 1 References:


1. DeLozier JB, Reaves L, King LE, Rees RS. Brown recluse spider
bites of the upper extremity. South Med J 1988; 81:181-4.

2. Wright SW, Wrenn KD, Murray L, Seger D. Clinical presentation


and outcome of brown recluse spider bite. Ann Emerg Med 1997;
30:28-32.

3. Mold JW, Thompson DM. Management of brown recluse spider


bites in primary care. J Am Board Fam Pract 2004; 17:347-52.

2009 Self-Assessment Examination | 47


94. The free lateral arm flap is perfused by the: sided elbow pain, decreased control, and reduced
throwing velocity. Injury to the medial collateral ligament
A. Brachial artery
can be confirmed by physical examination (moving valgus
B. Ulna artery stress test) and appropriate imaging studies (computed
C. Posterior radial collateral artery tomography arthrogram and magnetic resonance imaging).
D. Radial recurrent artery References:
E. Posterior interosseous artery 1. Lynch, J et al. Medial collateral ligament injury in the overhand
throwing athlete. J Hand Surg 2008; 33A:430–437.
Preferred Response: C
2. Cain EL Jr, Dugas JR, Wolf RS, Andrews JR. Elbow injuries in
Discussion: The lateral arm is perfused by the posterior throwing athletes: a current concepts review. Am J Sports Med
radial collateral artery (PRCA), one of the terminal 2003 Jul-Aug; 31(4):621-35.
branches of the profunda brachii. It has a large fascial
component located anterior and posterior to the lateral
intermuscular septum, which itself lies between the 96. A mother seeks consultation regarding her 5 year-
triceps posteriorly and the brachialis and brachioradialis old son who has a unilateral congenital forearm
muscles anteriorly. The PRCA consistently divides into amputation. She desires information for her child
two terminal branches—an anterior and a posterior regarding long-term prosthetic use. Your best
division. The anterior division is the nutrient vessel of advice is:
the flap. It can be used as a composite osteocutaneous A. A myoelectic hand will be the best choice
flap for immediate thumb reconstruction with or without
B. The child will never use a prosthetic hand
vascularized tendon. No major artery is sacrificed by
using this flap and the vascularity of the upper extremity C. The child will be most likely to use a passive
is unaffected. cosmetic hand
D. Bimanual function will be close to normal if a
Reference:
prosthesis is fitted immediately
Katsaros J, Tan E, Zoltie N. The use of the lateral arm flap in upper
limb surgery. J Hand Surg 1991; 16A:598-604. E. He will not use a body powered device if he is fitted
with a myoelectric hand first

Preferred Response: C
95. In throwing athletes, the late cocking phase of the
overhand throw has the greatest risk for injury to: Discussion: Seventy-six children with unilateral below-
elbow amputation were fitted in random sequence with a
A. The anterior bundle of the medial collateral ligament
myoelectric (MYO) and a body-powered (BP) prosthetic
B. The posterior bundle of the medial collateral ligament hand of identical size, shape, and glove color. Subjects
C. The anterior bundle of the lateral collateral ligament ranged from six to 17 years, nine months and included
D. The common flexor-pronator mass 67 children with congenital limb deficiency and nine who
sustained traumatic amputation. After training, each child
E. The arcuate ligament overlying the epicondylar groove
wore each hand for three months. On the form board test
Preferred Response: A requiring only prosthetic use, subjects took 13.7% longer
with the MYO and committed more errors with the MYO,
Discussion: Medial collateral ligament injuries occur specifically in dropping objects and delaying their grasp
almost exclusively in overhand-throwing athletes. The late and release. Object displacement, the most common
cocking phase of the overhand throw places a marked error, occurred nearly as often with BP as MYO. MYO
valgus moment across the medial elbow. This repetitive was minimally faster on a test of ten practical activities
force reaches the tensile limits of the medial collateral designed for bimanual prehension. Card playing was
ligament, subjecting it to microtraumatic injury and 39.8% faster with BP, whereas donning socks, cutting
attenuation. The anterior bundle of the medial collateral paper, and bandage application were 27.8%, 12.5%, and
ligament has been identified as the primary restraint to 10.9% faster with MYO. Performance with both hands
valgus load and is the focus of reconstruction. Diagnosis was rated as decidedly poorer than normal quality. No
of medial collateral ligament injuries should be suspected major clinically important differences were found in the
in any overhand throwing athlete with a history of medial- comparison of performance.

48 | American Society for Surgery of the Hand


Thirty-four unilateral below-elbow amputees from 97. During exploration of the ulna nerve (“U”) at the
the Shriners Hospitals for Children/Twin Cities were elbow for ulna nerve compression, a structure was
retrospectively analyzed in long-term follow-up. All encountered. What is the structure marked “**”?
of these patients were provided with a variety of
A. Arcade of Struthers
prosthetic options, including a “passive” cosmetic upper
extremity device. Most of the patients were also fitted B. Gantzer’s muscle
with conventional prostheses using a body-powered C. Ligament of Struthers
voluntary closing terminal device (97%) as well as D. Anconeus epitrochlearis muscle
myoelectric prostheses (82%). The average follow-up
E. Triceps muscle
was 14 years, with many of the patients being followed
up throughout their entire childhood. All patients Preferred Response: D
were sent questionnaires, and the authors carried
out patient interviews and chart review. Final analysis Discussion: The anconeus epitrochlearis muscle is
indicated that 15 patients (44%) selected a simple an anomalous muscle that arises from medial border
cosmetic “passive hand” as their prosthesis of choice. of olecranon and adjacent triceps and inserts into the
In long-term follow-up, 14 patients (41%) continued medial epicondyle. It may be found in about 10% of
as multiple users. Fourteen patients (41%) selected patients undergoing cubital tunnel release. It may cause
the conventional prosthesis using a voluntary closing ulnar neuritis.
terminal device as the prosthesis of choice. Only five References:
patients (15%) selected the myoelectric device as their
1. Hirasawa Y, Sawamura H, Sakakida K. Entrapment neuropathy
primary prosthesis. The authors conclude that successful due to bilateral epitrochleoanconeus muscles: a case report.
unilateral pediatric amputees may choose multiple J Hand Surg 1979; 4:181-184.
prostheses on the basis of function and that frequently
2. Dahners LE, Wood FM. Anconeus epitrochlearis, a rare cause
the most functional prosthesis selected in the long term is of cubital tunnel syndrome: a case report. J Hand Surg 1984;
the simplest in design. 9A:579-580.

References: 3. Masear VR, Hill JJ and Cohen SM. Ulnar compression


neuropathy secondary to the anconeus epitrochlearis muscle.
1. Edelstein JE, Berger N. Performance comparison among children
J Hand Surg 1988; 13A:720-724.
fitted with myoelectric and body-powered hands. Arch Phys Med
Rehabil 1993; 74(4):376-80.

2. Crandall RC, Tomhave W. Pediatric unilateral below-elbow


amputees: retrospective analysis of 34 patients given multiple
prosthetic options. J Pediatr Orthop 2002; 22(3):380-3.

Question 97, Figure 1

2009 Self-Assessment Examination | 49


98. A patient with the ulna styloid fracture in Figure 1 99. When using the EIP for a tendon transfer, the EIP
is more likely to have DRUJ instability than a can be most reliably identified by:
patient with an ulna styloid fracture at the tip
A. Its position relative to the EDC tendon at the index
because which of the following structures is more MP joint
likely to be disrupted?
B. Having the most distal muscle belly
A. The dorsal radioulnar ligaments
C. Its size relative to the EDC tendon
B. The volar radioulnar ligaments
D. Presence of tendon interconnections
C. The ligamentum subcruentum
E. Its shape relative to the EDC tendon
D. The ECU subsheath
Preferred Response: B
E. The meniscus homologue
Discussion: The EIP tendon is an excellent transfer
Preferred Response: C
for a ruptured EPL. The index finger will reliably retain
Discussion: When the ulna styloid fracture is at the base independent extension. The EIP is usually ulnar to the
as shown in Figure 1 the deep portion of the TFCC is EDC, but not always (Figure 1). The EIP muscle belly
disrupted. This will lead to more instability than just a is reliably the most distal muscle (72/72). In this study,
fracture at the tip1. The volar and dorsal radius-ulna ligament it was never dorsal but could be volar. It may have
and the meniscus and ECU may be injured with either injury. interconnections. The EIP may also have more than one
Although technically not a ligament, the term “Ligamentum tendon slip (Figure 2).
subcruentum” has come to represent the deep fibers of
Reference:
the TFC (inserting into the fovea – Figure 2) and is now
used commonly by many investigators as interchangeable Gonzalez MH, Weinzweig M, Kay T, Grindel S. Anatomy of the
extensor tendons to the index finger. J Hand Surg 1996;
with the term “deep TFC radio ulna ligaments”2.
21A:988-991.
References:
1. Haugstvedt R, Berger RA, Berglund LJ, Neale PG, Sabick MB.
An analysis of the constraint properties of the distal radioulnar
ligament attachments to the ulna. J Hand Surg 2002; 27(1):61-67.

2. Kleinman WB. Stability of the distal radioulna joint: biomechanics,


pathophysiology, physical diagnosis, and restoration of function what
we have learned in 25 years. J Hand Surg 2007; 32(7):1086-1106.

Question 98, Figure 1

Question 98, Figure 2


Question 99, Figure 1. Relative position of the extensor tendons at
the index metacarpophalangeal joint: six variations.

50 | American Society for Surgery of the Hand


101. Which opponensplasty requires creation of
a pulley?
A. Abductor digiti minimi
B. Adductor pollicis
C. Extensor indicis propius
D. Extensor carpi radialis longus
E. Flexor digitorum superficialis

Preferred Response: E

Discussion: The FDS and EIP are common choices for


opponensplasty. The FDS needs to have a pulley on the
ulna side of the wrist. The EIP uses the ulna side of the
forearm as a pulley and a pulley does not have to be
constructed. The EIP is relatively expendable and usually
independent index extensor function is maintained. It is
weaker than the FDS, but has adequate strength for this
positioning transfer. The disadvantages of an EIP transfer
Question 99, Figure 2. Accessory slip of extensor indicis proprius
include the need for multiple incisions, the transfer,
(EIP) tendon. Tendon running to long finger.
passes over the median nerve, and if there is scarring
over the ulna forearm adhesions may develop.

100. Surgical treatment of an extension contracture to References:


the MP joint requires: 1. Smith RJ. Tendon transfer of the hand and forearm. Little Brown
Co., 1987:67-70.
A. Release of the check rein ligaments
2. Chhabra AB, Dorf ER. Tendon transfer surgery. In: Hand Surgery
B. Release of the volar plate Update IV. American Society for Surgery of the Hand, Rosemont:
C. Checking the flexor tendon excursion 2007:439-454.

D. Dorsal capsulotomy
E. Preservation of the collateral ligaments

Prerferred Response: D

Discussion: MP joint contractures are common and


usually in extension rather than flexion. There are no
checkrein ligaments to release and usually the volar
plate is not significantly involved. The flexor tendons
do not have to be checked if there is good PIP active
flexion since there are not the primary flexors of the
MP joint. The collateral ligaments almost always have
to be partially released and postoperative instability
is uncommon.

Reference:
Shin AY, Amadio PC. Stiff Finger Joints. In: Green DP, Hotchkiss RN,
Pederson WC, Wolfe SW eds. Green’s Operative Hand Surgery. 5th ed.
Philadelphia: Churchill Livingston/Elsevier, 2005:414.

2009 Self-Assessment Examination | 51


102. A 28 year-old professional chef amputated the 103. A 38 year-old mountain climber has a frostbite
tip of her dominant left ring finger. Examination involving both hands. He is evacuated to
revealed an oblique amputation of soft tissue a mountain first aid station, but cannot be
involving the pulp distal to the DIP crease with transported by snowmobile to a permanent facility
an intact nail plate and exposed bone (Figure 1). for 12 hours. What treatment should wait until he
Recommended treatment is: is in the permanent facility?
A. Debridement and primary closure A. Core body temperature rewarming
B. Skin graft B. Tetanus prophylaxis
C. A V-Y advancement flap C. Cold water baths
D. Volar advancement flap D. Rapid rewarming at 40-44° C
E. Thenar flap E. Oral antibiotic to prevent infection

Preferred Response: E Preferred Response: D

Discussion: There are many options for care of fingertip Discussion: Frostbite is one of the two major groups of
injuries. In this young woman with exposed distal phalanx, cold injuries. This is usually associated with prolonged
loss of significant volar pulp and intact nail plate optimal cold exposure, inadequate nutrition, fatigue and some
treatment would be to maintain length. A skin graft would degree of panic. Early treatment includes restoration
not be appropriate because of exposed bone. A V-Y of normal core temperatures, management of shock,
advancement flap is not appropriate because of the amount fractures and malnutrition. The frostbitten area must be
of soft tissue loss. The volar advancement flap is best protected. Specific treatment of deep frostbite should
for the thumb because of the excellent dorsal circulation. In only be undertaken when adequate ongoing care without
the finger, it has been unreliable and may be complicated re-exposure to cold is available. Rapid rewarming at
by dorsal skin loss. A cross finger flap from an adjacent 40-44° C is the most important step in salvage of tissue
finger is an alternative, but not listed as an answer for and function, but should only be performed when the
this question. A cross finger flap does require a skin involved part will not be exposed to the cold. Tetanus
graft to cover the donor finger. The thenar flap offers prophylaxis is a part of the management of frostbite.
several advantages including palmar skin match, adequate Antibiotics are only used for a confirmed infection.
soft tissue, and an inconspicuous donor site. With the Cold-water baths and rubbing snow on the injured part
H-modification, it can be closed primarily without a skin are not part of the treatment for frostbite.
graft. The flap is divided and inset at 2-3 weeks and range
of motion started. The complication of joint stiffness and References:
flexion contracture may be lessen with early motion, 1. Frostbite. In: Green DP, Hotchkiss RN, Pederson WC, Wolfe
SW eds. Green’s Operative Hand Surgery 5th ed. Philadelphia:
therapy, and patient choice (young, without arthritis or preop
Churchill Livingston/Elsevier, 2005:2186-2187.
stiffness and women who may be more flexible than men).
2. Mills WJ Jr. Frostbite: a method of management including rapid
References: rewarming, Northwest Med 1966; 65:119-125.
1. Watt AJ, Chang J. Fingertip Injuries and Amputations. In: Hand
Surgery Update IV. American Society for Surgery of the Hand,
Rosemont: 2007:505-507. 104. The gracilis muscle can be used as a functioning
2. Melone CG, Beasley RW, Carstens JH. The thenar flap: an free muscle transfer. The innervation of the gracilis
analysis of its use in 150 cases. J Hand Surg 1982; 7:291-297. muscle is by the:
A. Pudendal nerve
B. Femoral nerve
C. Obturator nerve
D. Tibial nerve
E. Peroneal nerve

Preferred Response: C

Question 102, Figure 1

52 | American Society for Surgery of the Hand


Discussion: The gracilis is the most suitable muscle a result of patient follow-up. However, if follow-up is
for functional muscle transfers. There is a single “nondifferential”—that is, not related to the exposure or
motor nerve—a branch of the obturator nerve—that is outcome of interest—then an important selection bias
composed of two or three fascicles. The nerve enters the might not be present.
muscle immediately proximal to the vascular pedicle and
Reference:
lies under the adductor longus. With nerve stimulation,
the adult gracilis muscle will shorten more than 50% of its Spindler KP, Kuhn JE, Dunn W, Matthews CE, Harrell, Jr FE,
Dittus RS J. Reading and reviewing the orthopaedic literature: a
extended length for a functional contraction of 12 to 15
systematic, evidence-based medicine approach. Am Acad Ortho
cm. The fascicles of the motor nerve can be teased apart Surg 2005; 13:220-229.
and each fascicle separately stimulated. By using a nerve
stimulator with frequency and voltage control, it was
usually possible to separate the muscle into longitudinal, 106. A 10 year-old female with cerebral palsy presents
separately functioning neuromuscular territories. 90% of with persistent wrist flexion posture. Physical
the time, a single fascicle will control the anterior 20% examination reveals mild wrist flexion and ulnar
to 50% of the muscle, and the remaining portion of the deviation posturing, but with near full active range
muscle is controlled by the remaining fascicles. This of motion. Motion laboratory evaluation revealed
functional separation is quite useful when the muscle is phasic firing of the flexor carpi ulnaris and
used to provide independent thumb and finger flexion. extensor carpi radialis brevis and longus. The next
Reference: most appropriate treatment is:
Manktelow RT, Anastakis DJ. Free Functioning Muscle Transfers. A. Observation only
In: Green DP, Hotchkiss RN, Pederson WC, Wolfe SW eds. Green’s
B. Botulinum toxin injection to FCU
Operative Hand Surgery. 5th ed. Philadelphia: Churchill Livingston/
Elsevier, 2005:1757. C. Lengthening of FCU
D. FCU to ECRB transfer
E. FCU to ECRB transfer with proximal row carpectomy
105. Selection bias in clinical trials is reduced by:
A. Randomization by the surgeon Preferred Response: B

B. Blind randomization Discussion: The wrist flexion contracture is mild, and the
C. Randomization by day of the week patient has full range of motion. Observation alone would
not correct the problem, and lengthening or transfer of
D. Similarity of comparison populations
the FCU is unnecessary when only a mild contracture
E. Decreased population size exists. While not yet approved by the FDA for this use,
Preferred Response: D the next step of botulinum toxin injection is appropriate to
minimize phasic firing of the FCU.
Discussion: Selection bias may be introduced when
the populations under study are first assembled. It is The FDA has not approved Botulinum toxin injection
critically important to be sure that the two populations are for this clinical use.
similar. When they are different, the researcher cannot be References:
certain that the difference in the effect of an intervention 1. Yang TF, Fu CP, Kao NT, Chan RC, Chen SJ. Effect of botulinum
is the result of the intervention or the result of different toxin type A on cerebral palsy with upper limb spasticity. Am J
characteristics of the two groups. To reduce the likelihood Phys Med Rehabil 2003; 82(4):284-9.
of selection bias, prospective studies should have well- 2. Van Heest AE, Ramachandran V, Stout J, Wervey R, Garcia
defined entry criteria and recruitment processes. Clinical L. Quantitative and qualitative functional evaluation of upper
trials are best performed by randomly assigning patients extremity tendon transfers in spastic hemiplegia caused by
to treatment groups. There are also pseudorandom cerebral palsy. J Pediatr Orthop 2008; 28(6):679-83.
methods of assigning patients that may be subject to
bias (e.g., by surgeon, by the days of the week that
patients present to the clinic). Even randomized trials
may be at risk for a group assignment bias if clinicians
are able to predict the next group assignment in a
clinical trial. Another form of selection bias occurs as

2009 Self-Assessment Examination | 53


107. Dynamic splinting, when compared with passive References:
splinting, for zone VI extensor tendon repairs has 1. Egloff DV, Verdan C. Pollicization of the index finger for
been shown to improve: reconstruction of the congenitally hypoplastic or absent thumb.
J Hand Surg 1983; 8A(6):839-48.
A. Short and long-term range of motion
2. McDonald TJ, James MA, McCarroll HR Jr, Redlin H.
B. Short-term but not long-term range of motion Reconstruction of the type IIIA hypoplastic thumb. Tech Hand Up
C. Neither short-term nor long-term range of motion Extrem Surg 2008; 12(2):79-84.

D. The incidence of early tendon rupture


E. The incidence of late tendon rupture

Preferred Response: B

Discussion: While dynamic splinting may improve


short-term range of motion post-surgical repairs of
extensor tendons, studies have failed to demonstrate
better long-term outcome.

References:
1. Mowlavi A, Burns M, Brown RE. Dynamic versus static splinting
of simple zone V and zone VI extensor tendon repairs: a
prospective, randomized, controlled study. Plast Reconstr Surg
2005; 115(2):482-7. Question 108, Figure 1
2. Bulstrode NW, Burr N, Pratt AL, Grobbelaar AO. Extensor tendon
rehabilitation a prospective trial comparing three rehabilitation
regimes. J Hand Surg 2005; 30B(2):175-9.

108. In choosing the most appropriate reconstruction of


the problem shown in Figures 1 and 2, the surgical
treatment involves ligation of the:
A. Radial digital artery to the index finger
B. Ulnar digital artery to the index finger
C. Radial digital artery to the long finger
D. Ulnar digital artery to the long finger
E. Radial digital artery to the ring finger Question 108, Figure 2

Preferred Response: C

Discussion: In creating the index finger rotational flap,


the finger is tethered by the radial digital artery to the long
finger. Ligating this branch will allow the common digital
nerve to the second web to rotate with the index into the
thumb position.

54 | American Society for Surgery of the Hand


109. A 38 year-old female sustained a thermal burn to 110. A 35 year-old male presents to the emergency
her upper extremity (Figures 1 and 2). The hand department after sustaining a fingertip injury while
is vascular and sensate. Definitive management cleaning his paint gun. Examination (Figure 1)
for treatment of this injury leading to the best reveals a 2 mm wound at the left long fingertip
outcome must include: and a swollen digit. The next management step
should be:
A. Dressing changes
B. Split thickness skin grafting A. Elevation of the left hand and arm

C. Full thickness skin grafting B. Compartment pressure measurement in the finger

D. Free vascularized flap coverage C. Needle aspiration

E. Amputation D. Limited incision to enlarge the puncture wound


E. Wide surgical debridement of the entire digit
Preferred Response: D
Preferred Response: E
Discussion: The deeply burned forearm in the figures
shown is best reconstructed with free flap coverage due Discussion: The injury represents a high-pressure
to exposed tendons and other deep structures that lack a injection. The wound will often appear innocuous.
healthy vascular bed. Skin grafting will have poor viability Emergency open debridement reduces the risk of
over such structures, and amputation is drastic when amputation.
there is a functional hand.
References:
References: 1. Hogan CJ, Ruland RT. High-pressure injection injuries to the
1. Tredget EE. Management of the acutely burned upper extremity. upper extremity: a review of the literature. J Orthop Trauma 2006;
Hand Clin 2000; 16(2):187-203. 20(7):503-11.

2. Takeuchi M, Nozaki M, Sasaki K, Nakazawa H, Sakurai H. 2. Bekler H, Gokce A, Beyzadeoglu T, Parmaksizoglu F. The surgical
Microsurgical reconstruction of the thermally injured upper treatment and outcomes of high-pressure injection injuries of the
extremity. Hand Clin 2000; 16(2):261-9. hand. J Hand Surg Eur Vol. 2007; 32(4):394-9.

Question 109, Figure 1


Question 110, Figure 1

Question 109, Figure 2

2009 Self-Assessment Examination | 55


111. The diagnosis of extrinsic flexor tendon tightness 112. A 45 year-old right hand-dominant male presents
on physical examination is best demonstrated four months after a fall from a ladder complaining
with the wrist and digits in what position? of stiffness of his right ring finger. The patient
recalls immediate pain and a “tearing” sensation
A. Wrist extension, MP extension
prior to losing his grip. The patient reports a
B. Wrist extension, MP flexion progressive loss of digital extension, despite three
C. Wrist flexion, MP extension months of supervised hand therapy. Physical
D. Wrist flexion, MP flexion examination demonstrates active PIP joint
motion from 60º to 100º. Plain radiographs are
E. Wrist neutral, MP flexion
normal. MRI is shown in Figure 1. Recommended
Preferred Response: A treatment is:

Discussion: Extrinsic flexor tendon tightness most A. Profundus tendon repair


commonly follows trauma to the extrinsic flexors, either B. Staged flexor tendon reconstruction
within the hand or forearm. A volar forearm mass or flexor C. PIP joint volar release
tenosynovitis may also result in extrinsic flexor tendon
D. PIP joint volar release and extensor tenolysis
tightness secondary to a mass effect. It is most often
noted following flexor tendon injury and subsequent E. PIP joint volar release and pulley reconstruction
repair secondary to scarring and shortening of the Preferred Response: E
muscle/tendon unit. With the wrist and MP joints in full
extension, in the absence of digital contracture, the PIP Discussion: Originally described by Bollen in 67 British
and DIP joints will be tight; and in some cases, it will rock climbers, closed ruptures of the digital flexor pulley
not be possible to fully extend the PIP and DIP joints. system often presents in a delayed fashion with PIP
With the wrist and MP joints in flexion, full PIP and DIP joint flexion contractures. The mechanism of injury is
extension should be possible. a rapidly applied extension force in the acutely flexed
digit, resulting in a closed rupture of the retinacular
References: sheath, rather than a flexor profundus avulsion. In the
1. Curtis RM. Capsulectomy of the interphalangeal joints of the setting of a significant PIP flexion contracture, one most
fingers. J Bone Joint Surg 1954; 36A:1219-1232.
commonly finds a rupture of multiple pulleys, including
2. Lister G. The Hand: Diagnosis and Indications, 4th ed. Churchill A2, A3 and A4. In such a case, significant flexor tendon
Livingstone, London, 2002. bowstringing results precluding successful non-operative
management. The proper treatment includes release of
the sheath scar and pulley reconstruction. Temporary PIP
joint pinning may also be required.

References:
1. Bollen SR. Soft tissue injury in extreme rock climbers. Br J Sports
Med 1988; 22:145-147.

2. Bowers WH, Kuzma GR, Bynum DK. Closed traumatic rupture of


finger flexor pulleys. J Hand Surg 1994; 19A:782-787.

Question 112, Figure 1


56 | American Society for Surgery of the Hand
113. What is the most appropriate indication for the use 114. To best limit occupational radiation scatter to the
of a skin graft in Dupuytren’s disease? hand surgeon while using the mini C-arm, the
surgeon should:
A. 50° primary MP joint contracture
B. 50° primary PIP joint contracture A. Wear a lead apron at all times

C. 50° recurrent MP contracture with isolated cord B. Orient the unit horizontally and stay on the source
side of the C-arm during imaging
D. 50° recurrent PIP contracture with isolated cord
C. Increase the kVp and mA to improve the contrast of
E. 50° recurrent PIP contracture with diffuse a dark image
skin involvement
D. Increase the distance of the patient’s extremity from
Preferred Response: E the x-ray source

Discussion: Skin grafts are especially useful in E. Not be concerned since radiation exposure is a
Dupuytren’s patients with severe, diffuse disease and non-issue while using the mini C-arm
in recurrences—particularly when multiple joints are Preferred Response: D
involved. Though rarely required in primary cases, if
the disease is infiltrative and diffuse, skin grafts can Discussion: Occupational radiation exposure continues
be useful. Full thickness skin grafts are preferred, to concern all of us who regularly utilize in-office and
providing more esthetic and durable skin coverage while intra-operative fluoroscopy. Until fairly recently, the
exhibiting less tendency to retract than partial thickness literature regarding the use of the mini C-arm has been
grafts. Most authors have observed that recurrence is lacking. The potential risk of low dose radiation exposure
uncommon beneath a graft, though a recent article by remains unclear, but it is clear that we should take steps
Roush and Stern did not find that full thickness skin to minimize radiation exposure.
grafting prevented recurrences. Grafts rarely do not
Scattered radiation is radiation that arises from interaction
“take,” despite placing them directly over neurovascular
between the radiation beam and the part being imaged, or
bundles and the flexor sheath.
any other object that is struck by radiation. The amount of
References: scatter is dependent on the characteristics of the object
1. Hall PN, Fitzgerald A, Sterne GD, Logan AM. Skin replacement in being imaged, the energy of the X-ray photons, and the dose
Dupuytren’s disease. J Hand Surg 1997; 22B:193-197. of radiation delivered. Radiation exposure decreases as the
square of the distance, so placing the patient’s extremity as
2. Hueston JT. The control of recurrent Dupuytren’s contracture by
skin replacement. Br J Pl Surg 1969; 22:152-156. far from the X-ray source as possible will reduce the amount
of radiation available to interact with the patient’s extremity,
3. McCash CR. The open palm technique in Dupuytren’s
and therefore reduce the scattered radiation. Placing the
contracture. Br J Pl Surg 1964; 17:271-280.
patient’s hand as far from the source as possible also
4. Roush TF, Stern PJ. Results following surgery for recurrent reduces the patient’s direct and scatter exposure. Keeping
Dupuytren’s disease. J Hand Surg 2000; 25A:291-296.
the surgical team’s hands out of the beam, whenever
possible, greatly reduces direct radiation hand exposure.

Other measures to decrease surgeon exposure include


using the mini C-arm machine, along with adjustment in
contrast and brightness for a dark image, rather than an
increase in kVp and mA. In addition, one should consider
using the mini C-arm in a horizontal orientation allowing
one to stand behind the image intensifier, thus lessening
radiation exposure. The use of radioprotective devices such
as lead aprons, thyroid collars and protective eyewear
can lessen exposure to the torso, thyroid and eyes, but
does not lessen exposure to the surgeon’s hands, and
therefore, other measures should be considered.

Reference:
Athwal GS, Bueno RA, Wolfe SW. Radiation exposure in hand
surgery: mini versus standard C-arm. J Hand Surg 2005;
30:1310-1316.

2009 Self-Assessment Examination | 57


115. A 6 year-old boy presents to the emergency room
following a fall from the monkey bars complaining
of severe elbow pain. Physical examination
reveals an isolated injury with a closed fracture of
the distal humerus, absent distal pulses, sluggish
capillary refill and definite coolness of the hand.
Radiographs demonstrate supracondylar distal
humerus fracture (Figure 1). The most appropriate
management includes:
A. Skeletal traction
B. Closed reduction percutaneous pin fixation
C. Open reduction internal fixation
D. Closed reduction percutaneous pin fixation and
forearm fasciotomies
E. Closed reduction percutaneous pin fixation with
intraoperative vascular reassessment

Preferred Response: E Question 115, Figure 1

Discussion: Limb ischemia in the setting of a markedly


displaced pediatric supracondylar fracture remains a true
orthopedic emergency. Loss of limb and/or loss of function 116. A 24 year-old male rugby player sustained
secondary to limb ischemia and associated compartment an unstable elbow dislocation with a Type 2
syndrome can result from the associated vascular injury. coronoid fracture. After anatomically stabilizing
The currently recommended treatment in this case includes the coronoid with a single screw and repairing
emergent operative reduction and pin fixation (closed, if the lateral collateral ligament, intraoperative
possible), followed by an assessment of the distal pulses. If evaluation demonstrates subluxation of the elbow
the limb remains pulseless, most recommend intraoperative within 60º of extension. The most appropriate
angiography. This fairly simple and rapid procedure may treatment is:
aid in the planning of the vascular procedure and may A. Cast immobilize the elbow at 90° for four weeks
prevent unnecessary exploration of the brachial artery in
B. Repair the medial facet of the coronoid with
cases of arterial spasm. If this is not feasible, a brachial
another screw
artery exploration is appropriate.
C. Steinmann pin fixation of the ulnohumeral joint for
References: six weeks
1. Copley LA, Dormans JP, Davidson RS. Vascular injuries and their D. Repair the anterior capsule
sequelae in pediatric supracondylar humeral fractures: toward a
goal of prevention. J Pediatr Orthop 1996; 16:99-103. E. Apply a hinged external fixator
2. Luria S, Sucar A, Eylon S, Pinchas-Mizrachi R, Berlatzky Y, Anner Preferred Response: E
H, Liebergall M, Porat S. Vascular complications of supracondylar
humeral fractures in children. J Pediatr Orthop 2007; 16B:133-143. Discussion: The elbow remains unstable following
surgical intervention. While some surgeons may
immobilize the ulnohumeral joint for a brief time by
either casting or pinning, prolonged immobilization is not
desirable for the traumatic elbow. Repair of the anterior
capsule is unnecessary following coronoid fixation. The
chances of having significant medial facet involvement
with a Type 2 coronoid fracture is low. A hinged external
fixator will maintain a concentric joint, protect the ligament
repair, and allow for early motion. Another acceptable
treatment would be to repair or reconstruct the medial
collateral ligament, a choice not offered .

58 | American Society for Surgery of the Hand


References: 118. A 54 year-old male is taken to surgery for
1. Morrey BF, O’Driscoll SW. Complex elbow instability. In: Morrey post-traumatic contracture release of his elbow
BF, ed. The Elbow and its Disorders. 3rd ed. Philadelphia: WB twelve months after a simple dislocation.
Saunders; 2000:423. Preoperatively, his range of motion was 45º of
2. McKee MD, Pugh DM, Wild LM, Schemitsch EH, King GJ. extension and 90º of flexion. Preoperative
Standard surgical protocol to treat elbow dislocations with radial radiographs showed no development of heterotopic
head and coronoid fractures. Surgical technique. J Bone Joint bone formation or joint degeneration. In addition to
Surg 2005; 87A Suppl 1(Pt 1):22-32. capsular release, which of the following would likely
improve the patient’s outcome?

117. A 32 year-old male sustained an injury to his right A. Release of one or both collateral ligaments
long finger playing flag football. X-ray revealed B. Ulnar nerve decompression
a volar dislocation of the PIP joint. Following a C. Application of hinged external fixator
concentric closed reduction, the most appropriate
D. Fenestrate the distal humerus
management is:
E. Immobilize in full extension for two weeks
A. Immediate active motion
B. Guided passive motion Preferred Response: B

C. Extension block splinting Discussion: Long-standing contractures of the elbow


D. Static PIP extension splinting have repercussions on extra-articular structures, such
as tendons and nerves. Prolonged lack of excursion can
E. Dynamic external fixation
cause the ulnar nerve, for example, to become encased
Preferred Response: D in fibrosis. After contracture release, the motion of
the newly freed joint pulls on the tethered ulnar nerve,
Discussion: The primary difference between volar which causes pain and paresthesias. Patients without
dislocations and volar rotary dislocations of the PIP ulnar nerve symptoms prior to contracture release often
joint is that volar dislocations are often associated with develop them postoperatively. The pain caused by
central slip avulsion injuries. In volar rotary dislocations, ulnar nerve tethering has been identified as a significant
the condyle of the proximal phalanx penetrates between hindrance for patients attempting to maintain motion after
the central slip and lateral band, leaving both structures contracture release. It is recommended to preemptively
intact. After reduction, the joint is usually stable and early decompress or transpose the ulnar nerve at the same
motion is allowed. Volar dislocations, however, cannot time as the contracture release, even if the release is
be mobilized and the extensor tendon must be either performed arthroscopically. In the absence of hetertopic
repaired or statically splinted. Extension block splinting bone formation, the collateral ligaments should be
and dynamic external fixation are indicated for fracture/ preserved, maintaining joint stability and negating the
dislocations of the PIP. need for a hinged external fixator. While deepening of the
References: coronoid and/or olecranon fossa may be helpful in some
cases, fenestrating the distal humerus is not indicated for
1. Schernberg F, Elzein F, Gillier P, Gerard Y. Dislocations of the
proximal interphalangeal joints of the long fingers. Anatomo- this patient. Immobilization following contracture release
clinical study and therapeutic results. Ann Chir Main 1982; is contraindicated.
1(1):18-28.
References:
2. Vicar AJ. Proximal interphalangeal joint dislocations without
1. Martin BG, Johansen JA, Edwards SG. Complications related to
fractures: Hand Clin 1988 Feb; 4(1):5-13.
simple dislocations of the elbow. Hand Clin 2007; 24(1):9-25.

2. Lindenhovius AL, Jupiter JB. The posttraumatic stiff elbow: a


review of the literature. J Hand Surg 2007; 32A(10):1605-23.

2009 Self-Assessment Examination | 59


119. A 37 year-old female substance abuser inadvertently Discussion: This patient has gout, which can mimic
injected an undisclosed substance into her brachial septic arthritis. The diagnosis often depends on joint
artery. She presents with painful digital motion and aspiration. Synovial aspirations with WBC counts of
swelling. Arteriogram reveals multiple occlusions of greater than 80,000 are indicative of bacterial septic
the small vessels to the index, long, ring, and small arthritis, with the exception of gonococcal infections
fingers. The most appropriate initial treatment is: which WBC counts may be around 50,000-70,000.
Typically, WBC counts less than 50,000 are indicative
A. Heparin
of non-infectious etiologies, such as gout, RA, and
B. Thrombolytic therapy Reiter’s syndrome. Gout is commonly seen in the setting
C. Digital fasciotomy of renal insufficiency and treatment options should be
D. Superficial arch reconstruction modified because of this. Colchicine and nonsteroidal
anti-inflammatory medications are reasonable treatments
E Amputation
for acute gouty attacks, but are relatively contraindicated
Preferred Response: B for this patient because of his renal insufficiency. Steroid
injections can be very effective in relieving symptoms
Discussion: Embolic phenomena can lead to many caused by gouty arthritis.
different injuries requiring different interventions. Once
the terminal small vessels become involved, vascular References:
reconstruction is not possible. Isolated involvement 1. Freeland A, Senter B. Septic arthritis and osteomyelitis.
requires supportive therapies such as anticoagulation, Hand Clin 1989; 5:533-552.
vasodilation, and close observation. Diffuse involvement, 2. Werlen D, Gabay C, Vischer TL. Costeroid therapy for the
however, requires more aggressive intervention because treatment of acute attacks of crystal-induced arthritis: an
of the heightened risk of multiple amputations. In this effective alternative to nonsteroidal antiinflammatory drugs.
case, thrombolytic agents, such as urokinase, are Rev Rhum Engl Ed. 1996; 63(4):248-54.
recommended. Although compartment syndrome is a
complication of embolic disease, there is no evidence of
a compartment syndrome here, in this particular case, 121. Which nerve provides sensory innervation to the
fasciotomies are not indicated. dorsal radiocarpal ligament?
A. Dorsal sensory branch of the radial
References:
1. Koman LA, Ruch DS, Smith BP, Smith TL. Vascular Disorders.
B. Posterior interosseous branch of the radial
In: Green DP, Hotchkiss RN, Pederson WC, Wolfe SW eds. C. Palmar cutaneous branch of the median
Green’s Operative Hand Surgery. 5th ed. Philadelphia: Churchill
Livingston/Elsevier, 2005:2290-2291.
D. Dorsal sensory branch of the ulnar

2. Jones NF. Acute and chronic ischemia of the hand:


E. Motor branch of the ulnar
pathophysiology, treatment, and prognosis. J Hand Surg 1991;
Preferred Response: B
16A(6):1074-83.
Discussion: A rich sensory innervation is provided by
the posterior interosseous nerve to the dorsal radiocarpal
120. A 65 year-old male with a history of renal ligament. These nerve endings are involved in the
insufficiency and an upper respiratory infection proprioceptive and pain modulating function of the
presents with an acutely swollen, painful right wrist. dorsal capsule.
Aspiration of the wrist demonstrates a synovial
WBC count of 20,000 and negatively birefringent References:
crystals. The most appropriate treatment involves: 1. Lin YT, Berger RA, Tomita K, et al. Nerve endings of the wrist
joint: a preliminary report of the dorsal radiocarpal ligament.
A. Surgical incision and drainage J Orthop Res 2006; 24:1225-1230.
B. Nonsteroidal anti-inflammatory medication 2. Tolo ET, Shin AY. Fracture dislocations of the carpus. In: Trumble
C. Colchicine TE, Budoff JE eds. Hand Surgery Update IV. American Society for
Surgery of the Hand, Rosemont: 2007:271-292.
D. Steroid injection
E. Allopurinol

Preferred Response: D

60 | American Society for Surgery of the Hand


122. Traumatic disruption of the radial collateral Type II swan-neck deformity develops when synovitis
ligament of the thumb MP joint is prone to a of the MP joint attenuates the sagittal bands leading
pathologic sequence of joint instability with to MP subluxation and intrinsic tightness. For Type II
progressive ulnar and volar subluxation. This is deformities, intrinsic release must also be performed.
due to the unopposed force of which muscle?
Type III and IV deformities both have limited PIP motion.
A. Abductor pollicis longus In Type III, motion is limited by the extensor mechanism,
B. Abductor pollicis brevis collateral ligaments and contracted skin. Type IV has
C. Adductor pollicis severe intraarticular PIP joint destruction. Type III is
treated by lateral band mobilization, while Type IV is
D. Extensor pollicis brevis treated by PIP fusion or arthroplasty.
E. Opponens pollicis
References:
Preferred Response: C 1. Dodds SD, Brugel-Sanchez, Swigert CR. Rheumatoid arthritis of
the hand: soft tissue reconstruction. In: Trumble TE, Budoff JE
Discussion: The radial collateral ligament (RCL) is a eds. Hand Surgery Update IV. American Society for Surgery of
critical lateral stabilizer of the thumb MP joint. The the Hand, Rosemont: 2007:673-685.
intrinsic anatomy of the RCL is not a mirror image of
2. Kiefhaber TR, Strickland JW. Soft tissue reconstruction for
the ulnar side. Disruption of the RCL, coupled with the rheumatoid swan-neck and boutonnierre deformities: long-term
unopposed dynamic force of the adductor pollicis, leads results. J Hand Surg 1993; 18A:984-989.
to the pathologic sequence of joint instability, ulnar and
volar subluxation and degenerative joint disease.
124. Following a severe crush injury to the hand, the
References:
characteristic position assumed by the swollen
1. Culp R, Ratner J. Acute thumb injuries. In: Trumble TE, Budoff JE
hand consists of:
eds. Hand Surgery Update IV. American Society for Surgery of
the Hand, Rosemont: 2007:51-61. A. MP joint flexion
2. Melone CP, Beldner S, et al. Thumb collateral ligament injuries: B. PIP joint extension
an anatomic basis for treatment. Hand Clin 2000; 16:345-357.
C. DIP joint extension
D. Thumb adduction
123. In rheumatoid arthritis swan-neck deformity with E. Wrist extension
no intrinsic tightness, PIP joint imbalance leads to
tightening of which structure? Preferred Response: D

A. Volar capsule of the PIP joint Discussion: Edema results from any insult to the hand.
Accumulation of fluid or hematoma within the capsular
B. Lumbrical muscle
structures of the joint or within the synovial space will
C. Flexor digitorum superficialis impair joint function and subsequently promote joint
D. Triangular ligament stiffness and contracture. The joints assume the positions
E. Transverse retinacular ligament of maximum fluid capacity. This results in the intrinsic
minus hand, characterized by interphalangeal joint
Preferred Response: D flexion, metacarpophalangeal joint extension, thumb
adduction and wrist flexion. These changes can become
Discussion: Type I swan-neck deformity has full flexibility
fixed resulting in joint contractures.
of the PIP joint. As PIP hyperextension develops, the
lateral bands displace dorsally and the transverse References:
retinacular ligaments stretch. The dorsal triangular 1. Ghidella SD, Segalman KA, et al. Long-term results of surgical
ligament tightens while the volar capsule attenuates. management of proximal interphalangeal joint contracture.
With less tension on the lateral bands, the DIP joint flexes J Hand Surg 2007; 27A:799-805.
in a mallet deformity. 2. Watson HK, Weinzweig J. Stiff joints. In: Green DP, Hotchkiss
RN, Pederson WC eds. Green’s Operative Hand Surgery 4th ed.
Philadelphia: Churchill Livingston, 1999:552-562.

2009 Self-Assessment Examination | 61


125. The etiology of a swan-neck deformity in a 126. Which artery serves as the pedicle of a latissimus
rheumatoid arthritis patient with a positive Bunnell dorsi flap?
intrinsic tightness test and full active PIP flexion is:
A. Circumflex scapular
A. DIP joint mallet deformity with arthritis B. Thoracodorsal
B. DIP joint mallet deformity without arthritis C. Lateral thoracic
C. PIP joint stiffness with lateral band contracture D. Thoracoacromial
D. PIP joint arthritis with joint destruction E. Posterior intercostal
E. MP joint synovitis and subluxation
Preferred Response: B
Preferred Response: E
Discussion: The subscapular artery is a branch off the
Discussion: Type II swan-neck deformity develops from axillary artery. It branches into the thoracodorsal artery
MP joint synovitis and subluxation. The sagittal bands and the circumflex scapular artery. The thoracodorsal
are attenuated, thereby weakening the extensor force. artery supplies the latissimus dorsi muscle. The
Subsequent intrinsic contracture occurs. It is not enough to circumflex scapular artery supplies the scapular flap. The
perform procedures that simply limit PIP hyperextension. An thoracodorsal artery also divides within the latissimus dorsi
intrinsic release, either alone or combined with MP joint muscle into an upper transverse (medial) branch and a
arthroplasty, must be performed as well. lower descending (lateral) branch. The latissimus dorsi
muscle flap can be pedicled on the posterior intercostal
References: arteries, but not for coverage on the upper extremity.
1. Dodds SD, Brugel-Sanchez V, Swigert CR. Rheumatoid arthritis
of the hand: soft tissue reconstruction. In: Trumble TE, Budoff JE References:
eds. Hand Surgery Update IV. American Society for Surgery of 1. Jones NF, Lister GD. Free skin and composite flaps. In: Green DP,
the Hand, Rosemont: 2007:673-685. Hotchkiss RN, Pederson WC eds. Green’s Operative Hand Surgery.
2. Kiefhaber TR, Strickland JW. Soft tissue reconstruction for 4th ed. Philadelphia: Churchill Livingston, 1999:1159-1200.
rheumatoid swan-neck and boutonnierre deformities: long-term 2. Mathes SJ, Nahai F. Clinical atlas of muscle and myocutaneous
results. J Hand Surg 1993; 18A:984-989. flaps. St. Louis: Mosby, 1979:369-391.

Question 126, Figure 1


This article was published in Green’s Operative Hand Surgery, 5th
edition, Volume 2, Pederson, W.C. and Lister, L., Skin Grafts and
Skin Flaps (Skin Flaps), Page 1690, Copyright Elsevier (2005).

62 | American Society for Surgery of the Hand


127. Electrodiagnostic studies are performed in a 128. A child is born with radial deficiency. Evaluation
patient with carpal tunnel syndrome. Abnormal for associated systemic conditions includes a
antidromal peak sensory latency measured at chromosomal challenge test for early diagnosis of:
14 cm is greater than how many milliseconds?
A. Holt-Oram syndrome
A. 2 B. TAR syndrome
B. 2.5 C. VACTERL syndrome
C. 3 D. Fanconi anemia
D. 3.5 E. Hunter-Thompson syndrome
E. 4
Preferred Response: D
Preferred Response: D
Discussion: Holt-Oram, TAR, VACTERL and Fanconi
Discussion: Nerve conduction studies are frequently used anemia are syndromes associated with radial deficiency.
electrodiagnostic tests for carpal tunnel syndrome. A sensory All forms of radial deficiency warrant systemic evaluation
specific recording is called a sensory nerve action potential irrespective of the degree of expression. The appropriate
(SNAP). The nerve can be stimulated in an antidromic (from work-up usually necessitates referral to pediatric
proximal to distal) or orthodromic (from distal to proximal) subspecialists. Children with Fanconi anemia do not
fashion. The speed of conduction is similar in either direction. have signs of bone marrow failure at birth. The majority
The distance (in millimeters) and time (in milliseconds) of of children experience signs of aplastic anemia between
action potential travel between electrodes must be known in the ages of 3 and 12 years (median age of 7 years). A
order to measure the speed of conduction. Action potential chromosomal challenge test is currently available that
velocity (distance/time), latency (time between the stimulus allows the detection of the disease prior to the onset of
and the onset of the action potential), and amplitude are bone marrow failure. Since bone marrow transplant is the
typically recorded. The question involves antidromic sensory only cure for Fanconi anemia, this preliminary diagnosis is
nerve latency across a 14 cm measured segment. Values crucial for the child and family.
above 3.5 milliseconds (ms) are considered abnormal.
Abnormal values of orthodromic sensory nerve latency References:
measured at a standard 8 cm distance across the wrist 1. Kozin SH. Upper extremity congenital anomalies: current
concepts review. J Bone Joint Surg 2003; 85:1564-1575.
are typically greater than 2.2 ms.
2. McCarroll HR. Congenital anomalies: a 25-year review. J Hand
References: Surg 2000; 25A:1007-1037.
1. Means KR, Segalman KA. Carpal tunnel syndrome. In: Trumble
TE, Budoff JE eds. Hand Surgery Update IV. American Society for
Surgery of the Hand, Rosemont: 2007:399-407.
2. Townshend DN, Taylor PK, Gwynne-Jones DP. The outcome of
carpal tunnel decompression in elderly patients. J Hand Surg
2005; 30A:500-505.

3. Slutsky DJ. Nerve conduction studies in hand surgery. J Hand


Surg 2003; 3(3):152-169.

2009 Self-Assessment Examination | 63


129. The recommended treatment for improving severe Discussion: In a patient with a strong history of complex
finger flexion contractures in a stroke patient with regional pain syndrome (CRPS), generally, surgery should
palmar skin maceration is: be avoided due to the risk of triggering another episode
of CRPS.
A. Occupational and/or physical therapy
B. Antispasticity medication Symptomatic prominent hardware should not be ignored.
C. Flexor-pronator slide A meta-analysis of the literature found a 7.8% incidence
of tendon rupture and tenosynovitis, and a 16% rate of
D. Functional electrical stimulation plate removal in 603 patients, 53% of whom were treated
E. Superficialis to profundus tendon (STP) transfer with a volar plate.
Preferred Response: E If surgery is indicated in these individuals, the administration
of a perioperative stellate ganglion block has been found
Discussion: The superficialis to profundus (STP) procedure
to significantly reduce the recurrence of CRPS.
is used primarily for addressing finger-in-palm deformities
to improve hygiene. It is not a good procedure for improving References:
motor function of the digits. It is not the procedure of 1. Reuben SS, Rosenthal EA, Steinberg RB: Surgery on the affected
choice in patients with good hand/finger function. upper extremity of patients with a history of complex regional
pain syndrome: a retrospective study of 100 patients. J Hand
References: Surg 2000; 25A(6):1147-51.
1. Hier DB, Mondlock J, Caplan LR. Recovery of behavioral
2. Margaliot Z, et al. A meta-analysis of outcomes of external
abnormalities after right hemisphere stroke. Neurology 1983;
fixation versus plate osteosynthesis for unstable distal radius
33345-350.
fractures. J Hand Surg 2005; 30A:1185-1199.
2. Tafti MA, Cramer SC, Gupta R. Orthopaedic management of
upper extremity stroke patients. J Am Acad Orthop Surg 2008;
16:462-470.
131. What is the most likely diagnosis of the lesion
seen in the 34 year-old female in Figures 1 and 2?

130. A 35 year-old female presents with complaints of A. Synovial cell sarcoma


dorsal wrist pain associated with finger flexion B. Giant cell tumor of bone
two years after undergoing an ORIF of a distal C. Osteosarcoma
radius fracture. She was diagnosed with Complex
D. Enchondroma
Regional Pain Syndrome (CRPS) after her surgery
and required extensive hand therapy to resolve E. Metastatic lesion
her symptoms. Radiographs reveal her fracture
Preferred Response: B
to be well united with prominent hardware. Your
recommendation for this patient is: Discussion: Giant cell tumor (GCT) of bone affects
A. No operation is indicated, due to her CRPS history skeletally mature individuals, F > M, in the 20 to 50 age
group. This tumor is rare in children and patients older
B. Proceed with removal of the hardware than 60 years of age. GCT’s are locally very aggressive
C. Denervation procedure, leaving the hardware intact and as a result can cause considerable destruction. They
D. Hardware removal and stellate ganglion block are relatively common, accounting for 4% of all primary
bone tumors. Multifocal lesions are usually Brown tumors
E. Refer to a pain specialist for chronic pain management
of hyperparathyroidism. Brown tumors may be confused
Preferred Response: D with GCT’s. The histogenesis of GCT’s is unclear with
characteristics of macrophages and osteoclastic cells.

References:
1. O’Donnell RJ, Springfield DS, Motwani HK, et al. Recurrence of
giant cell tumors of the long bones after curettage and packing
with cement. J Bone Joint Surg 1994; 76A:1827.

2. Ferracini R, Masterson EL, Bell RS, Wunder JS. Distal ulnar


tumours. Results of management by en bloc resection in
nine patients and review of the literature. J Hand Surg 1998;
23B(4):517-21.

64 | American Society for Surgery of the Hand


Discussion: Drawing of a lateral radiograph of the wrist
showing the anatomic landmarks used in the triangulation
method. The normal DL:PL ratio is 0.5 to 1.0.

The distal landmark is located at the junction of the


dorsal and middle thirds of the (dorsopalmar) width of the
capitate at the level of the third carpometacarpal joint.

The scapholunate angle is the traditional method


used for assessing VISI and DISI deformities of the
wrist. Other angulatory measurements such as the
radiolunate, capitolunate, and rarely the lunotriquetral,
are sometimes used.

In a comparative study involving the various radiological


methods available, the triangulation method was found
to have the best inter/intraobserver correlation. In part, the
difficulty with many of these angulatory measurements is that
Question 131, Figure 1 there are large standard deviations with these values which
reduces their clinical specificity. Additionally, the angulatory
measurements are more susceptible to positional problems
compared to radiological ratios. The method is performed
by drawing a point between the dorsal and middle third of
the capitate at the level of the CMC joint on a true lateral
x-ray. Additionally, points on the dorsal and palmar poles
of the lunate are also drawn. Connecting the three dots
forms a triangle with a dorsal limb (DL) and a palmar limb
(PL). A DL:PL ratio greater than 1.0 indicates a DISI deformity
and a DL:PL ratio less than 0.5 is a VISI deformity. If there
is a borderline value, then the radiometacarpal (RM) angle
is measured and the value is checked on a “normagram”
to assess for carpal instability. Hence, the triangulation
ratio was found to have a better agreement among
examiners than the other radiographic methods.

References:
1. Pirela-Cruz MA, Hansen MF. Assessment of midcarpal deformity
of the wrist using the triangulation method. J Hand Surg 2003;
28A(6):938-42.
Question 131, Figure 2
2. Cho MS, Battista V, Dubin NH, et al. Assessment of four
midcarpal radiologic determinations. Surg Radiol Anat. 2006;
28(1):92-7. Epub 2005 Dec 9.
132. Which of the following carpal measurements
has the best inter and intra observer correlation
for determining VISI and DISI deformities of
the wrist?
A. Radiolunate angle
B. Scapholunate angle
C. Lunotriquetral angle
D. Triangulation ratio
E. Capitolunate angle

Preferred Response: D

Question 132, Figure 1


2009 Self-Assessment Examination | 65
133. A 27 year-old male suffers a transverse laceration 134. Which of the following best describes the primary
to the volar wrist. He has no sensation in the finger action of bone morphogenic protein 2 (BMP2) in
pads and no active finger flexion. At surgery, a fracture healing?
transected 3 mm diameter artery is encountered
A. Chemotactic effect on inflammatory cells,
within the carpal tunnel. This finding is associated stimulation of extracellular matrix synthesis,
with what anomaly: angiogenesis, recruitment of endogenous fibrogenic
A. Duplication of the ulnar nerve cells to the injury site, and bone resorption
B. Failure of formation of the flexor pollicis longues B. Degradation of the cartilage and bone allowing
the invasion of blood vessels during endochondral
C. Heart defects, most commonly atrial septal defects
ossification and bone remodeling
D. High bifurcation of the median nerve
C. Differentiation of undifferentiated mesenchymal
E. A single nerve incorporating the function of both the cells into chondrocytes and osteoblasts, and
median and radial nerves osteoprogenitors into osteoblasts
Preferred Response: D D. Decreased osteoclast activity through up regulation
of receptor activator nuclear factor-kappaB ligand
Discussion: The presence of a persistent median artery (RANKL) mRNA in primary bone marrow stromal cells
is associated with an increased incidence of a high
E. Up regulation of noggin, gremlin, sclerostin, and
bifurcation of the median nerve, with the artery lying
chordin mRNA
between. In one ultrasound study, a persistent median
artery was found in 16% of wrists in asymptomatic Preferred Response: C
volunteers, and the persistent median artery was
associated with a high median nerve division or bifed Discussion: One of the main actions of BMP2 is that it
median nerve in 63%. Much lower prevalence has been is believed to stimulate the proliferation and differentiation
reported, including a 0.6% prevalence of a persistent of osteoblasts, increasing the number of osteoblasts
median artery in Korean surgical patients. by helping undifferentiated mesenchymal cells become
chondrocytes and osteoblasts, and osteoprogenitors
When exploring a volar wrist laceration, the finding of into osteoblasts.
either a smaller than expected median nerve, particularly
a very radial or very ulnar median nerve, or the presence Cytokines (IL-1, IL-6, TNF-a) act through a chemotactic
of a persistent median artery, may suggest that there may effect on other inflammatory cells, stimulation of
be a second branch of the median nerve to repair. extracellular matrix synthesis, angiogenesis, recruitment
of endogenous fibrogenic cells to the injury site, and
References: at later stages bone resorption. Metaloproteasis act
1. Gassner EM, Schocke M, Peer S, Schwabegger A, Jaschke W, by degradation of the cartilage and bone allowing
Bodner G. Persistent median artery in the carpal tunnel: color the invasion of blood vessels during the final stages
Doppler ultrasonographic findings. J Ultrasound Med 2002 Apr; of endochondral ossification and bone remodeling.
21(4):455-61.
Parathryroid hormone upregulates receptor activator
2. Ahn DS, Yoon ES, Koo SH, Park SH. A prospective study of the nuclear factor-kappaB ligand (RANKL) mRNA, increasing
anatomic variations of the median nerve in the carpal tunnel in osteoclast activity. Noggin, gremlin, sclerostin, and
Asians. Ann Plast Surg 2000 Mar; 44(3):282-7.
chordin are BMP antagonists.

References:
1. Dimitriou R, Tsiridis E, Giannoudis PV. Current concepts of molecular
aspects of bone healing. Injury. 2005 Dec; 36(12):1392-404.

2. Huang JC, Sakata T, Pfleger LL, Bencsik M, Halloran BP, Bikle


DD, Nissenson RA. PTH differentially regulates expression of
RANKL and OPG. J Bone Miner Res 2004; 19(2):235-44. Epub
2003 Dec 16.

66 | American Society for Surgery of the Hand


135. Which of the following measures has the highest 136. A 9 year-old female fell down stairs approximately
level literature evidence for prophylaxing against 3 feet, injuring her right elbow 3 days ago and
CRPS in elderly females with distal radius fractures? presents with elbow pain. Radiographs are
shown (Figures 1 and 2). The most appropriate
A. Axillary block
management is:
B. Steroid/Bretylium Bier block
A. Sling until comfort improves, then early mobilization
C. Pregabalin
B. Immobilization in splint (then cast) for 4 weeks
D. Stellate ganglion blocks
C. Closed manipulation and immobilization for 4 weeks
E. Vitamin C
D. Closed reduction and percutaneous pinning
Preferred Response: E E. Open reduction and internal fixation
Discussion: Although from the same author, two reports Preferred Response: E
of a randomized trial from the Netherlands showed a
deceased incidence of complex regional pain syndrome Discussion: The X-rays show a shear fracture of the
in patients with distal radius fractures given Vitamin C. capitellum in a child, which is a relatively rare injury in
In the most recent study, the prevalence of complex this age group, in contrast to a lateral condyle fracture.
regional pain syndrome was 2.4% (eight of 328) in the Older literature, with limited follow up, older patients,
Vitamin C group and 10.1% (10 of 99) in the placebo and limited fixation options, suggested excision of
group (p = 0.002). Patients who received at least 500 mg the fragment as an option. Anatomic closed reduction
per day for 50 days showed the lowest incidence. The is unlikely to be achievable on a routine basis. Open
patients with CRPS were elderly females, and there was reduction and internal fixation restores the anatomy, and
an association between development of CRPS and early allows primary bone healing.
cast complaints. The reduction of CPRS was also found
Reference:
in a sequential cohort study in the French literature.
De Boeck H, Pouliart N. Fractures of the capitellum humeri in
References: adolescents. Int Orthop. 2000; 24(5):246.
1. Zollinger PE, Tuinebreijer WE, Breederveld RS, Kreis RW. Can
vitamin C prevent complex regional pain syndrome in patients
with wrist fractures? A randomized controlled, multicenter dose-
response study. J Bone Joint Surg 2007; 89A(7):1424-31.

2. Cazeneuve JF, Leborgne JM, Kermad K, Hassan Y. Vitamin C


and prevention of reflex sympathetic dystrophy following surgical
management of distal radius fractures. Acta Orthop Belg. 2002;
68(5):481-4.

3. Zollinger PE, Tuinebreijer WE, Kreis RW, Breederveld RS. Effect


of vitamin C on frequency of reflex sympathetic dystrophy in wrist
fractures: a randomised trial. Lancet 1999; 354:9159.

Question 136, Figure 1

Question 136, Figure 2

2009 Self-Assessment Examination | 67


137. A 50 year-old previously healthy male presents References:
with bilateral upper extremity weakness. 1. Jaeger H, Maier C. Calcitonin in phantom limb pain: a double-
Physical exam reveals wasting of both thenar blind study. Pain. 1992; 48(1):21-7.
and hypothenar intrinsic muscles and tongue 2. Eichenberger U, Neff F, Sveticic G, Björgo S, Petersen-Felix S,
fasciculations. The most likely diagnosis is: Arendt-Nielsen L, Curatolo M. Chronic phantom limb pain: the
effects of calcitonin, ketamine, and their combination on pain and
A. Amyotrophic lateral sclerosis
sensory thresholds. Anesth Analg. 2008; 106(4):1265-73.
B. Multiple nerve compression syndrome
C. Multiple sclerosis
139. Which of the following is the most common cause
D. Spinal cord tumor
of revision amputations in children?
E. Syrinx
A. Contractures
Preferred Response: A
B. Deep venous thrombosis
Discussion: Amyotrophic lateral sclerosis (ALS) is a C. Phantom limb pain
disorder of the anterior horn cells that leads to progressive D. Bony overgrowth
muscle weakness and atrophy. Sensory changes are
E. Skin breakdown
not seen or are minimal. Cranial motor nerves can be
involved, and can manifest the disease with visible Preferred Response: D
facilitations in the tongue (“bag of worms”), although
this is not pathognomonic for ALS. The combination of Discussion: Bony overgrowth requiring revision does
muscle weakness and wasting in the distribution of more occur in children, particularly in younger children. Joint
than one peripheral nerve, sensory sparing, and visible disarticulations do not undergo overgrowth.
fasciculations of the tongue, suggest the possibility of ALS.
In the absence of other factors, joint contractures
Reference: in children rarely occur. Deep venous thrombosis is
Goetz TJ. Textbook of Clinical Neurology, 3rd ed. Philadelphia: extraordinarily rare in children. The incidence of phantom
Saunders/ Elsevier, 2007. limb pain in upper extremity amputations would be
expected to depend on both the criteria used to define
phantom limb pain, as well as the aggressiveness
138. Which of the following measures has the highest which the symptoms are elicited, particularly from a
level literature evidence for reducing acute suggestible population. The incidence of phantom limb
phantom limb pain? pain after amputations has been reported to be high.
Ebrahimzadeh et al. studied young adults with upper
A. Calcitonin
extremity amputations secondary to war injuries and
B. Clonazepam found phantom limb pain in 32% and phantom sensation
C. Metoprolol in 64%. Krane and Heller, in pediatric lower extremity
D. Pregabalin amputees, reported an incidence of phantom pain of
83-92%, with the symptoms documented in the formal
E. Steroid/Bretylium Bier block medical record in only 40% of those identified as having
Preferred Response: A phantom limb pain. Wilkins et al. included both lower
and upper extremity pediatric amputees, although more
Discussion: Although all of the medications above have of the amputations were upper extremity amputations.
been suggested for neuropathic pain, the literature is They found that 3.7% had phantom limb pain in their
anecdotal for all of the options except calcitonin. A cross congenital group, but in children with amputations that
over study by Jaeger and Maier showed a reduction of were not the result of congenital differences, 48.5%
the visual analog pain score in 21 patients with acute had phantom limb pain. According to Thomas et al.,
phantom limb pain treated with Calcitonin. However, in a phantom limb pain in pediatric amputations secondary
recent randomized crossover trial by Eichenberger et al., to burns was found in 0/4 upper limb amputations
no benefit was found for calcitonin for the treatment of secondary to thermal flame burns, but in 5/10 with upper
chronic phantom limb pain, leaving only the first trial and limb amputations secondary to electrical burns. But,
anecdotal reports to support the use of calcitonin. regardless of the frequency of phantom limb pain, revision
amputation is rarely performed for phantom limb pain.

68 | American Society for Surgery of the Hand


References: 141. A 3 year-old male suffered a right index finger
1. O’Neal ML, Bahner R, Ganey TM, Ogden JA. Osseous Overgrowth crush in a car door one week ago and has a
After Amputation in Adolescents and Children. Journal of Pediatric swollen stiff proximal interphalangeal joint.
Orthopaedics 1996; 16(1):78-84. Radiographs are shown. The best option for
2. Ebrahimzadeh MH, Fattahi AS, Nejad AB. Long-term follow-up of management is:
Iranian veteran upper extremity amputees from the Iran-Iraq war
A. Immobilization
(1980-1988). J Trauma 2006; 61(4):886-8.
B. Closed reduction and immobilization
3. Krane EJ, Heller LB. The prevalence of phantom sensation and pain
in pediatric amputees. J Pain Symptom Manage 1995; 10:21–29. C. Closed reduction and pinning
4. Wilkins KL, McGrath PJ, Finley JA. Katz J. Phantom limb sensations D. Open reduction and pinning
and phantom limb pain in child and adolescent amputees. Pain
E. Volar plate arthroplasty
1998; 78:7–12.
5. Thomas CR, Brazeal BA, Rosenberg L, Robert RS, Blakeney PE, Preferred Response: D
Meyer WJ. Phantom limb pain in pediatric burn survivors. Burns
2003; 29(2):139-142. Discussion: This fracture-dislocation involves the middle
phalanx base epiphysis, with the epiphysis flipped within
the joint. Limited information regarding non-operative
140. In considering the advantages and disadvantages treatment of this injury is available. In a case treated
of transhumeral amputation versus elbow non-operatively initially, and then treated with operative
disarticulation, a transhumeral amputation: excision of the epiphysis eight years later, subsequent
range of motion was limited. The rest of the reported
A. Facilitates suspension of the prosthetic limb cases were all surgically-reduced, and sometimes,
B. Allows use of standard internal hinge elbow joints supplemented with K-wires. The fractures were explored
C. Improves rotational stability of the stump from dorsal, except for one that was explored both
dorsally and volarly to achieve reduction.
D. Avoids limb length discrepancies and overgrowth
in children References:
E. Minimizes redundant tissue between prosthesis 1. Hashizume H, Nishida K, Mizumoto D, Takagoshi H, Inoue H.
and stump Dorsally displaced epiphyseal fracture of the phalangeal base.
J Hand Surg 1996; 21B(1):136-8.
Preferred Response: B
2. Keene JS, Engber WD, Stromberg WB Jr. An irreducible
Discussion: An elbow disarticulation preserves the phalangeal epiphyseal fracture-dislocation. A case report.
Clin Orthop Relat Res 1984; (186):212-5.
condyles, which provide a flare on which to suspend
the prosthesis, as well as an improved rotational
moment arm, minimizing rotation of the stump within the
prosthetic socket. A trans-humeral amputation allows the
use of an internal elbow hinge that lies in the anatomic
position of the original elbow joint and matches brachial
length, avoiding a more bulky external hinge. An elbow
disarticulation avoids overgrowth in children, but has
limited tissue coverage, which has been anecdotally
reported in both upper and lower extremities to lead to
breakdown and subsequent revision surgery.

References:
1. Jebson P, Louis D. Amputations. In: Green DP, Hotchkiss RN,
Pederson WC, Wolfe SW eds. Green’s Operative Hand Surgery, 5th
ed. Philadelphia: Churchill Livingston/Elsevier, 2005:1939-1983.

2. de Luccia N, Marino HL. Fitting of electronic elbow on an elbow


disarticulated patient by means of a new surgical technique. Prosthet
Orthot Int. 2000; 24(3):247-51. Question 141, Figure 1 Question 141, Figure 2
3. O’Neal ML, Bahner R, Ganey TM, Ogden JA. Osseous overgrowth
after amputation in adolescents and children. Journal of Pediatric
Orthopaedics 1996; 16(1):78-84.

2009 Self-Assessment Examination | 69


142. A 70 year-old woman in Figure 1 presents with a
humeral nonunion, with the original fracture being
treated by open reduction and internal fixation
with a compression plate 2 years ago. At surgery
for repair of the nonunion, the plate is found to be
placed with the upside-down. Postoperatively, the
issue of the plate being placed with the upside-
down is most appropriately addressed with the
patient as follows:
A. Document the information of the plate placement
thoroughly in the dictated operative note, but do not
specifically volunteer that the plate was upside-down
unless asked Question 142, Figure 1
B. Inform the patient that an upside-down plate does
not function effectively as a compression plate and
that this was a major contributor to the nonunion 143. You are performing surgery for an injury that was
C. Recommend to the patient that they pursue reported to have occurred at work and is therefore
malpractice litigation against the previous surgeon being paid for by workers’ compensation.
Before going back to the operating room, the
D. Tell the patient that the plate was placed upside-down
anesthesiologist reports that the patient, after
E. Wait to make sure there are no complications from receiving preoperative sedation, said that the
your surgery and for the humerus to heal before injury actually occurred at home. You should:
bringing up the issue of whether plate placement
caused the nonunion A. Cancel the surgery
B. Carefully document the conversation with
Preferred Response: D
the anesthesiologist in the chart
Discussion: “It is a fundamental ethical requirement C. Inform the insurance company
that a physician should at all times deal honestly and D. Disregard what the patient said after
openly with patients. Patients have a right to know their receiving medication
past and present medical status and to be free of any
E. Question the patient further prior to beginning
mistaken beliefs concerning their conditions. Situations
the procedure
occasionally occur in which a patient suffers significant
medical complications that may have resulted from the Preferred Response: D
physician’s mistake or judgment. In these situations, the
physician is ethically required to inform the patient of all Discussion: There are multiple conflicting needs in this
the facts necessary to ensure understanding of what has scenario. The needs of the patient, the physician, the
occurred. Only through full disclosure is a patient able to payer, and society are not aligned. Even if the problem
make informed decisions regarding future medical care.” is evaluated from only the perspective of the physician,
there is still conflict between the differing factors, including
“Ethical responsibility includes informing patients of responsibility to the patient, responsibility to society, one’s
changes in their diagnoses resulting from retrospective own moral code, responsibilities toward co-workers, and
review of test results or any other information. This external reputation and one’s self-perception. However,
obligation holds even though the patient’s medical in this scenario, one factor has primacy. After sedation,
treatment or therapeutic options may not be altered by ethically and legally the patient is not considered
the new information.” competent. Therefore, as a general principle, anything said
after the patient receives sedation should be disregarded,
Reference:
unless the statement requires a medical response—such
AMA code of medical ethics, opinion 8.12.
as a patient complaining of shortness or breath and chest
pain. Additionally, as stated in the AMA code of ethics,
“A physician shall, while caring for a patient, regard
responsibility to the patient as paramount.”

Reference:
AMA code of ethics, VIII.

70 | American Society for Surgery of the Hand


144. During surgery, requested equipment that is needed Discussion: Ignoring minor infractions that are self-
for the surgery is not available, but the equipment correcting is often a reasonable course. However,
is required to proceed with the case. Your assistant this issue may not be self-correcting, and the
goes to get the equipment, but the hospital administration will see this issue as a major potential
employee in the supply area is on break and refuses liability. Ignoring what they perceive to be a critical
to look for the equipment. Your assistant engages problem will reduce one’s credibility with the
in an argument with the employee using language administration, and reduce one’s overall effectiveness.
that violates hospital nondiscrimination policy. The However, the response to the problem must be fair
hospital employee reports the language used to and cannot focus on punishing only those lowest
the hospital administration. Your most appropriate in the hierarchy, or one will also lose credibility and
response would be to: effectiveness with employees. Further, even though
the administration wants complete capitulation and will
A. Ignore the issue
in the short term reward throwing the assistant to the
B. Apologize to the administration for your assistant’s wolves, in the long-term, even a hospital administrator
behavior, inform the administration that the assistant will see this action as pusillanimous and one will become
will be terminated, and institute sensitivity training ultimately less effective with the administration as well
for all of your remaining employees
as employees. Answer D allows the problem with the
C. Have the head of the Human Resources Department assistant to be addressed, allows the problem with
engage in conflict resolution between your assistant the hospital employee to be addressed, allows the
and the hospital employee, allowing your assistant system problems that contributed to the confrontation
to confront and resolve the underlying issues of to be addressed, and reframes an adversarial conflict
their prejudice regarding who is to blame into a cooperative approach
D. Apologize for your assistant’s lack of to a communal goal: improving patient care.
professionalism, state that the assistant will
be disciplined, but also state clearly that the References:
professionalism of everyone involved needs 1. Katz JD. Conflict and its resolution in the operating room. J Clin
scrutiny, and that the system problems allowing this Anesth. 2007; 19(2):152-8.
issue to occur must be addressed 2. Fisher R, Ury W. Getting to yes: negotiating agreement without
E. Demonstrate loyalty to your assistant and publicly giving in. New York: Penguin Books 1992.
back his or her behavior, while privately counseling
your assistant about his or her behavior

Preferred Response: D

2009 Self-Assessment Examination | 71


145. A 57 year-old female has a six month history of 146. The 44 year-old woman in Figure 1 complains of hand
pain and mild swelling about the index MCP joint. pain and stiffness. What is the most likely diagnosis:
There is no history of trauma. The arc of motion
A. Giant cell tumor of tendon sheath
is 10º extension to 80º flexion. Her x-rays are
normal. Her laboratory studies, including a CBC, B. Ganglion cyst
sedimentation rate, ANA, and uric acid are all C. Pseudogout
within normal limits. Her MRI is shown in Figure 1 D. CREST syndrome
below. The most likely diagnosis would be:
E. Psoriatic arthritis
A. Intraosseous lipoma
Preferred Response: D
B. Intraosseous ganglion
C. Giant cell tumor Discussion: The figures show multiple nodules of
cutaneous calcification in a patient with CREST
D. Avascular necrosis
syndrome, a type of systemic sclerosis, also known
E. Metastatic tumor as scleroderma. The two clinical subsets, limited
cutaneous and diffuse cutaneous, are distinguished
Preferred Response: D
by the extent of skin involvement, antibody profile and
Discussion: Dieterich’s disease, or avascular necrosis pattern of organ involvement.
of the metacarpal head, is an unusual finding which may
Common procedures include resection of calcifications,
present in early stages with normal appearing x-rays.
first web space release, MP joint releases or MP
This may be seen in patients with a history of trauma,
resection arthroplasty. The most common deformity is
steroid use, or SLE, but can also be idiopathic. The
advanced flexion deformity of the PIP joints. This has
long finger metacarpal is most commonly involved, but
been treated successfully in multiple series with PIP
involvement of all metacarpals have been reported. The
fusion. PIP arthroplasty has been associated with poor
MRI images will show hypointensity in the metacarpal
head on the T1 images, usually in the subarticular area, range of motion and wound healing problems, sometimes
and hyperintensity on the T2 images with marrow edema. requiring prosthesis removal.
Other diagnostic possibilities, based on the MRI findings, References:
would include infection, which would be unlikely with 1. Bogoch ER, Gross DK. Surgery of the hand in patients with
these laboratory studies. systemic sclerosis. J Rheumatol. 2005; 32(4):642-48.

References: 2. Nalebuff EA. Surgery in patients with systemic sclerosis of the


hand. Clin Orthop Relat Res 1999; 366:91-97.
1. Thienpart E, Vandsande W, De Smet L. Dieterich’s Disease:
Avascular necrosis of the metacarpal head: a case report. Acta 3. Gilbart MK, Jolles BM, Lee P, Bogoch ER. Surgery of the hand in
Orthop Belgica 2001; 67(2):182-184. severe systemic sclerosis. J Hand Surg 2004; 29B; 6:599-603.
2. Barrueco J, Zapatero M, Lignereux Y, Regodon S.
Osteochondritis dissecans of the head of the second metacarpal
bone. J Hand Surg 1992; 17A:1079-81.

3. Karlakki SL, Bindra RR. Idiopathic avascular necrosis of the


metacarpal head. Clin Orthop. 2003; 406:103-108.

Question 146, Figure 1

Question 145, Figure 1

72 | American Society for Surgery of the Hand


Discussion: This patient has a partial tear of the triceps.
It involves the long head, which arises from the scapula
and then joins the lateral head to form the superficial
muscular sleeve, which attaches onto the triceps tendon.
The medial (or deep) head of the triceps muscle arises
from the posterior humerus distal to the spiral groove.
The medial head sits deep to the triceps aponeurosis
which is created by the confluence of the more superficial
lateral and long heads. With partial tears, the insertion of
the aponeurosis on the olecranon may be intact.

The medial head inserts by a separate tendon near the


olecranon. Testing for weakness with the elbow flexed
more than 90° may help suggest a partial tear of the
medial head. Triceps tears can be confirmed by MRI.
Early surgical repair, preferably within three weeks, is the
recommended treatment of triceps tendon injuries.

References:
Question 146, Figure 2 1. Madsen M, Marx RG, Millett PJ, Rodeo SA, Sperling JW,
Warren RF. Surgical anatomy of the triceps brachi tendon:
anatomical study and clinical correlation. Am J Sports Med 2006;
34(11):1839-43.
147. A 22 year-old patient presents with pain in the
posterior arm after an injury 4 weeks prior. He 2. Van Reit RP, Morrey BF, Ho E, O’Driscoll SW. Surgical treatment of
distal triceps ruptures. J Bone Joint Surg 2003; 85A(10):1961-67.
was lifting weights and felt a pop in the arm.
He complains of persistent pain and weakness. 3. Kibuule LK, Fehringer EV. Distal triceps tendon rupture and
His exam shows swelling and tenderness in the repair in an otherwise healthy pediatric patient: a case report and
review of the literature. J Shld Elbow Surg 2007; 16:e1-e3.
muscular portion of the posterior arm, with a
palpable defect medially. He has weakness with
extension against resistance. His x-ray is normal,
and an MRI is shown in Figures 1 and 2. His injury
occurred in which head of the triceps:
A. Long
B. Lateral
C. Radial
Question 147, Figure 2
D. Deep
E. Medial

Preferred Response: A

Question 147, Figure 1

2009 Self-Assessment Examination | 73


148. A 34 year-old police officer presents with
right wrist pain after a fall from a ladder.  His
neurovascular exam is intact. His x-rays are
shown in the Figures below. The best treatment
would consist of:
A. Closed reduction with long arm cast
B. Closed reduction with external fixation
C. Closed reduction with percutaneous pin fixation
D. Open reduction internal fixation
E. Open reduction with fragment excision

Preferred Response: D

Discussion: The radiographs show an intra-articular


coronal fracture involving the volar margin of the radius,
with displacement of the carpus due to loss of the
buttress support of the volar rim. Because this fracture
is inherently unstable, proper treatment requires open
reduction of the volar fragment to restore articular Question 148, Figure 1
congruity and proper support for the volar carpus.
Disruption of the volar ligaments, particularly the long and
short radiolunate ligaments, may be a component of the
fracture and require repair if found to be unstable after
fracture reduction and fixation.

References:
1. Ilyas AM, Mudgal CS. Radiocarpal fracture-dislocations. J Am
Acad Orthop Surg 2008; 16:647-55.

2. Harness NG, Jupiter JB, Orbay JL, Raskin KB, Fernandez DL.
Loss of fixation of the volar lunate facet fragment in the fractures
of the distal part of the radius. J Bone and Joint Surg 2004;
86A(9):1900-08.

3. Chin KR, Jupiter JB. Wire loop fixation of volar displaced


osteochondral fractures of the distal radius. J Hand Surg 1999;
24A:525-33.

Question 148, Figure 2

74 | American Society for Surgery of the Hand


149. When comparing in-situ ulnar nerve 150. Pathophysiologic changes resulting from
decompression versus anterior transposition in peripheral nerve compression include:
the surgical treatment of cubital tunnel syndrome,
A. Wallerian degeneration
anterior transposition affords:
B. Increased endoneurial fluid pressure
A. Improved sensory recovery
C. Increased retrograde axonal transport
B. Decreased sensory recovery
D. Schwann cell proliferation
C. Improved motor recovery
E. Thickening of the myelin sheath
D. Decreased motor recovery
Preferred Response: B
E. No difference in motor or sensory recovery
Discussion: A compressed peripheral nerve undergoes
Preferred Response: E
multiple changes. The severity is dependent on the
Discussion: Several recent studies (therapeutic level I or magnitude, duration and character of the compressive
II) have looked at meta-analysis of randomized, controlled trauma. Tissue pressures above 20 mm can limit
trials to evaluate the efficacy of simple decompression epineural blood flow, creating ischemic changes including
versus anterior transposition for the treatment of cubital thickening of the walls of the plexus of microvessels.
tunnel syndrome. Although one study noted a trend Endoneurial fluid pressure is increased (rather than
toward improved clinical outcome with transposition of the decreased) and confined because of the diffusion barrier
ulnar nerve, either subcutaneous or submuscular, none of in the perineural membrane. This increased endoneurial
the randomized studies showed a statistically significant pressure interferes with endoneurial microcirculation,
difference between simple decompression and any type inhibiting blood flow to the fasicles.
of transposition. One investigator recommended simple
Pressures of 30 mm will limit axonal transport—both
decompression in cases without deformity of the elbow as
anterograde and retrograde. Pressures of 50 mm can
it is a less invasive operative procedure.
alter the structure of the myelin sheath. Other changes
References: include increased fibrosis in the perineural tissues, with
1. Macadam SA, Gandhi R, Bezuhly M, Lefaivre K. Simple impaired nerve gliding. All of these changes will adversely
decompression versus anterior subcutaneous and submuscular affect nerve function.
transposition of the ulnar nerve for cubital tunnel syndrome: a
meta-analysis. J Hand Surg 2008; 33A:1314-24. References:
2. Zlowoddzki M, Chan S, Bhandari M, Kallianen L, Schubert W. 1. Rempel D, Dahlin L, Lundborg G. Pathophysiology of nerve
Anterior transposition compared with simple decompression compression syndromes: response of peripheral nerves to
for the treatment of cubital tunnel syndrome: a meta-analysis of loading. J Bone Joint Surg 1999; 81A(11):1600-10.
randomized controlled trials. J Bone Joint Surg 2007; 89:2591-8. 2. Lundborg G, Dahlin LB. Anatomy, function and physiology of
3. Nabhan A, Ahlhelm F, Kelm J, Reith W, Schwendtfeger K, peripheral nerves and nerve compression. Hand Clin 1996;
Steudel WI. Simple decompression or subcutaneous anterior 12(6):185-92.
transposition of the ulnar nerve for cubital tunnel syndrome.
J Hand Surg 2005; 30B(35):521-4.

2009 Self-Assessment Examination | 75


Notes

76 | American Society for Surgery of the Hand


6300 North River Road, Suite 600
Rosemont, IL 60018-4256
Phone: (847) 384-8300
Fax: (847) 384-1435
E-mail: Info@assh.org
www.assh.org

Das könnte Ihnen auch gefallen